Pretest Neurology

अब Quizwiz के साथ अपने होमवर्क और परीक्षाओं को एस करें!

A 26-year-old woman weighing in excess of 300 lb has headache and blurred vision that began 2 weeks prior to consulting a physician. She has no vomiting or diplopia. Examination of her eyes reveals florid papilledema but without hemorrhages. Her neurologic examination is otherwise entirely normal. She had a similar problem while pregnant with her fourth child.

. The answer is a. (Ropper, pp 601-605.) This woman with headaches, papilledema, and slightly blurred vision probably has pseudotumor cerebri. This idiopathic increase in ICP usually occurs in obese young women, during pregnancy, or with hypervitaminosis. The extraordinarily high CSF opening pressure associated with pseudotumor cerebri does not produce herniation of the brain, and performing a spinal tap does not place the patient at increased risk for transforaminal herniation. In order to accurately measure pressure, the lumbar puncture must be performed with the patient in the lateral recumbent position or the patient placed in that position after the CSF is accessed. The CSF glucose content, protein content, cell count, and gamma globulin studies in persons with pseudotumor cerebri should all be unremarkable. Treatment most commonly involves weight loss, diuretics, and serial lumbar punctures. Brain MRI will be normal.

A 53-year-old office worker presents to clinic stating, "My hands are numb." Upon questioning she says that both of her hands have a sensation like "a shot at the dentist." She also believes that her hands are weaker than they used to be. It fluctuates during the course of the day, but is worst at the end of the work day and during the early morning hours. The symptoms have been progressively worsening over the past 1-to-2 years. On examination the abductor pollicus brevis is weak bilaterally, and there is decreased sensation to pinprick over the anterior portions of digits 1-to-3. Which of the following will most likely be positive? a. Tinel sign b. Brudzinski sign c. Kernig sign d. Monrad-Krohn test e. Babinski sign

The answer is a. (Biller, pp 394-396.) The patient presentation is typical for carpal tunnel syndrome. Tinel sign is the sensation of "tingling" radiating away from the percutaneous percussion of a peripheral nerve. For carpal tunnel syndrome Tinel sign of the median nerve at the wrist has ~60% sensitivity and ~67% specificity. Brudzinski and Kernig signs are indications of meningeal irritation. Monrad-Krohn test is used to confirm psychogenic upper extremity monoparesis. Babinski sign is an indication of upper motor neuron damage.

A 65-year-old man has had many years of deteriorating kidney function caused by diabetes. At age 59, dialysis was begun because of electrolyte abnormalities. Which of the following is the most common neurological complication of chronic renal failure? a. Peripheral neuropathy b. Delirium c. Seizures d. Dementia e. Labile affect

The answer is a. (Fauci, p 269.) The type of peripheral neuropathy most commonly developing with chronic renal failure is a symmetric, distal, mixed sensorimotor neuropathy. The legs are generally affected first and most severely. Men are more commonly affected than women. Most of the peripheral neuropathies in patients with chronic renal failure involve axonal degeneration. The neuropathy usually improves with dialysis.

In this MRI scan, the site most likely to produce a noncommunicating hydrocephalus when it is obstructed is identified by which of the following? a. Arrow A b. Arrow B c. Arrow C d. Arrow D e. Arrow E

The answer is a. (Osborn, pp 14-27.) Arrow A denotes the aqueduct of Sylvius, which connects the third ventricle with the fourth ventricle. This sagittal view of the lower part of the brain provides a high-resolution view of the posterior fossa. Noncommunicating hydrocephalus is characterized by a blockage of the ventricular system, which may be congenital or acquired. A communicating hydrocephalus is due to a mismatch between CSF formation and absorption rather than a blockage of flow. What appears to be a connection between the most inferior aspect of the fourth ventricle and the cisterna magna (Arrow E) is an artifact. This is the obex of the fourth ventricle, and there is a complete roof over this ventricle, which communicates with the subarachnoid space through the foramens of Luschka and Magendie.

A 46-year-old woman with depression has a brain CT performed at the request of her psychiatrist. There is the incidental finding of a dense mass that appears to originate from the tentorium cerebelli. The tentorium cerebelli separates the superior cerebellum from the cerebrum and is a common site of origin for which of the following? a. Meningiomas b. Ependymomas c. Hemangioblastomas d. Medulloblastomas e. Astrocytomas

The answer is a. (Osborn, pp II[4]:72-77.) All five of the tumors listed are common in the posterior fossa. The tentorium cerebelli is a fold of meninges. Consequently, it is a relatively common site for the development of meningiomas. A tumor arising on the tentorium may extend either superiorly or inferiorly. Inferior extension of the tumor may damage CNs and make complete extirpation of this benign neoplasm impossible. An ependymoma is a type of brain tumor that begins in the choroid plexus. A hemangioblastoma is a usually benign vascular tumor. An astrocytoma is composed of glial cells and is intermixed with healthy brain tissue. Medulloblastomas are a type of primitive neuroectodermal tumor (PNET) that occurs in the posterior fossa.

A 72-year-old man slipped and fell in the bathroom 1 week ago. He hit the right side of his head, but did not think it was necessary to seek medical attention. He finally goes to his doctor because his son thinks his balance is off. Computed tomography (CT) of the brain may fail to reveal a small subdural hematoma in this patient for which of the following reasons? a. The lesion is subacute b. The hematoma extends into the brain from the subdural space c. The resolution of the CT machine is greater than 2 mm d. The subdural hematoma is less than 4 hours old e. The patient has extensive cerebral atrophy

The answer is a. (Osborn, pp I[2]:26-27.) Initially subdural blood will be denser than brain and thus readily apparent on CT scan. Within a few days of formation, the contents of a subdural hematoma are degraded into less dense fluid. This fluid is transiently similar in density to the cerebral cortex and may be difficult to distinguish by CT. Eventually it will be hypodense to brain. If the fluid collection is too small to produce substantial deformation of the underlying hemisphere, identification of the subdural collection may be difficult. Angiogram will reveal displacement of the cerebrocortical vessels, but more rapid and less invasive assessment of the patient is feasible with MRI.

A 36-year-old man with intractable complex partial seizures and mesial temporal sclerosis (MTS) undergoes left temporal lobectomy. He is most likely to develop which of the following problems after surgery? a. Right superior quadrantanopsia b. Right inferior quadrantanopsia c. Right homonymous hemianopsia d. Right hand weakness e. Aphasia

The answer is a. (Patten, p 334.) The most common complication of temporal lobectomy is a visual field defect caused by interruption of fibers from the optic tracts passing over the temporal horn of the lateral ventricles. Superior quadrantanopsia is more common than hemianopsia. Some deficits may improve if the injury does not completely damage the nerves. Language deficits, particularly dysnomia, occur less frequently. Hemiparesis is uncommon (< 2%), because the surgery is performed at a distance from the motor fibers of the corticospinal tract. Other neurological problems that can occur include diplopia caused by extraocular nerve deficits and facial paresis.

A 26-year-old graduate student presents to the emergency room with a severe left-sided throbbing headache associated with nausea, vomiting, and photophobia. She has tried taking ibuprofen without relief. On further questioning, she relates that she has been having similar headaches three to four times per month for the past year. Her mother had a similar problem. Her examination is normal. Immediate therapy for this patient's present headache might include which of the following drugs? a. Sumatriptan b. Nitroglycerine c. Verapamil d. Amitriptyline hydrochloride e. Phenobarbital

The answer is a. (Ropper, p 172.) This patient has common migraine. Of the agents listed, only sumatriptan is generally considered of use to abort a headache. The triptans are a group of medications that act as agonists at serotonergic receptors (specifically, the 5HT-1 receptors), and they have been found to be very effective at stopping migraine headaches. Vera-pamil and amitriptyline hydrochloride may be used as prophylactic (preventative) therapy. Phenobarbital is an anticonvulsant and is not typically used to treat migraine. Nitroglycerine can actually precipitate headaches in susceptible individuals. Nausea is a frequent accompaniment of migraine. Metoclopramide hydrochloride may be effective in relieving the nausea, but it also reduces gastric stasis, which can retard absorption of oral medications. Certain antiemetics, such as prochlorperazine, may relieve nausea and also provide relief from the headache itself. Additional agents that might be of benefit in abortive therapy include ibuprofen, aspirin, acetaminophen, isometheptene, or ergotamine.

A 28-year-old police officer has been generally healthy except for mild, easily controlled hypertension. He sustains a gunshot wound to the upper arm. This type of trauma may cause partial damage to the median nerve that may leave the patient with which of the following? a. Easily provoked pain in the hand b. Weakness on wrist extension c. Atrophy in the first dorsal interosseous muscle d. Numbness over the fifth digit e. Radial deviation of the hand

The answer is a. (Ropper, pp 137-138.) Trauma to nerves in the extremities may give rise to causalgia, a disturbance in sensory perception characterized by hypesthesia, dysesthesia, and allodynia. Hypesthesia is a decrease in the accurate perception of stimuli. Dysesthesia is persistent discomfort, which in the situation described is likely to be an unremitting burning pain. Allodynia is the perception of pain with the application of nonpainful stimuli. Bullets and other high-velocity missiles need not hit the nerve to cause damage. Enough energy is transmitted as the missile passes through adjacent tissues to produce substantial damage to the nerve. Choices b through d involve motor or sensory findings due to either ulnar or radial nerve damage.

An 81-year-old woman with a history of type 2 diabetes mellitus and atrial fibrillation presents with right body weakness and slurred speech. She realized that there was a problem on awakening in the morning, and her husband called emergency medical services (EMS), who brought her to the emergency room. There are no word-finding difficulties, dysesthesia, or headaches. She is taking warfarin. Physical examination findings include blood pressure of 210/95 and irregularly irregular heartbeat. There is leftside neglect with slurred speech. There is a corticospinal pattern of weakness of the right body, with the face and upper extremity worse than the lower extremity. Routine chemistries and cell counts are normal. Her INR is 1.7. Computed tomography (CT) of the head reveals a large right-sided subdural hematoma. The intracranial material appearing most dense on CT is which of the following? a. Blood clot b. White matter c. Gray matter d. Cerebrospinal fluid (CSF) e. Pia mater

The answer is a. (Ropper, p 19.) CT scanning measures the density of intracranial as well as extracranial structures. Bone appears much denser than blood, but blood is obvious on the unenhanced (precontrast) CT scan precisely because it is much denser than white matter, gray matter, and CSF. The resolution of the CT scan is generally not sufficient to differentiate the pia mater from the gray matter on which it lies. Other meningeal structures, such as the dura mater, may appear denser than brain, especially if there is some calcification in the membranes.

Upon routine examination by her pediatrician, a 9-year-old girl is discovered to have precocious puberty. More careful examination discovers the presence of papilledema. This patient is most likely to have which of the following? a. A pineal region tumor b. An oligodendroglioma c. A Kernohan class II astrocytoma d. A brainstem glioma e. An ependymoma

The answer is a. (Ropper, p 641.) The pineal region is the source of an extraordinarily diverse group of tumor types, ranging from astrocytomas (derived from glial tissue) to chemodectomas (derived from sympathetic nervous tissue). Several different types of germ cell tumors arise from the tissues in this region, presumably from embryonal cell rests. In the United States, pineal tumors account for only 1% of intracranial tumors, but one-third of these pineal tumors are germ cell tumors, including germinomas and choriocarcinomas.

A 17-year-old girl presents initially with fever and progressive weakness. An extensive neurological evaluation including electromyography (EMG)/nerve conduction studies (NCS) suggests a motor neuron disease. The motor neuron disease most certainly traced to a virus is which of the following? a. Poliomyelitis b. Subacute sclerosing panencephalitis (SSPE) c. Progressive multifocal leukoencephalopathy (PML) d. Subacute human immunodeficiency virus (HIV) encephalomyelitis e. Kuru

The answer is a. (Ropper, p 712.) SSPE, PML, kuru, and HIV encephalomyelitis are all viral diseases affecting the CNS, but poliomyelitis is the only one that causes a purely motor neuron disease. Poliomyelitis virus attacks the anterior horn cells in the spinal cord. It is most likely to be confused with Guillain-Barré syndrome if the typical CSF picture of a viral meningoencephalitis is not found with the progressive motor neuron impairment. With poliomyelitis, the CSF will usually exhibit an elevated protein and white cell count. During the initial stages of the infection, the patient will usually have fever.

Which of the following types of visual field cuts is most often seen with vitamin B12 deficiency? a. Centrocecal scotoma b. Homonymous hemianopsia c. Bitemporal hemianopsia d. Binasal hemianopsia e. Hemianopsia with central sparing

The answer is a. (Ropper, pp 1117-1121 .) The blind spot that normally occurs in each eye enlarges and extends temporally to involve central vision in patients with chronic vitamin B12 deficiency. This is similar to the blind spot that is associated with alcohol and tobacco excess, a problem called tobacco-alcohol amblyopia. Tobacco-alcohol amblyopia also seems to develop because of a vitamin B deficiency, but the deficiency is presumed to be of thiamine rather than of cobalamin.

A 42-year-old woman is being evaluated for gait difficulties. On examination, it is found that her ability to walk along a straight line touching the heel of one foot to the toe of the other is impaired. This finding is most common with which of the following? a. Cerebellar dysfunction b. Parietal lobe damage c. Temporal lobe damage d. Ocular motor disturbances e. Dysesthesias in the feet

The answer is a. (Ropper, pp 113-114 .) Walking along a straight line with the heel of one foot touching the toe of the other foot is called heel-to-toe walking, or tandem gait. It is a routine test for ethanol intoxication because alcohol exposure impairs the coordination of gait as governed by the cerebellum. Tandem gait will be abnormal with many other problems, including weakness, poor position sense, vertigo, and leg tremors, but such abnormality in the absence of these other problems suggests a cerebellar basis for the problem.

A 31-year-old right-handed woman has a history of alcohol abuse requiring detox. Currently, she says she is drinking about nine beers 3 days per week. She drank five glasses of wine and three beers 5 days ago. Last night, she had 10 beers. This morning, she awoke feeling well. She was speaking with her fiancé, went to the bathroom, and got back into bed. She had no headache, fever, chills, nausea, vomiting, or pain. Suddenly her body became stiff with arms flexed for a few seconds, followed by rhythmic jerking of both arms. Her legs were shaking, but less so. Her eyes were open, and she was foaming at the mouth. After 1 minute, this stopped, and she initially did not recognize her fiancé or his sister. She slowly returned to a normal level of consciousness over a 10-minute period. She remembers events just prior to the episode, and she remembers being in the car on the way to the hospital. Her only medication is a multivitamin. She denies illicit drugs. Her examination is entirely normal. Routine labs and a brain MRI are normal. Following cessation of drinking, what is the peak time period for alcohol withdrawal seizures? a. 1-to-2 days b. 2-to-3 days c. 3-to-4 days d. 4-to-5 days e. 5-to-6 days

The answer is a. (Ropper, pp 1135-1139 .) This is a typical example of alcohol withdrawal seizure. The greatest risk for alcohol withdrawal seizures occurs within the first day after drinking cessation, in contrast to delirium tremens, which usually occurs within 2-to-4 days of drinking cessation. By 7-to-14 days all signs of withdrawal have largely resolved.

A 22-year-old dance instructor routinely develops headaches on the weekend. The headaches are almost always limited to the right side of her head and centered about the right temple. She knows that a headache is coming because of changes in her vision that precede the headache by 20-to-30 minutes. She sees scintillating lights just to the left of her center of vision. This visual aberration then expands and interferes with her vision. The blind spot that it creates appears to have a scintillating margin. As the blind spot clears, the headache starts. It rarely lasts more than 1 hour, but is usually accompanied by nausea and vomiting.

The answer is a. (Ropper, pp 166-174.) Classic migraine is usually familial, involves a unilateral, throbbing head pain, and diminishes in frequency with age. The blind spot, or scotoma, that may develop as part of the aura of a classic migraine attack will involve the same visual field in both eyes. This defect usually changes over the course of minutes. It typically enlarges and may intrude on the central vision. The margin of the blind spot is often scintillating or dazzling. If this margin has a pattern like the battlement of a castle, it is called a fortification spectrum, or teichopsia. Homonymous hemianoptic defects of the sort that develop during the aura of a classic migraine indicate an irritative lesion that is affecting one part of the occipital cortex in one hemisphere of the brain. The changes in the scotoma over the course of minutes indicate that the irritative phenomenon sets off a cascade of events in the visual cortex that temporarily disturbs vision in a progressively larger area. Other focal neurological phenomena may precede classic migraine; the most common are tingling of the face or hand, mild confusion, transient hemiparesis, and ataxia. Fatigue, irritability, and easy distractibility often develop before a migraine. Affected persons usually also have hypersensitivity to light and noise during an attack

A 32-year-old woman is being evaluated for headaches. They started about 6 months ago and occur a few times per week, lasting until she falls asleep. The pain is constant and focused at the front and back of the head. The pain is unrelated to position and tends to be worse later in the day. There is mild photophobia. Which of the following findings is most likely? a. Slightly reduced neck range of motion and paracervical tenderness b. Papilledema c. Abnormal brain magnetic resonance imaging (MRI) d. Abnormal brain computed tomography (CT) e. Abnormal electroencephalogram (EEG)

The answer is a. (Ropper, pp 176-177.) The history is typical for a tension-type headache. These headaches are often associated with neck muscle spasm leading to reduced neck range of motion and paracervical tenderness. Papilledema and neuroimaging abnormalities would be associated with headaches caused by an intracranial mass. The EEG is generally normal in patients with headaches, unless there is underlying damaged brain.

A 56-year-old man is struck over the parietal area of the head during a robbery. He loses consciousness for 35 minutes but has no focal weakness or numbness on regaining consciousness. Within 2 days of the incident, his wife finds him unresponsive in bed early in the morning. She calls for an ambulance, but before it arrives her husband becomes more alert and asks for something to eat, saying he wants to have some supper before he goes to bed for the night. The ambulance attendant first on the scene notes that the patient is disoriented to place and time and has weakness of his right arm and leg.

The answer is a. (Ropper, pp 304-335.) After significant head trauma, the victim is at considerable risk for a seizure. A patient's seizure threshold is lowest when he or she is asleep or sleep-deprived. If the posttraumatic seizure occurs during sleep, it may go unnoticed. The patient's improving cognition suggests a postictal state. His hemiparesis is probably a Todd paralysis, but any patient with posttraumatic seizures and focal weakness must be investigated for an acute or chronic subdural hematoma.

A 21-year-old man reports several episodes over the previous 4 years during which he lost consciousness. He had no warning of the impending episodes, and with each episode he injured himself. Observers told him that he abruptly developed a blank stare and stopped talking. His body became stiff and he arched his back. After several seconds of this type of posturing, his arms and legs started shaking violently. During one of these episodes, he dislocated his right shoulder. He routinely bit his tongue and urinated in his pants during the episodes

The answer is a. (Ropper, pp 305-308.) With generalized tonic-clonic seizures, the EEG develops abnormalities all over the cortex simultaneously. The patient may recall a strange sensation before the attack, but it is equally likely that no premonitory sign or aura will occur. Partial seizures may secondarily generalize to this type of seizure. If the patient has frequent generalized tonic-clonic seizures, he or she will be at high risk for a variety of injuries, such as dislocated shoulders, broken bones, and head trauma. Patients with this type of seizure always lose consciousness during the attack and may be confused for minutes or hours after the ictus, the most obvious segment of the seizure.

A 29-year-old man with a history of febrile seizures as a child has developed medication refractory complex partial seizures within the past 2 years. An MRI reveals the abnormality indicated by the arrow. Which of the following is true regarding this condition? a. This patient may benefit from a neurosurgical procedure. b. The patient will probably die within 2 years. c. The seizures will most likely stop with further medication titration. d. A head computed tomography (CT) should be performed. e. A cerebral angiogram may confirm the diagnosis.

The answer is a. (Ropper, pp 310-313.) The history and MRI are typical for MTS. The arrow in the MRI is specifically pointing at the sclerotic right hippocampus. This is the most common cause of intractable complex partial seizures in adults. The prognosis for improved seizure control with additional medications is poor; however, surgical resection of the right anterior temporal lobe may produce seizure freedom in up to 80% of cases. If this patient had a high-grade malignant brain tumor, he would probably die within 2 years. A cerebral angiogram may confirm the diagnosis of a vascular malformation.

A 35-year-old woman has worsening intermittent headaches and "dizzy spells" over the course of several months. Examination in between attacks is normal. An MRI reveals a mass in the posterior fossa suggestive of an ependymoma. This patient is potentially at risk of dying because of which of the following? a. Transforaminal herniation b. Emboli from the tumor c. Vascular occlusion by the tumor d. Hemorrhagic necrosis of the tumor e. Status epilepticus

The answer is a. (Ropper, pp 349-351.) As a tumor of the posterior fossa enlarges, the contents of the posterior fossa will be compressed and ultimately forced upward or downward. If the herniation is upward, it is called transtentorial because it is across the tentorium cerebelli. If it is downward, it is called transforaminal because it is across the foramen magnum. Ependymomas are not especially vulnerable to hemorrhagic necrosis. Tumors in the posterior fossa generally do not produce seizures.

A 62-year-old man has had a left hemisphere stroke. He has impaired naming and repetition. His speech is nonfluent. Comprehension is preserved.

The answer is a. (Ropper, pp 466-470.) Broca is the classic anterior (nonfluent) aphasia and is characterized as described in the question. It is most often associated with a lesion of the left inferior frontal gyrus.

A 24-year-old woman abruptly loses all speech during the third trimester of an otherwise uncomplicated pregnancy. She has a history of severe migraines during which she occasionally develops a transient right hemiplegia. Her comprehension is good, and she is frustrated by her inability to speak or write. She is unable to repeat simple phrases, but she does begin to produce simple words within 5 days of the acute disturbance of language.

The answer is a. (Ropper, pp 466-470.) Cerebrovascular occlusions are unusual at the age of 24, but this woman had two risk factors for stroke: her migraine headaches and her pregnancy. The stroke probably involved the frontal lobe cortex about the third frontal convolution on the dominant side. Speech becomes telegraphic (ie, consisting of short phrases with omission of small connecting words such as articles and conjunctions) with a Broca aphasia, but permanent loss of all ability to produce meaningful language is unlikely if the area of infarction is less than a few centimeters across. The most persistent difficulty usually exhibited by patients with this type of stroke is a permanent loss of syntax.

A 38-year-old right-handed woman presented with early morning headaches. There was papilledema on examination. Below is shown a T1-weighted postcontrast brain MRI of the patient. Which of the following symptoms is most likely to also be present? a. Aphasia b. Neglect c. Left hemiparesis d. Left homonymous hemianopia e. Alexia without agraphia

The answer is a. (Ropper, pp 466-470.) The patient has a tumor in the left frontal-parietaltemporal region. This area is critical for language. This particular patient presented with Wernicketype aphasia (impaired naming, comprehension, and repetition with fluent speech). Neglect and left hemiparesis would be more likely with right brain lesions. A left homonymous hemianopia would be caused by an occipital lesion on the right side. Alexia without agraphia is a disconnection syndrome associated with lesions involving the left occipital lobe and splenium of the corpus callosum.

A 65-year-old right-handed woman began having neurological problems about 1 week ago. She began experiencing nausea, vomiting, and numbness in the left hand and left foot. Today she had a generalized convulsion, and since then she has had a throbbing headache that is worse when she bends forward. On examination, the only deficits she has are loss of double simultaneous tactile stimulation and left lower facial droop when smiling. MRI reveals a lesion suggestive of a primary brain tumor. Which of the following is the most common source of primary brain tumors? a. Glial cells b. Neurons c. Meningeal cells d. Lymphocytes e. Endothelial cells

The answer is a. (Ropper, pp 622-628.) Between 2% and 5% of all tumors occurring in the general population are primary CNS tumors. In adults, the most common primary brain tumor is the astrocytoma. In children, brain tumors are more likely to arise in the posterior fossa. Even in childhood, glial cell tumors, such as the cerebellar astrocytoma and the optic glioma, are common.

A 72-year-old woman has a head computed tomography (CT) performed because of headaches. It is significant for a left hemisphere mass with an overlying hyperostosis of the skull. She most likely has which of the following? a. Meningioma b. Pituitary adenoma c. Astrocytoma d. Schwannoma e. Hemangioblastoma

The answer is a. (Ropper, pp 628-630.) Hyperostosis is thickening of the bone and is much less commonly induced by tumors in or about the brain than is thinning of the bone. Thinning occurs especially with pituitary adenomas, which may cause erosions in the floor of the sella turcica as an early feature. Calcifications may develop in schwannomas or astrocytomas, but both of these tumor types will usually cause bony erosions where they impinge on the skull. Calcifications may develop in many primary or metastatic brain tumors, but calcification sufficient to be readily seen on a skull xray suggests an astrocytoma, meningioma, oligodendroglioma, or metastatic tumor. Calcification can be visualized on CT scan in about 17% of medulloblastomas. With meningiomas, hyperostosis may develop in the bone adjacent to the tumor even if there is no infiltration of the bone by the tumor.

A 29-year-old woman with a history of malignant melanoma presents to her primary care doctor with a new type of headache. Examination is normal, and a head CT is ordered. Multiple small brain hemorrhages are revealed by the scan. Metastatic lesions to the brain most often appear in which of the following locations? a. At the gray-white junction b. In the thalamus c. In the posterior fossa d. In the caudate e. In the sella turcica

The answer is a. (Ropper, pp 631-634.) Metastatic lesions are spread primarily by the vascular system. The gray-white junction (where the white matter and the gray matter meet) is the interface at which bloodborne cells are most likely to lodge and grow. No part of the brain is exempt from the spread of metastases, but the cerebral hemispheres and the cerebellum are especially vulnerable.

The shortest life expectancy with metastatic disease to the brain will be found in the patient with which of the following metastatic cancers? a. Malignant melanoma b. Breast cancer c. Lung cancer d. Renal cancer e. Prostate cancer

The answer is a. (Ropper, pp 631-634.) The outlook with malignant melanoma, breast cancer, lung cancer, or renal cancer metastatic to the brain is poor and limited to a matter of months, but malignant melanoma is especially grim because it is highly likely to bleed after it metastasizes to the brain. Malignant melanoma and choriocarcinoma are likely to produce lethal intracranial hemorrhages, and the former may in fact first become apparent only after it has precipitated an intracranial hemorrhage. Prostate cancer does not typically metastasize to the brain.

A 4-year-old boy presents with ataxia, lethargy, and obstructive hydrocephalus.

The answer is a. (Ropper, pp 637-638.) Medulloblastomas are one of the most common CNS tumors of childhood. They typically develop in the cerebellum, causing ataxia. Astrocytomas may also occur in children infratentorially, primarily in the cerebellum and brainstem. In either location, hydrocephalus may develop because of obstruction at the level of the fourth ventricle. The astrocytoma that develops in the cerebellum is usually cystic. Medulloblastomas are invariably infratentorial, at least initially. They may extend supratentorially or become disseminated supratentori-ally through seeding of cells carried in the CSF. Ependymomas, another common tumor type in children, are derived from the lining of the ventricles and also carry the risk of hydrocephalus and seeding throughout the CNS.

A 67-year-old woman has a 2-month history of progressive gait disturbance. On examination, she has dysmetria of the limbs; a wide-based, unsteady gait; and hypermetric saccades. A hard, firm breast lump is discovered.

The answer is a. (Ropper, pp 655-661.) Paraneoplastic cerebellar degeneration is characterized by subacute and relentlessly progressive ataxia, dysarthria, and nystagmus. Myoclonus, opsoclonus (irregular jerking of the eyes in all directions), diplopia, vertigo, and hearing loss may also occur. Imaging may eventually reveal cerebellar atrophy, and pathology will disclose loss of Purkinje cells in the cerebellum as the primary abnormality. The most common associated tumor types are small cell carcinoma of the lung, ovarian carcinoma, and lymphoma, in that order. Approximately 50% of patients may harbor anti-Purkinje cell antibodies (called anti-Yo antibodies), and these are especially commonly found in women with breast cancer or gynecologic malignancies. Interestingly, the symptoms of paraneoplastic cerebellar degeneration often precede the symptoms of the underlying tumor itself, leading to speculation that the immune reaction that damages the nervous system may, in fact, be protective against the tumor.

A 75-year-old left-handed woman presented to the emergency room with what at first was thought to be a stroke. History was significant for pneumonia 5 weeks ago. Following neuroimaging, the situation became less clear, and ultimately an enhancing brain lesion was aspirated via stereotaxic needle placement. Culture of the aspirate grew out bacteria. The most likely organism is which of the following? a. Streptococcal b. Staphylococcal c. Bacteroides spp. d. Proteus spp. e. Pseudomonas spp.

The answer is a. (Ropper, pp 683-686.) Both aerobic and anaerobic streptococcal bacteria occur in more than half of all brain abscesses. Staphylococcus aureus most often occurs in patients who have had penetrating head wounds or have undergone neurosurgical procedures. Enteric bacteria (eg, Escherichia coli, Proteus, and Pseudomonas) account for twice as many abscesses as S aureus.

A 41-year-old homosexual man is brought to medical attention by his partner because of headache, sluggish mentation, and impaired ambulation worsening over the previous week. The patient is known to be HIV seropositive, but has done well in the past and has not sought regular medical attention. On examination, his responses are slow and he has some difficulty sustaining attention. He has a right hemiparesis with increased reflexes on the right. Routine cell counts and chemistries are normal. A contrast head CT reveals several ring-enhancing lesions. Eventually, surgical aspiration of one of the lesions reveals that they are abscesses. Abscesses in the brain most often develop from which of the following? a. Hematogenous spread of infection b. Penetrating head wounds c. Superinfection of neoplastic foci d. Dental trauma e. Neurosurgical intervention

The answer is a. (Ropper, pp 683-686.) There are many bases for abscess formation in the brain, but the most frequent causes are blood-borne infections from sources in the lung, heart, sinuses, and ears. Extension of infection from a chronic otitis or mastoiditis was much more common before the introduction of antibiotics. Facial or dental infections may spread to the brain through valveless veins draining about the muscles of mastication and communicating with the venous drainage of the brain

A 52-year-old right-handed woman who has abused intravenous drugs for many years has an HIV antigen test that is positive. CD4+ T-lymphocyte count is normal. A brain CT scan reveals several hemorrhagic lesions. NCS reveal generalized slowing in the legs, and EEG exhibits focal slowing over the left parietal lobe. Cardiac catheterization suggests aortic valve disease, and the patient's CSF is xanthochromic. Which of the following is the probable site of injury in the central nervous system (CNS)? a. An arterial wall b. The ventricular endothelium c. The pia arachnoid d. The dura mater e. The perivenular space

The answer is a. (Ropper, pp 684, 816.) The most likely explanation for this patient's deficits is bleeding from a mycotic aneurysm. This type of aneurysm is usually relatively small and might not be evident on CT scanning or even on arteriography. An arteriogram would miss the lesion if it had destroyed itself when it bled or if the aneurysmal sac was completely thrombosed. The name mycotic is misleading. It suggests a fungal etiology, but it actually refers to the appearance of these aneurysms, which tend to be multiple. These aneurysms occur with either gram-positive or gram-negative infections, but the responsible organisms usually have relatively low virulence. Mycotic aneurysms form over the cerebral convexities with subacute bacterial endocarditis. The aneurysm develops from an infected embolus originating on the diseased heart valves and lodging in the arterial wall. Bleeding from these small aneurysms is largely directed into the subarachnoid space. More virulent organisms that produce valvular heart disease are more likely to produce a meningitis or multifocal brain abscess with seeding of infected emboli to the brain. With acquired immune deficiency syndrome (AIDS), a fungus could be the causative agent, but patients with endocarditis more typically have streptococcal or staphylococcal infections. Even if mycotic aneurysms form with endocarditis, they need not inevitably become symptomatic.

A 13-year-old boy is brought into the emergency room lethargic with a stiff neck and fever. Despite aggressive therapy, the child dies. Postmortem evaluation reveals that the child had primary amebic meningoencephalitis. This condition is usually acquired through which of the following means? a. Freshwater swimming b. Eating contaminated meat c. Eating calves' brains d. Anal intercourse e. Animal bites

The answer is a. (Ropper, pp 703-704.) Primary amebic meningoencephalitis is usually caused by organisms from the genera Hartmanella or Acanthamoeba. The parasites enter the nervous system through the cribriform plate at the perforations for the olfactory nerves. An especially lethal form of this meningoencephalitis may develop with Naegleria spp. Other parasites, such as S mansoni, may be acquired through swimming in contaminated freshwater, but it is unlikely that other parasites reach the nervous system through direct invasion across the cribriform plate. Schistosomiasis is acquired when the cercarial phase of the organism penetrates the swimmer's skin and finds its way into the blood.

A 67-year-old man presents with headache, fever, disorientation, and seizures. CSF testing establishes that the patient has the most common form of acute encephalitis. The CSF changes late in the course of this disease typically include which of the following? a. An increased number of lymphocytes b. A glucose content of less than two-thirds the serum level c. A protein content of less than 45 mg/dL d. A normal opening pressure e. A predominance of polymorphonuclear white blood cells (WBCs)

The answer is a. (Ropper, pp 719-721.) The increased number of lymphocytes in the CSF of the patient with herpes encephalitis ranges from more than 12 to several hundred cells per cubic millimeter of fluid. Red blood cells may be apparent in the CSF late in the course of the disease, but their absence does not eliminate the possibility of herpes encephalitis. CSF pressure is usually increased, and the glucose content is usually normal or only slightly depressed.

A 31-year-old homosexual man has had headache, sleepiness, and poor balance that have worsened over the past week. The patient is known to be HIV seropositive, but has done well in the past and has not seen a doctor in over 1 year. On examination, his responses are slow and he has some difficulty sustaining attention. He has a right hemiparesis with increased reflexes on the right. Routine cell counts and chemistries are normal. Which of the following is the most appropriate next step in management? a. Head computed tomography (CT) with contrast b. Noncontrast head CT c. Perform a lumbar puncture d. Start antiretroviral therapy e. Start intravenous heparin

The answer is a. (Ropper, pp 726-730.) The differential diagnosis is rather broad at this point. One should look for an infectious or malignant mass with a contrast-enhanced CT or MRI. A noncontrast head CT is less sensitive for abscess or tumor. A lumbar puncture should be done only after you are sure that there is not significant mass effect. This patient has an acute problem, which should be addressed now. Antiretroviral therapy will help him in the long term, but does not need to be initiated in the emergency room. Intravenous heparin is a treatment for embolic stroke. Embolic stroke is unlikely in this case, and further evaluation is needed before treatment with intravenous heparin is considered.

A 52-year-old generally healthy woman has had a gradual neurological deterioration over the past 6 to 8 months. It began with depression and a mild change in personality. Eventually she developed weakness and nonpurposeful movements of her left hand, as well as significant cognitive decline. All serologies were negative. MRI showed abnormal restricted diffusion in portions of the cortical gray matter and deep nuclei. Electroencephalography (EEG) had diffuse slowing and triphasic waves. Routine spinal fluid examination in this patient would be expected to show which of the following? a. No abnormalities on routine studies b. Elevated protein c. More than 100 lymphocytes d. More than 1000 red blood cells e. Decreased glucose

The answer is a. (Ropper, pp 737-741.) The spinal fluid examination in a patient with spongiform encephalopathy, or Creutzfeldt-Jakob disease, is typically normal. On occasion, the protein level may be mildly elevated, and, in up to 20% of cases, there may be an increase in the ratio of immunoglobulin G to total protein, occasionally with oligoclonal bands. In many, but not all patients, a somewhat specific protein (14-3-3 proteinase inhibitor) may be present.

A 60-year-old man is clinically suspected to have had a subarachnoid hemorrhage. A lumbar puncture shows 7000 red blood cells (RBCs) in tube 1 and 7200 in tube 4. There are nine WBCs in each. The fluid is xanthochromic. The opening pressure is 22 cm H2O. Which of the following is the next best step in managing this case? a. Arrange for a cerebral angiogram and call a neurosurgical consult b. Give the patient a prescription for sumatriptan and send him home c. Immediately give 2 g of intravenous ceftriaxone d. Immediately start intravenous acyclovir e. Repeat the lumbar puncture

The answer is a. (Ropper, pp 747-748, 808-816.) This patient probably has a subarachnoid hemorrhage and must be evaluated for an aneurysm. This does not appear to be bacterial meningitis. It is not emergent that ceftriaxone be given in this case. Sumatriptan is a treatment for migraine, and this patient's history and CSF results do not support a diagnosis of migraine. Repeating the lumbar puncture will not help with the diagnosis or treatment. Intravenous acyclovir would be used to treat herpes encephalitis. Although there are often RBCs in the spinal fluid of such patients, the overall history makes herpes encephalitis unlikely. If this had been a traumatic lumbar puncture, the number of RBCs would have declined from tube 1 to tube 4.

A 52-year-old woman with diabetes awakens with right body weakness. Examination at the hospital indicates relatively symmetric upper motor neuron pattern of weakness involving the face, arm, and leg. There are no sensory abnormalities. Language is preserved. A stroke associated with this presentation is most likely with damage to which of the following? a. Internal capsule b. Cerebellum c. Putamen d. Caudate e. Amygdala

The answer is a. (Ropper, pp 748-772.) Pure motor deficits are especially likely in hypertensive persons with small infarctions, called lacunae. The pure motor stroke is the most common type of lacunar stroke. The affected person usually has hemiplegia unassociated with cognitive, sensory, or visual deficits. The posterior limb of the internal capsule is the usual site of injury. The lacunae are assumed to develop because of an occlusive lesion in an arteriole that supplies the injured structure. Isolated lesions of the cerebellum may be associated with impaired coordination, but strength is generally preserved. Lesions of the caudate or putamen, which spare the internal capsule, will generally not cause weakness. They may be grossly asymptomatic or cause more subtle, possibly transient, cognitive or motor deficits. The amygdala is involved in memory formation and emotion

A 72-year-old retired school teacher has the abrupt onset of right face and hand weakness, disturbed speech production, and a right homonymous hemianopsia. This is most likely attributable to occlusion of which of the following arteries? a. Left middle cerebral artery b. Left anterior cerebral artery c. Left vertebrobasilar artery d. Right anterior choroidal artery e. Left posterior inferior cerebellar artery (PICA)

The answer is a. (Ropper, pp 748-772.) The left middle cerebral artery supplies the cortex around the sylvian fissure, as well as some of the frontal lobe structures involved in speech. The optic radiation loops through the temporal lobe on its way to the occipital cortex and is usually damaged with occlusion of the middle cerebral artery. The likely speech disorder with an injury of the left frontal lobe is a Broca aphasia. Comprehension would be expected to be largely intact, but if the patient has damage to enough of the temporal lobe cortex, a Wernicke aphasia might develop. Choroidal artery occlusions might produce focal weakness, but speech problems would be less likely. Occlusion of the PICA can produce a variety of brainstem and cerebellar signs, but this combination of deficits would be unlikely with a lesion outside the cerebral cortex. Anterior cerebral artery stroke would be expected to cause lower extremity weakness and would not affect vision, although it could result in decreased speech production. Vertebral or basilar artery occlusion would primarily affect brainstem or cerebellar structures. If either posterior cerebral artery had its blood supply compromised, there could be visual loss and a posterior (fluent) aphasia.

A 74-year-old man is diagnosed with an acute stroke. A right middle cerebral artery occlusion is demonstrated by magnetic resonance angio-gram shown below. Which of the following is the most likely cause of this patient's present condition? a. Atherosclerosis b. Fibromuscular dysplasia c. Mitral valve prolapse d. Arterial dissection e. Meningovascular inflammation

The answer is a. (Ropper, pp 773-778.) Atherosclerosis may produce cerebral infarction by a variety of mechanisms, including emboli to the brain and local occlusion of atheromatous vessels. Platelet emboli may form on ulcerated atheromatous plaques in major vessel walls and ascend to the brain. The atherosclerotic plaque involves subintimal proliferation of smooth muscle, fatty deposits in the intima, inflammatory cells, and excessive elaboration of the connective tissue matrix in the vessel wall. Thrombi may form on the surface of the plaque and occlude the vessel, even if the plaque is not large enough to produce substantial narrowing of the vessel. Fibromuscular dysplasia is a relatively uncommon cause of cranial vessel occlusion that develops with segmental overgrowth of fibrous and muscular tissue in the media. Meningovascular inflammation is a rare process that occurs in some infectious or inflammatory disorders, such as syphilis, tuberculous meningitis, or sarcoid.

A 52-year-old patient presents with headache and sudden onset of mania. Her head CT is shown below. Two hours later her blood pressure is 225/110 mm Hg, her heart rate is 40 beats per minute, and her consciousness is fluctuating. Which of the following is the best management over the next 4 hours for this patient? a. Craniotomy b. Antihypertensive medication c. Transvenous pacemaker placement d. Ventriculoperitoneal shunt e. Antiepileptic medication

The answer is a. (Ropper, pp 805-808.) Without emergency surgery, the patient will die. Her blood pressure and pulse abnormalities will correct themselves when the intracranial mass is removed. Her loss of consciousness will not correct itself with antiepileptics. Shunt placement will not likely prevent brain herniation and may in fact accelerate it. The hematoma must be evacuated, and the bleeding giving rise to the hematoma must be stopped. Immediate treatment should include elevation of head and hyperventilation.

A 23-year-old woman has had 1 week of worsening facial pain. She describes it as an intense shooting pain that comes and goes. It is present only on her right face. Which of the following is most likely to be this patient's underlying problem? a. Multiple sclerosis b. Tolosa-Hunt syndrome c. Migraine d. Anterior communicating artery aneurysm e. Falx meningioma

The answer is a. (Ropper, pp 883, 884.) Multiple sclerosis is often associated with trigeminal neuralgia, which is then termed symptomatic trigeminal neuralgia because it occurs as a symptom of another illness. This is caused by a plaque within the brainstem. Other causes of symptomatic trigeminal neuralgia include basilar artery aneurysms, acoustic schwannomas, and posterior fossa meningiomas, all of which may cause injury to the fifth cranial nerve by compression. The TolosaHunt syndrome is a presumably inflammatory disorder that produces ophthalmoplegia associated with headache and loss of sensation over the forehead. Pupillary function is usually spared, and the site of pathology is believed to be in the superior orbital fissure or the cavernous sinus. It is usually not associated with trigeminal neuralgia.

In Tay-Sachs disease, the enzymatic abnormality responsible for the neurological deficits is deficiency of which of the following? a. Hexosaminidase A b. Glucocerebrosidase c. Phosphofructokinase d. Glucose phosphorylase e. Sphingomyelinase

The answer is a. (Swaiman, pp 667-668.) Children with Tay-Sachs disease die prematurely and exhibit mental retardation, seizures, and blindness. This is a ganglioside storage disease that occurs more commonly in Ashkenazi Jews than in the general population. The early-onset form will produce macrocephaly and a cherry red spot in the fundus.

An 8-month-old boy develops spasticity, head retraction, and difficulty swallowing. His physician discovers an abnormal accumulation of glucosylceramide and tells the parents their child will continue to deteriorate and likely die within 3 years. This child has which of the following? a. β-glucosidase deficiency b. Niemann-Pick disease c. Krabbe disease d. Fabry disease e. Tay-Sachs disease

The answer is a. (Swaiman, pp 670-671.) The disease responsible for the accumulation of glucosylceramide is Gaucher disease. Gaucher disease is inherited as an autosomal recessive trait and may be diagnosed by demonstrating deficient glucocerebrosidase in fibroblasts or leukocytes. The severity of disease varies from nonneuronopathic types to acute infantile neuronopathic disease. Gaucher disease produces hepatosplenomegaly and may cause lethal CNS disease. It is one of a collection of storage diseases called sphingolipidoses, which include Niemann-Pick disease, Krabbe disease, and Fabry disease. Fabry disease involves the accumulation of another ceramide, trihexosylceramide. All the sphingolipids are nothing more than lipids that contain a sphingosine moiety. Sphingosine is a class of long-chain compounds with hydroxyl groups on carbons 1 and 3 and an amino group on carbon 2. They form ceramides by joining with fatty acids across the subterminal amino group. GM2 ganglioside accumulates in Tay-Sachs disease, and galactosyl sulfatides accumulate in metachromatic leukodystrophy.

A 46-year-old man complains of progressive visual problems. He notices problems with discriminating objects both up close and far away. His deficits have progressed over the course of 3 months. He has a 12-year history of pipe smoking, a 14-year history of daily aspirin use, and a 20- year history of alcohol intake. He usually drinks 4 oz of gin daily. Examination reveals enlargement of the physiologic blind spot to the point where it extends into central vision.

The answer is a. (deficiency amblyopia) (Ropper, pp 1122-1123 .) The vitamin deficiency specifically responsible for injury to the optic nerve in persons who chronically smoke tobacco and drink ethanol is still uncertain. It probably arises from combined deficits of vitamins B1 , B12 , and riboflavin. This condition is also known as nutritional optic neuropathy and as tobacco-alcohol amblyopia. There has been considerable speculation about its arising as a consequence of chronic cyanide poisoning from tobacco smoking combined with vitamin B12 deficiency associated with alcoholism, but this theory has little support.

A 68-year-old man presents with acroparesthesia, sensory ataxia, memory loss, and impotence. On examination, there are upper motor neuron signs in all four extremities. He also has anemia and a sore tongue. Eventually, a nutrient deficiency is diagnosed. This nutrient, critical for normal neurological function, must be absorbed by binding to which of the following? a. A cyanide atom and form cyanocobalamin b. An intrinsic factor c. The parietal cells of the stomach d. The ileal mucosa e. The jejunal mucosa

The answer is b. (Fauci, p 2540.) Intrinsic factor is a glycoprotein that is secreted by the gastric parietal cells. In most people with vitamin B12 deficiency, the problem is inadequate production of intrinsic factor rather than inadequate vitamin B12 in the diet. Persons with pernicious anemia usually have an atrophic gastritis with inadequate intrinsic factor production as a consequence. This is presumed to be mediated by an autoimmune disorder.

A patient has a borderline low vitamin B12 level. Testing for what compound is a more sensitive test for B12 deficiency? a. Cysteine b. Methylmalonic acid c. Methionine d. Succinic acid e. Propionic acid

The answer is b. (Fauci, p 2540.) Vitamin B12 (cobalamin) is an essential cofactor in the conversion of L-methylmalonyl-CoA to succinyl-CoA and of homocysteine to methionine. Without sufficient vitamin B12 , the conversion of propionic acid to succinic acid is blocked, and the intermediate compound, methylmalonic acid, accumulates in the blood. It is readily excreted and may help in the diagnosis of cobalamin deficiency when it is found in excess in the urine. Serum homocysteine levels, but not cysteine levels, may also be elevated because the conversion of homocysteine to methionine is disrupted if vitamin B12 is not available to expedite the methylation of homocysteine. Without this conversion of homocysteine to methionine, folate metabolism is disturbed, which probably interferes with DNA synthesis in red blood cell precursor cells.

An 82-year-old man with a history of pulmonary tuberculosis in 1947 presents with left body weakness and neglect. MRI shows a right frontal lesion, which is subsequently biopsied. The pathology suggests that the patient has recurrent tuberculosis. This mass lesion most likely consisted of which of the following? a. Dysplastic central nervous system (CNS) tissue b. Caseating granulomas c. Heterotopias d. Gram-positive bacteria e. Mesial sclerosis

The answer is b. (Fauci, pp 1008-1009.) Rupture of a large caseating granuloma into the ventricles or the subarachnoid space may produce an abrupt and often lethal deterioration. If the mass becomes large enough before it ruptures, it may in all respects imitate a brain tumor. Such lesions may respond to antituberculous medications even when they are quite large, and the patient may be spared surgical intervention.

A 70-year-old man presents with right body (face and arm > leg) weakness, which he says has gradually developed over several months. Medical history includes hypertension, smoking, and the occasional use of prostitutes. RPR serological testing is positive. Which of the following consequences of this patient's likely diagnosis may present a picture easily confused with brain tumor? a. A reaction to penicillin treatment occurs. b. An intracranial gumma forms. c. Tabes dorsalis is the primary manifestation of the disease. d. Meningovascular disease develops. e. The patient is a newborn with congenital acquired disease.

The answer is b. (Fauci, pp 1040-1042.) A gumma is a largely or entirely avascular granuloma. It rarely develops intracranially, but when it does, it may grow to several centimeters across. The lesion starts as an inflammatory process but becomes fibrosed as it evolves. The term gumma has traditionally been reserved for granuloma-like lesions caused by spirochetal infection.

A 48-year-old left-handed man develops increased sensitivity to sound in his left ear. A brain MRI reveals a posterior fossa mass. This symptom may develop in one ear with damage to which of the following ipsilateral CNs? a. V b. VII c. VIII d. IX e. X

The answer is b. (Ropper, pp 1329-1333.) The facial nerve innervates the stapedius muscle of the middle ear. With paralysis of this muscle, undamped transmission of acoustic signals across the stapedius bone of the middle ear produces hyperacusis. Hyperacusis is an indication that the damage to the facial nerve is close to its origin from the brainstem, because the nerve to the stapedius muscle is one of the first branches of the facial nerve. The tensor tympani is controlled by the motor fibers in the fifth CN. With damage to this nerve, the tympanic membrane has some inappropriate slack, but the patient does not usually comment on increased sensitivity to sound in the affected ear. Damage to CN VIII may cause a loss of hearing, but not hyperacusis. CNs IX and X are not involved in hearing.

A 44-year-old man presents with left arm shaking. Two days ago, the patient noticed left arm paresthesias along the lateral aspect of his left arm and left fourth and fifth fingers while he was reading. He thinks he may have been leaning on his left arm at the time; the symptoms resolved after 30 seconds. This morning, he noted the same feelings, lasting a few seconds, but then his fourth and fifth fingers started shaking rhythmically, and the shaking then spread to all of his fingers, his hand, and then his arm up to his elbow. This episode lasted a total of 30 seconds. He denies any strange smells or tastes, visual changes, or weakness. Afterward, his fingers felt locked in position for a few seconds. Then he felt as if he did not have control of his hand and had difficulty donning his socks. He and his wife decided to drive to the emergency room, and in the car he had trouble putting his seat belt latch into its socket. Examination and routine labs are normal. Which of the following is the most appropriate next action? a. Discharge the patient to follow up in clinic in 2 weeks b. Obtain a brain MRI c. Obtain an EEG d. Obtain an orthopedic consult e. Order electromyography (EMG) and nerve conduction studies (NCS)

The answer is b. (Fauci, pp 2504-2506.) This history is typical of a simple partial seizure. A focal brain lesion must be ruled out. It would be wrong to discharge the patient to follow up in clinic in 2 weeks without at least a CT scan and preferably an MRI. Although he probably had a seizure, obtaining an EEG at this point will not be as helpful as an MRI. The diagnosis of seizure has already been made clinically, now the etiology should be investigated. This is unlikely to be a peripheral nerve problem, and therefore an orthopedic consult or EMG and NCS are not indicated.

A 25-year-old woman with a history of epilepsy presents to the emergency room with impaired attention and unsteadiness of gait. Her phenytoin level is 37 (normal therapeutic range 10-20). She has white blood cells (WBCs) in her urine and has a mildly elevated thyroid-stimulating hormone (TSH) level. Examination of the eyes would be most likely to show which of the following? a. Weakness of abduction of the left eye b. Lateral beating movements of the eyes c. Impaired convergence d. Papilledema e. Impaired upward gaze

The answer is b. (Ropper, p 331.) Most rhythmic to-and-fro movements of the eyes are called nystagmus. Nystagmus has a fast component in one direction and a slow component in the opposite direction. Nystagmus with a fast component to the right is called right-beating nystagmus. Phenytoin may evoke nystagmus at levels of 20-to-30 mg/dL. The eye movements typically appear as a laterally beating nystagmus on gaze to either side; this type of nystagmus is called gaze-evoked. If the patient has nystagmus on looking directly forward, he or she is said to have nystagmus in the position of primary gaze. Therapeutic levels for phenytoin are usually 10-to-20 mg/dL, and some patients develop asymptomatic nystagmus even within that range. Ataxia, dysarthria, impaired judgment, and lethargy may also occur at toxic levels of phenytoin. Many other drugs, such as alcohol, barbiturates, and other sedatives, also evoke nystagmus. Weakness of abduction of the left eye, or abducens palsy, is due either to injury to the sixth CN or to increased ICP. Impaired convergence can occur normally with age or may be a sign of injury to the midbrain. Papilledema is a sign of increased ICP. Impaired upward gaze may occur in many conditions, but would not be expected to occur due to a toxic phenytoin level.

A 9-year-old boy presents with bilateral CN VII deficits. Serum and CSF analysis suggests B burgdorferi is the etiology. Which of the following medications is the most appropriate treatment? a. Streptomycin b. Ceftriaxone c. Gentamicin d. Isoniazid e. Rifampin

The answer is b. (Ropper, p 699.) If there is meningeal involvement, high-dose penicillin or ceftriaxone must be given intravenously for 10-to-14 days. Tetracycline qid for 30 days should be used for patients who are allergic to the intravenous treatments.

A 10-year-old girl is referred to a physician because of rapidly deteriorating school performance. Over the course of a few weeks, the child has lost interest in her schoolwork, appeared apathetic at home, and had frequent temper tantrums with little provocation. A psychiatric evaluation reveals that, in addition to emotional lability, the child has substantial intellectual deficits that appear to be new. Within 1 month of this evaluation, the child has a generalized tonic-clonic seizure. A neurologist examining the child discovers chorioretinitis, ataxia, hyperactive reflexes, and bilateral Babinski signs. Her EEG exhibits periodic bursts of high-voltage slow waves followed by recurrent low-voltage stretches (burst suppression pattern). The CSF is remarkable for an increase in the γglobulin fraction. The child becomes increasingly lethargic and obtunded over the ensuing 2 months. She remains in a coma for several months before dying.

The answer is b. (Ropper, p 734.) Subacute sclerosing panencephalitis (SSPE) usually develops in children and is rarely seen after the age of 18. Most affected children have had a bout of measles (rubeola) that occurred before they were 2 years old. SSPE may not appear for as long as 6- to-8 years after the episode of measles. Death usually occurs within 1-to-3 years after the onset of symptoms. SSPE produces a CSF pattern similar to that seen with multiple sclerosis, whose features include an increase in the γ-globulin fraction and the presence of oligoclonal bands. The measles virus appears to be directly responsible for this demyelinating disease, and the oligoclonal bands that appear in the CSF include a substantial proportion of measles-specific antibody. Eosinophilic inclusions are typically present in the cytoplasm and nuclei of neurons and glial cells.

CT scanning of a patient's head within 2 hours of a newly acquired epidural hematoma should reveal which of the following? a. A normal brain b. A lens-shaped density over the frontal lobe c. Increased cerebrospinal fluid (CSF) density with a fluid-fluid level d. Multifocal attenuation of cortical tissue e. Bilateral sickle-shaped densities over the hemispheres

The answer is b. (Ropper, p 858.) The typical shape of an epidural hematoma is that of a biconvex mass that displaces normal brain tissue. Parts of the ventricular system may be dilated as obstructive hydrocephalus develops in parts of the system. Transfalcial herniation with displacement of frontal lobe tissue across the midline and under the falx cerebri is likely with an epidural hematoma on one side of the head. Although subdural hematomas are often bilateral, epidural hematomas are invariably unilateral.

A 53-year-old left-handed man presents with asterixis, esophageal varices, splenomegaly, and abdominal ascites. He is likely to exhibit altered consciousness on the basis of which of the following? a. Renal tubular acidosis b. Impaired hepatic detoxification of portal blood c. Splenomegaly-induced anemia d. Copper intoxication e. Vitamin B12 deficiency

The answer is b. (Ropper, pp 1090-1093.) The clinical picture presented suggests hepatic failure. Copper poisoning may lead to hepatic failure, but the altered consciousness would be a consequence of the liver disease rather than the heavy metal poison itself. Similarly, vitamin B12 deficiency may lead to dementia, but it would not produce the signs of hepatic insufficiency exhibited by this patient. Encephalopathy that develops with chronic hepatic disease and portal hypertension is often called portal-systemic encephalopathy because of the importance of toxin-laden blood bypassing the liver as portal hypertension develops. Precisely what toxins produce the encephalopathy is still debatable, but ammonia is probably the most important one. This type of encephalopathy will develop if flow through the liver is obstructed and the liver is otherwise normal. This is distinct from the terminal coma that may develop with acute hepatic necrosis.

A 61-year-old man develops progressive cramping of his legs and a pins-and-needles sensation in his feet over the course of 1 year. He consults a physician when he notices paresthesias in his hands and unsteadiness of his gait. His family reports that he has had some urinary incontinence, but was too embarrassed to report it. On examination, he has a spastic paraparesis with severe disturbance of position and vibration sense in his legs. Despite obvious spasticity in the legs, the deep tendon reflexes are absent at the knees and ankles. Peripheral blood smear reveals hypersegmented polymorphonuclear leukocytes.

The answer is b. (Ropper, pp 1117-1121 .) The slow evolution of gait difficulty, bladder dysfunction, paresthesias, hyporeflexia, impaired position and vibration sense, and anemia suggests combined systems disease, the neurological equivalent of pernicious anemia. Persons with this disease may have a diet rich in vitamin B12 , but they will develop deficiency if they lack intrinsic factor in the stomach. Patients usually acquire a megaloblastic anemia associated with the spastic paraparesis. Finding hyper-segmented polymorphonuclear cells on the peripheral blood smear helps establish the diagnosis.

The pathologic specimen depicted here shows the only intracranial lesion found in this patient. This patient would be expected to exhibit which of the following symptoms? a. Seizures b. Gait ataxia c. Hemiparesis d. Visual loss e. Hallucinations

The answer is b. (Ropper, pp 113-114 .) This specimen is a transverse section through the brainstem and cerebellum. There is a large area of discoloration and disturbed anatomy in the left cerebellar hemisphere that is producing little mass effect. Because this is the only lesion postulated for this patient, there is no reason to suspect seizure activity, because that phenomenon would be unlikely in the absence of a cerebrocortical (or at least cerebral) lesion. The other findings listed would similarly not be expected in a patient with cerebellar damage.

An 18-year-old boy is brought into the emergency room after diving into a shallow pool. He is awake and alert, has intact cranial nerves (CNs), and is able to move his shoulders, but he cannot move his arms or legs. He is flaccid and has a sensory level at C5. Appropriate management includes which of the following? a. Naloxone hydrochloride b. Intravenous methylprednisolone c. Oral dexamethasone d. Intubation and preparation for immediate surgery e. Hyperbaric chamber therapy

The answer is b. (Ropper, pp 1187-1188.) High-dose intravenous methylprednisolone (Medrol) (30 mg/kg intravenous bolus followed by 5.4 mg/[kg-h] for 23 hours) has been shown to have a statistically significant, if clinically modest, benefit on the outcome after spinal cord injury when given within 8 hours of the injury. Naloxone hydrochloride and other agents, such as GM1 ganglioside, have not been shown to be of benefit. The role of surgical decompression, removal of hemorrhage, and correction of bone displacement is controversial. Hyperbaric chamber therapy may be useful for treatment of decompression illness (the bends).

A 19-year-old man notices discomfort in his ankles within a few days of recovering from an upper respiratory infection. Over the next 7 days, he develops progressive weakness in both of his legs and subsequently in his arms. He has no loss of sensation in his limbs, despite the progressive loss of strength. He does not lose bladder or bowel control, but on the tenth day of his weakness he develops problems with breathing and requires ventilatory assistance.

The answer is b. (Ropper, pp 1261-1270.) This young man with ascending paralysis with preserved sensation and sphincter control has Guillain-Barré syndrome. His CSF is largely normal except for its markedly high protein. The CSF is xanthochromic (ie, yellow) because of the high protein content of the fluid. Despite the pattern of weakness, which suggests an ascending myelitis, his CSF reveals a normal cell count. The CSF protein with Guillain-Barré syndrome may exceed 1 g, becoming so viscous that normal CSF flow patterns are disturbed.

A 35-year-old woman works as a keyboard operator and must type for 6 hours per day. Over the course of a few months she has developed pain in her wrists (right worse than left), as well as some paresthesias into the lateral palmar aspect of her hands. There is no atrophy. Conservative treatment for her condition consists of which of the following? a. Exploratory surgery b. Wrist splints c. Hydrocodone d. Shoulder sling e. Back brace

The answer is b. (Ropper, pp 1314-1315.) Pressure on the volar aspect of the wrist may produce recurrent injuries to the carpal tunnel through which the median nerve runs. The injury characteristically produces pain and paresthesias in the hand over the distribution of the sensory component of the median nerve. This sensory distribution extends over the palmar surface of the thumb and first four digits, with the fourth digit supplied on one side by the median nerve and on the other side by the ulnar nerve. Median nerve injuries are consequently said to split the fourth digit on sensory examinations. With carpal tunnel compression of the median nerve, the sensory disturbance may be incapacitating. Subsequently, weakness and atrophy may develop in the muscles that are innervated by the median nerve. The abductor pollicis brevis may be severely involved late in the progression of the disorder. Treatment options for carpal tunnel syndrome include avoiding aggravating factors, splints, and surgical procedures. The role of surgery is not clearly defined, but many experts agree that progressive cases or those with motor involvement should often consider surgical nerve release.

A 35-year-old man has severe throbbing pain waking him from sleep at night and persisting into the day. This pain is usually centered about his left eye and appears on a nearly daily basis for several weeks or months each year. It occurs most prominently at night within a few hours of falling asleep and is associated with a striking personality change in which the man becomes combative and agitated. He never vomits or develops focal weakness.

The answer is b. (Ropper, pp 174-176.) The term cluster headache refers to the tendency of these headaches to cluster in time. They may be distinctly seasonal, but the triggering event is unknown. The pain of cluster headache is usually described as originating in the eye and spreading over the temporal area as the headache evolves. In contrast to migraine, men are more often affected than women, and extreme irritability may accompany the headache. The pain usually abates in less than 1 hour. Affected persons routinely have autonomic phenomena associated with the headache that include unilateral nasal congestion, tearing from one eye, conjunctival injection, and pupillary constriction. The autonomic phenomena are on the same side of the face as the pain. These phenomena are similar to those elicited by the local action of histamine and gave rise to the now largely abandoned term Horton histamine headaches.

A 35-year-old woman has noticed that over the past 3-to-5 months she has had some difficulties with balance, particularly when she closes her eyes. On examination, she has decreased hearing in her left ear and also left body dysdiadochokinesia. Her physician orders a head CT, shown below. Given this CT scan, which was obtained without contrast enhancement, the physician must assume that the posterior fossa mass at the arrow is which of the following? a. Normal b. Calcified c. Highly vascular d. Granulomatous e. Highly cystic

The answer is b. (Ropper, pp 19, 25.) Calcified masses appear hyper-dense without contrast enhancement, whereas highly vascular lesions may appear dense on CT scanning after the patient has received intravenous contrast material. Tumors, granulomas, and other intracranial lesions enhance because of a breakdown in the blood-brain barrier. More cystic lesions may exhibit enhancement limited to the periphery of the cyst.

A 72-year-old woman with coronary artery disease and a history of cardiac artery stenting began developing episodes of transient visual loss several months ago. Each episode is restricted to her left eye and comes on suddenly. She says, "All of a sudden half of my vision is black." Each time the vision returns within 30 minutes. There have been approximately 10 of these events within the past 3 months. These symptoms are most likely related to which of the following? a. Retinal vein thrombosis b. Central retinal artery ischemia c. Posterior cerebral artery ischemia d. Middle cerebral artery ischemia e. Posterior ciliary artery ischemia

The answer is b. (Ropper, pp 225, 233.) The presumed mechanism of transient monocular blindness in carotid artery disease is embolism to the central retinal artery or one of its branches. Although classic teaching has emphasized the role that cholesterol emboli play in causing this blindness, it has been noted that cholesterol emboli (Hollenhorst plaques) may be seen on funduscopic examination even of asymptomatic individuals. Retinal vein thrombosis may produce a rapidly progressive loss of vision, with hemorrhages in the retina, but would not be associated with the transient attacks of amaurosis fugax. Although both posterior and middle cerebral artery ischemia can cause visual loss, they would not be expected to cause the monocular blindness of amaurosis fugax. Posterior ciliary artery ischemia can cause ischemic optic neuropathy, but this is usually acute, painless, and not associated with preceding transient monocular blindness or TIAs.

A 21-year-old right-handed woman works at an airport as a luggage handler. She is usually on the tarmac working in an environment in which loud noises are routine. Ear protection must be worn to protect against loss of hearing and the development of which of the following? a. Vertigo b. Tinnitus c. Ataxia d. Diplopia e. Oscillopsia

The answer is b. (Ropper, pp 282-283.) Acoustic trauma may produce severe tinnitus in persons who have relatively little hearing loss. Although the initial injury with acoustic trauma is sustained by the cochlear sensory cells, tinnitus may persist even after the acoustic nerve is cut. Tinnitus may take any one of several forms, ranging from a hissing sound to a high-pitched screaming noise.

A 42-year-old man sustained multiple injuries in an automobile accident. After orthopedic surgery, he is difficult to arouse. Assuming that his brainstem function is intact, when he is lying supine with his head slightly elevated (30°) and one external auditory meatus is irrigated with warm water, which of the following would be expected? a. Tonic deviation of the eyes toward the ear that is stimulated b. Nystagmus in both eyes toward the ear that is stimulated c. Tonic deviation of the ipsilateral eye toward the ear that is stimulated d. Nystagmus in both eyes away from the ear that is stimulated e. Tonic deviation of both eyes away from the ear that is stimulated

The answer is b. (Ropper, pp 291-292.) Caloric stimulation of the ear drives the endolymphatic fluid in the inner ear up or down, depending on whether warm or cold water is used. By tilting the head of a supine patient up 30° from the horizontal, the semicircular canal responsible for detecting horizontal head movements is placed in a vertical plane, and caloric stimulation drives the endolymph in that canal more effectively than it will the endolymph in the other semicircular canals. The vestibular organ exposed to warm water sends impulses to the brainstem that indicate that the head is moving to the side that is being warmed. The eyes deviate to the opposite side to maintain fixation on their targets, but the eye movement actually breaks fixation. A reflex nystagmus toward the ear that is being stimulated develops as the brain tries to establish refixation while the vestibular signals repeatedly prompt deviation of the eyes contralateral to the warm stimulus. The mnemonic COWS (Cold Opposite, Warm Same) may be useful for the direction of nystagmus.

A 30-year-old, normally developed, and generally healthy man has had new intermittent headaches for 1 year. Over the past several months they have been becoming more frequent and were twice accompanied by syncope. The patient has an MRI performed, and a colloid cyst of the third ventricle is identified. Which of the following is the most common complication of this lesion? a. Bitemporal hemianopsia b. Hydrocephalus c. Hemiparesis d. Optic atrophy e. Oscillopsia

The answer is b. (Ropper, pp 643-644.) Colloid cysts may produce transient or persistent obstruction of the flow of cerebrospinal fluid (CSF). Because this is an especially deep-seated lesion, it may be more practical to simply shunt the fluid from the lateral ventricles rather than attempt to excise the cyst. These cysts are usually lined with epithelial cells and may arise from a variety of sources, including low-grade neoplasms that involute early in their evolution.

A 19-year-old right-handed man who carries the diagnosis of epilepsy is seen in the urgent-care clinic. He had been healthy until about age 12, when he began to have episodes of eye fluttering lasting several seconds. Sometimes he would lose track of his thoughts in the middle of a sentence. There was one fall off of a bicycle that may have been related to one of these events. He has been treated with valproic acid. At one point he was off all medications, but the seizures returned. He is now at the end of his first semester of college and came in today because he had a witnessed generalized tonic-clonic seizure this morning. He had had only about 2 hours of sleep the night before because he was studying for a final examination. Which of the following is the most appropriate thing to tell this patient? a. "I know that you faked this seizure to avoid taking a test." b. "Lack of sleep may have contributed to triggering this seizure." c. "You can expect to have tonic-clonic seizures on a regular basis from now on." d. "Your seizures are getting worse and there is nothing we can do about it." e. "You should take the next semester off to recover and get extensive testing."

The answer is b. (Ropper, pp 304-335.) Lack of sleep is a common seizure trigger. There is no reason to believe that the patient faked the seizure. It is impossible to predict his future seizure course based on this one event; having one seizure does not necessarily mean that his seizures are getting worse, and even if they are, many treatments are available. There is no reason for the patient to take a prolonged leave of absence from school because of one seizure. This may even have detrimental psychological consequences. Other common seizure triggers include missing antiseizure medication doses, drugs (prescription or illicit), and psychological stress.

A 70-year-old woman with end-stage renal disease tends to develop restless legs syndrome as she becomes uremic. This may be controlled with which of the following drugs? a. Haloperidol b. L-Dopa c. Caffeine d. Nifedipine e. Rifampin

The answer is b. (Ropper, pp 382-383.) The restless legs syndrome is characterized by a feeling of discomfort in the legs that is relieved by movement. The sensation is felt deep within the limb, and is variably described as a pulling, stretching, or cramping. Restless legs syndrome occurs primarily at night, shortly after the patient lies down. It differs from akathisia, which is a restlessness that occurs during the daytime. It may be associated with peripheral neuropathy and anemia and is seen in patients with chronic renal disease, diabetes mellitus, and many other medical conditions. Exercise before going to bed may alleviate much of the discomfort. Agents that may be effective in alleviating symptoms include clonazepam, gabapentin, L-dopa, dopamine agonists (eg, pramipexole or ropinirole), and opiates. Neuroleptics, calcium channel blockers, and caffeine may worsen symptoms.

A 45-year-old woman with chronic atrial fibrillation discontinues warfarin treatment and abruptly develops problems with language comprehension. She is able to produce some intelligible phrases and produces sound quite fluently; however, she is unable to follow simple instructions or to repeat simple phrases. On attempting to write, she becomes very frustrated and agitated. Emergency MRI reveals a lesion of the left temporal lobe that extends into the superior temporal gyrus.

The answer is b. (Ropper, pp 466-470.) Presumably, an embolus from this woman's heart traveled to a branch of the middle cerebral artery that supplied her dominant hemisphere. The left hemisphere is usually the speech-dominant hemisphere. Wernicke aphasia is the most common of the so-called fluent aphasias: The affected person produces a string of sounds that may sound like a real language, but the sounds are generally meaningless. The patient seems to be unaware that his or her speech is incomprehensible. Comprehension and repetition are impaired. Typically, efforts at speaking produce only a meaningless string of phonemes that retain the rhythm and intonation of normal speech.

An obese 37-year-old woman has had a daily headache, worse in the morning, for 1 year. She has episodes of transient visual obscurations affecting each eye and also hears a pulsatile tinnitus. Examination is notable for bilateral papilledema. MRI is normal.

The answer is b. (Ropper, pp 601-605.) Pseudotumor cerebri, or idiopathic intracranial hypertension, is a condition of unknown cause that results in increased intracranial pressure, predominantly affecting obese women in their childbearing years. Symptoms include headaches, transient visual obscurations, progressive visual loss, pulsatile tinnitus, diplopia, and shoulder and arm pain. Neurological examination shows papilledema or optic atrophy if the syndrome has been long standing, and occasionally sixth nerve palsies may be present. Neuroimaging must be performed to exclude mass lesion or venous sinus obstruction, which can also lead to a similar syndrome of intracranial hypertension. Spinal fluid examination should be normal except for an elevated opening pressure. Additional causes of intracranial hypertension include systemic lupus erythematosus, renal disease, hypoparathyroidism, radical neck dissection, vitamin A intoxication, and steroid withdrawal. Treatment options include lumbar puncture, diuretics, ventriculoperitoneal shunting, and optic nerve sheath fenestration.

A previously healthy 31-year-old man collapses in the kitchen of his home while sitting at the table talking. His wife witnessed a convulsion that lasted about 2 minutes. He seems to recover fully within an hour. The history taken in the emergency room reveals that he has been having new headaches in the early morning hours over the past few weeks. A brain magnetic resonance imaging (MRI) indicates that there is an enhancing right frontal lesion that is most likely a primary brain neoplasm. Which of the following is the most common type of primary brain tumor? a. Meningioma b. Astrocytoma c. Lymphosarcoma d. Oligodendroglioma e. Medulloblastoma

The answer is b. (Ropper, pp 622-628.) The most common primary brain tumors are malignant astrocytomas. These are classified as grade 3 or 4. Grade 4 astrocytoma is more commonly called glioblastoma multiforme. It is malignant in the very conventional sense that it invades adjacent tissue. This type of glial tumor is usually seen in adults; men are more susceptible than women.

A 60-year-old woman presents to her internist with 2 months of new headaches and some difficulty walking. Further evaluation reveals multiple brain masses. Which of the following is the most common source of meta-static tumors to the brain in patients without a known primary tumor? a. Breast b. Lung c. Kidney d. Skin e. Uterus

The answer is b. (Ropper, pp 631-634.) The breast, lung, kidney, skin, and uterus are all common sources of metastases to the brain. The incidences of metastases from the lung account for two-thirds of cases of brain metastasis presenting without a known primary. Skin lesions metastasizing to the brain include malignant melanomas.

A 91-year-old woman has 3 days of gradually worsening fever and headache. She then develops blurry vision and a stiff neck. Her granddaughter becomes concerned and brings her to the emergency room. MRI with contrast has an enhancement pattern suggesting rhombencephalitis. CSF shows a mild pleocytosis with no organisms. All blood and CSF cultures are negative. Which of the following medications is the best treatment for the organism likely responsible for the patient's condition? a. Penicillin G b. Ampicillin plus gentamicin c. Tetracycline d. Ceftriaxone e. Rifampin

The answer is b. (Ropper, pp 678-679.) The presentation is highly suggestive of Listeria monocytogenes meningitis. This infection commonly develops in renal transplant recipients, patients with chronic renal disease, immunosuppressed persons, and occasionally in otherwise unimpaired persons. It may also affect neonates. This type of meningitis is not usually seen in older children. It may on occasion lead to intracerebral abscess formation. Third-generation cephalosporins are inactive against Listeria, and ampicillin and gentamicin are recommended therapy. Neither ampicillin nor penicillin alone is bactericidal.

A 22-year-old male abuser of intravenous heroin has been having severe headaches during sexual intercourse. Within a few minutes of one headache, he develops right-sided weakness and becomes stuporous. His neurologic examination reveals neck stiffness as well as right arm and face weakness. An unenhanced emergency CT scan reveals a 3-to-4 cm lesion in the cortex of the left parietal lobe. The addition of contrast enhancement reveals two other smaller lesions in the right frontal lobe but does not alter the appearance of the lesion in the left parietal lobe. Which of the following diagnostic studies is most likely to establish the basis for this patient's neurologic deficits? a. Human immunodeficiency virus (HIV) antibody testing b. Cerebrospinal fluid (CSF) examination c. Electroencephalography (EEG) d. Nerve conduction studies (NCS) e. Cardiac catheterization

The answer is b. (Ropper, pp 684, 816.) This young man almost certainly has numerous problems associated with his intravenous drug abuse, but the cause of his current problems is most likely bleeding from a mycotic aneurysm. Aneurysms are especially likely to bleed during exertion, such as that associated with sexual intercourse or defecation. The fact that the lesion appeared largely the same on unenhanced and enhanced CT scans suggests that it is a hematoma. HIV antibody testing might reveal evidence of exposure to HIV, but, aside from establishing that the patient was at increased risk of opportunistic infections, that test would provide little insight into the cause of the acute neurologic syndrome. The CSF would be expected to be xanthochromic (yellow), with many (> 20/μL) red blood cells (RBCs), or grossly bloody, thereby providing evidence of a recent subarachnoid hemorrhage. EEG would very likely reveal an asymmetric pattern associated with the left hemispheric lesion, but this too would provide little insight into the cause of the problem. NCS would not clarify the basis for a lesion of the CNS, because they examine only structures of the peripheral nervous system. Cardiac catheterization might reveal valvular abnormalities, but these need not be associated with disease of the CNS.

Which of the following is the most common cause of brain abscess in patients with AIDS? a. Cryptococcus neoformans b. Toxoplasma gondii c. Tuberculosis d. Cytomegalovirus e. Herpes zoster

The answer is b. (Ropper, pp 702-703.) Fungal abscesses develop with unusual frequency in patients with AIDS, but T gondii, an obligate intracellular parasite, is considerably more common than fungi as the cause of abscess formation. The fungi that do produce abscesses in persons with AIDS are most often Cryptococcus, Candida, Mucor, and Aspergillus. Myco-bacteria and atypical mycobacteria are also common causes of abscess formation in some populations.

A 21-year-old college student was found walking around his dormitory naked. He is disoriented, inattentive, and shows poor comprehension. In the emergency room he is found to have a fever of 102°F (38.8°C). There are no apparent motor, sensory, or coordination abnormalities. The emergency room physician orders a brain MRI and then decides to perform a lumbar puncture. Neuroimaging of the brain before attempting a lumbar puncture is advisable in cases of acute encephalitis for which one of the following reasons? a. The diagnosis may be evident on the basis of MRI alone. b. Massive edema in the temporal lobe may make herniation imminent. c. The CT picture may determine whether a brain biopsy should be obtained. d. Shunting of the ventricles is usually indicated, and the imaging studies are needed to direct the placement of the shunt. e. It may establish which pathology is responsible.

The answer is b. (Ropper, pp 719-721.) Although there is some controversy regarding whether lumbar puncture can precipitate herniation with a herpes encephalitis, most authorities believe it is best to assess the risk of herniation before doing a lumbar puncture. CSF examination is vital in establishing the diagnosis. A variety of infections may mimic herpes in both course and anatomic distribution. The CSF cultures and analysis of CSF constituents help to establish the probable cause of the encephalitis and to direct therapy

You are working in the emergency room when a 30-year-old man presents with a headache that started yesterday. As he was shoveling snow, he felt a sudden pain in the front of his head. The pain does not throb and has been relatively constant since. He says that now his neck also has become a little stiff. He carries a diagnosis of migraine headaches, but says that this is different than his usual headaches. He is afebrile and has a normal examination except for slight photophobia and mild discomfort with neck flexion. Which of the following is the most appropriate next step in management? a. Obtain a brain MRI b. Obtain a brain CT c. Obtain a cerebral angiogram d. Obtain an electroencephalogram (EEG) e. Obtain a psychiatric consult

The answer is b. (Ropper, pp 747-748, 808-816.) A sudden-onset new type of headache is a typical presentation of subarachnoid hemorrhage. As blood begins to irritate meninges, a stiff neck may occur. A CT scan will detect blood locally or diffusely in the subarachnoid spaces or within the brain or ventricular system in more than 90% of cases. It is more sensitive than an MRI in most cases and can be obtained more quickly. A cerebral angiogram could diagnose the etiology of a subarachnoid hemorrhage, such as an aneurysm. However, it is an invasive test and should not be done without first attempting to confirm the diagnosis with less risky tests. If you suspect seizures, an EEG might be useful. Because this patient did not have any loss of consciousness, personality changes, or hallucinations, there is no indication for a psychiatric consult.

A 56-year-old right-handed woman presents to the emergency room with a sudden-onset, severe, left-sided headache. The pain began when she stood up from her couch while watching TV. A head CT is normal. Which of the following is the most appropriate next step in management of this patient? a. Begin intravenous heparin b. Perform a lumbar puncture c. Obtain a brain MRI d. Obtain a cerebral angiogram e. Give the patient a prescription for zolmitriptan and send her home

The answer is b. (Ropper, pp 747-748, 808-816.) The history of a sudden-onset severe headache is very concerning for subarachnoid hemorrhage. Additionally, this patient is in the peak age range for a ruptured cerebral aneurysm. In suspected subarachnoid hemorrhage, CT will detect blood locally or diffusely in more than 90% of cases. However, if no blood is seen, the physician should proceed to a lumbar puncture. Elevated CSF red blood cells, xanthochromia, and increased opening pressure all may be caused by subarachnoid hemorrhage. A cerebral angiogram could diagnose the etiology of a subarachnoid hemorrhage, such as an aneurysm; however, it is a more invasive test and should not be done without first attempting to confirm the diagnosis with less risky tests. An MRI is unlikely to give new useful information in this case. Zolmitriptan is a treatment for migraines. This patient's history is not typical for a migraine. Heparin should be given in some cases of embolic stroke; in this case the anticoagulant may lead to uncontrollable bleeding from the aneurysm and likely death.

Occlusion of which of the following arteries typically produces Wallenberg (lateral medullary) syndrome? a. Basilar artery b. Vertebral artery c. Superior cerebellar artery d. Anterior inferior cerebellar artery (AICA) e. Anterior spinal artery

The answer is b. (Ropper, pp 748-772.) Most cases of lateral medullary infarction are caused by occlusion of the vertebral artery. Several small branches of the distal vertebral artery supply the lateral medulla. In some cases, occlusion of the PICA causes this syndrome. The PICA is the last large branch of the vertebral artery, and, when it is occluded, there may also be infarction of the inferior cerebellum accompanying that of the medulla. The basilar artery supplies the entire posterior brain circulation, and an occlusion would likely cause a much more widespread stroke. The superior cerebellar artery supplies the superior portions of the cerebellum, whereas AICA supplies portions of the cerebellum and lower cranial nerves

Following cardiac catheterization, a 60-year-old right-handed man acutely develops a loss of sensation involving the entire left side of his body (face, arm, and leg). Which of the following structures has most likely been damaged? a. Internal capsule b. Thalamus c. Hippocampus d. Globus pallidus e. Pons

The answer is b. (Ropper, pp 748-772.) Pure sensory strokes are most likely in the same persons who are susceptible to pure motor strokes and other lacunae. With hypertensive injury to the posteroventral nucleus of the lateral thalamus, the affected person will report contralateral numbness and tingling. During recovery from this type of stroke, paradoxical pain may develop in the area of sensory impairment. This paradoxical pain associated with decreased pain sensitivity is referred to as the thalamic pain syndrome. This style of stroke may also be secondary to a small embolus.

A 43-year-old man presents with a left CN III deficit and headache. The ocular symptoms began with papillary dilation and then progressed to oculomotor impairment. Which of the following is the most likely site of the lesion responsible for this presentation? a. Anterior communicating artery b. Posterior communicating artery c. Anterior cerebral artery d. Brainstem white matter e. Posterior cerebral artery

The answer is b. (Ropper, pp 808-816.) An aneurysm on the posterior communicating artery is especially likely to compress the oculomotor (third) nerve. Because the pupilloconstrictor fibers lie superficially on this nerve, problems with pupillary activity are routinely early phenomena. An ischemic injury to the third CN, such as that seen with diabetes mellitus, will usually spare these superficial fibers, presumably because they have a vascular supply that is fairly distinct from that of the rest of the third nerve. The pupillary response to both direct and consensual stimulation will be impaired with compression of these parasympathetic nerve fibers. This means that the pupil in the right eye will not constrict in response to light shining into either the right or the left eye. The normal pupil on the left will constrict with light shining into either the left or the right eye because the sensory input from the right eye is unimpaired. As the aneurysm enlarges, it impinges on the third-nerve fibers that supply the medial rectus muscle, weakness of which will be responsible for double vision. Lesions of the superior cerebellar artery and posterior cerebral artery can also compress the third nerve, which exits between them. It is therefore important that a complete angiogram, evaluating all four vessels, be performed in the evaluation for subarachnoid hemorrhage and third-nerve palsy.

A 35-year-old pregnant woman at term is admitted to the hospital for delivery. She has headaches and visual blurring. Her blood pressure is 180/100. On examination, she is edematous. Reflexes are increased. Protein is found in the urine. She then develops a generalized tonic-clonic convulsion.

The answer is b. (Ropper, pp 823-824.) Recent studies have established that magnesium sulfate (MgSO4 ) is the optimal treatment both to prevent seizures in women with hypertension at the time of admission for delivery (preeclampsia) and to treat seizures in established eclampsia. The dose is 4- to-5 g intravenously, followed by a 1 g/h intravenous infusion. Magnesium sulfate was shown to result in a reduction in recurrent seizures and in maternal morbidity and mortality compared with both diazepam and phenytoin. In addition, the fetus should be delivered as quickly as possible, using caesarean section if necessary. Eclampsia is an example of the posterior leukoencephalopathy syndrome.

A 21-year-old, generally healthy college student presents with loss of sensation in the right arm that has been progressive over a few days. She says that she had flu for about a week and noticed the symptoms started afterward. It began in her hand, and then very slowly progressed over 36 hours to involve the entire right upper extremity. Her physician ordered blood tests and an MRI. A T2 image is pictured below. Which of the following is the most likely process? a. Ischemic b. Demyelinating c. Neoplastic d. Hemorrhagic e. Psychogenic

The answer is b. (Ropper, pp 896-899.) The history and MRI are consistent with a demyelinating process, such as acute disseminated encephalomyelitis (ADEM) or multiple sclerosis. The MRI shown is a T2-weighted image. Several areas of abnormal high signal are apparent. MRI depends primarily on the water content of tissues, but it can be customized to look at more specific properties of tissues. A variety of methods are available for affecting the information produced by and analyzing the information generated by MRI. The T2-weighted image is not specific for demyelination, but it is useful in following changes in plaques of demyelination, an application that has been used in studies of agents useful in the management of multiple sclerosis. The MRI appearance and history are not highly consistent with ischemic, neoplastic, or hemorrhagic processes. Ischemic or hemorrhagic processes will typically present more acutely, whereas a neoplasm would either present very acutely or much more subacutely.

A 42-year-old attorney presents with a tremor in her hands that is most obvious when she is awake and trying to perform an action. She had first noticed it several years ago, but is concerned that it may be very slowly worsening. A tremor of this type is most likely caused by disease in which of the following structures? a. Thalamus b. Cerebellum c. Substantia nigra d. Spinal cord e. Internal capsule

The answer is b. (Ropper, pp 93-95.) Intention or kinetic tremors are most characteristic of damage to the cerebellum. Kinetic tremors of the hand or arms are most common with disease of the cerebellar hemispheres, but they may also develop with damage to the spinocerebellar tracts of the spinal cord. Damage to the substantia nigra, such as that occurring in Parkinson disease, produces a resting tremor that abates when the patient moves the involved limb intentionally. Damage to the thalamus is more likely to produce a sensory disturbance. Tremors may develop with spinal cord damage, but they do not follow a typical pattern and do not suggest a spinal cord origin. Internal capsule lesions typically cause weakness.

As you provide the parents of the patient (see question 173) with some information, it prompts many more questions. The overall incidence of primary brain tumors in children is approximately 1- to-5 per 100,000 per year. Which of the following account for the majority? a. Meningiomas and neurofibromas b. Astrocytomas and medulloblastomas c. Melanomas and choriocarcinomas d. Gliomas and adenomas e. Colloid cysts of the third ventricle

The answer is b. (Swaiman, p 1687.) Meningiomas may occur in childhood, but are more likely to appear and become symptomatic during adult life. Neurofibromas are not primary brain tumors, although schwannoma of the eighth cranial nerve (CN) is sometimes incorrectly referred to as an acoustic neurofibroma rather than an acoustic schwannoma. Colloid cysts of the third ventricle are not necessarily neoplastic, although most are assumed to have started as neoplasms rather than as developmental anomalies. Glioma is a broad category that includes the astrocytoma. Adenomas, such as pituitary adenomas, do develop in children, but much less commonly than either astrocytomas or medulloblastomas. CNS tumors account for a large proportion of the tumors seen in childhood. In fact, they are second in frequency only to childhood leukemias and account for 15% to 20% of childhood tumors.

A 4-year-old boy has the onset of episodes of loss of body tone, with associated falls, as well as generalized tonic-clonic seizures. His cognitive function has been deteriorating. EEG shows 1-to-2 Hz spike-and-wave discharges. Which of the following is the most likely diagnosis? a. Landau-Kleffner syndrome b. Lennox-Gastaut syndrome c. Juvenile myoclonic epilepsy d. Mitochondrial encephalomyopathy e. Febrile seizures

The answer is b. (Swaiman, pp 1031-1033.) Lennox-Gastaut syndrome is characterized by mental dysfunction, multiple seizure types, and 1-to 2-Hz generalized spike-wave discharges on EEG. It is often difficult to control the seizures that develop in children with this syndrome. Many affected children have a history of infantile spasms (West syndrome). Infants and children with infantile spasms exhibit paroxysmal flexions of the body, waist, or neck and usually have a profoundly disorganized EEG pattern called hypsarrhythmia. Landau-Kleffner syndrome is associated with loss of language function and an abnormal EEG during sleep. Juvenile myoclonic epilepsy is a relatively benign epilepsy syndrome with onset in late adolescence or early adulthood. Febrile seizures are generalized tonic-clonic seizures associated with fever in a child who is otherwise healthy.

A 50-year-old man had a brainstem stroke following a vertebral artery dissection secondary to an acute sports-related injury. This patient might be expected to develop dysphagia secondary to involvement of which of the following structures? a. Nucleus solitarius b. Nucleus and descending tract of cranial nerve (CN) V c. Nucleus ambiguus d. Lateral spinothalamic tract e. Inferior cerebellar peduncle

The answer is c. (Kandel, pp 884-885.) The nucleus ambiguus, located in the ventrolateral medulla, contains the motor neurons that contribute to the ninth (glossopharyngeal) and tenth (vagus) cranial nerves. The motor neurons of the nucleus ambiguus innervate the striated muscles of the larynx and pharynx as well as provide the preganglionic parasympathetic supply to thoracic organs, including the esophagus, heart, and lungs. Injury to this nucleus and its pathways causes hoarseness and dysphagia. The nucleus solitarius combines afferents from CNs VII, IX, and X responsible for visceral sensation. Its projections are primarily to parasym-pathetic and sympathetic preganglionic neurons in the medulla and spinal cord.

A 42-year-old man notices that his right pupil is smaller than the left. His wife has also commented that the right eye is "droopy looking." The only remarkable recent history is that he was tackled a little hard while playing football the day before. An axial T1-weighted magnetic resonance image (MRI) is shown below. Which of the following is present? a. Increased T2 signal in a periventricular distribution b. Contrast enhancement along the tentorial margin c. Increased T1 signal in the wall of the right carotid artery d. Enlarged optic nerve in the orbit e. Thrombosed cavernous sinus aneurysm

The answer is c. (Osborn, pp 72-73.) The most likely cause of this patient's symptoms and signs is a carotid dissection. Dissections are caused by a tear in the intimal lining of the vessel with penetration of blood beneath the intimal surface, forming an intramural hematoma (blood clot within the wall of the vessel). Carotid artery dissections can occur spontaneously, post-traumatically, or in the setting of underlying disease process that weakens the endothelial intima. They may appear as an area of increased signal within the lining of the vessel on T1-weighted images, on which a blood clot appears bright. Typically, this takes a crescentic pattern, and the lumen of the vessel is displaced eccentrically. An increased T2 signal in a periventricular distribution is typical of the plaques of demyelinating disease seen in multiple sclerosis. Contrast enhancement along the tentorial margin might be seen in inflammatory diseases of the dura mater or meninges. An enlarged optic nerve might be seen in the setting of optic neuritis or infiltration of the nerve by a tumor or other process, and this would be expected to cause visual impairment. A cavernous sinus aneurysm that is sufficient to produce oculosympathetic palsy would be expected to cause other deficits in ocular motility, because the CNs subserving oculomotor function course through the cavernous sinus.

A young man fractures his humerus in an automobile accident. As the pain from the injury subsides, he notices weakness on attempted flexion at the elbow. He develops paresthesias over the radial and volar aspects of the forearm. During the accident, he probably injured which one of the following nerves? a. Suprascapular nerve b. Long thoracic nerve c. Musculocutaneous nerve d. Radial nerve e. Median nerve

The answer is c. (Ropper, p 1314.) The musculocutaneous nerve is often damaged with fractures of the humerus. This nerve supplies the biceps brachii, brachialis, and coracobrachialis muscles and carries sensory information from the lateral cutaneous nerve of the forearm. Flexion at the elbow with damage to this nerve is most impaired with the forearm supinated. The suprascapular and long thoracic are motor nerves. The radial nerve provides sensation to the dorsal-radial aspect of the forearm. The median nerve does not provide sensation to the forearm.

A 19-year-old man is involved in a street fight in which he is viciously attacked with a lead pipe. A particularly forceful blow hits his left elbow. Blunt trauma to the elbow may lead to the development of which of the following? a. Wristdrop b. Weakness of the abductor pollicis brevis c. Clawhand or benediction sign (impaired extension of digits 4 and 5) d. Ulnar deviation of the hand e. Poor pronation of the forearm

The answer is c. (Ropper, p 1315.) The ulnar nerve runs superficially at the elbow in the ulnar groove. It continues forward under the aponeurosis of the flexor carpi ulnaris in the cubital tunnel. Damage to the nerve at this site may produce weakness in the interosseous and ulnar lumbrical muscles of the hand. With lumbrical weakness, the extensor sheaths of the digits are not properly positioned, and a claw deformity with impaired extension of the ulnar two digits develops when the patient tries to straighten his or her fingers.

A 22-year-old woman reports a scotoma progressing across her left visual field over the course of 30 minutes, followed by left hemicranial throbbing pain, nausea, and photophobia. Her brother and mother have similar headaches. Which of the following is present in this patient's condition but not in common migraine? a. Photophobia b. Familial pattern c. Visual aura d. Hemicranial pain e. Nausea

The answer is c. (Ropper, p 166.) Classic migraine, but not common migraine, is preceded by an aura of neurological dysfunction. The aura is most often visual in nature, consisting of bright flashing lights, scintillating scotomas, or field cuts. Both kinds of migraine are most often characterized by a hemicranial throbbing headache associated with nausea, vomiting, photophobia, and phonophobia (aversion to sound). Familial patterns are not unusual with either classic or common migraine, although with classic migraine the probability that another family member will have a similar problem approaches 80%.

A 12-year-old boy has left body weakness. A brain magnetic resonance imaging (MRI) scan reveals a polycystic lesion. The parasitic brain lesion most likely to have a large cyst containing numerous daughter cysts is that associated with which of the following? a. Taenia solium b. Schistosoma haematobium c. Echinococcus granulosa d. Diphyllobothrium latum e. Schistosoma japonicum

The answer is c. (Ropper, p 706.) Echinococcosis is usually acquired by ingesting material contaminated with fecal matter from sheep or dogs. Children are more likely to develop cerebral lesions than adults, but people at any age may develop this encephalic hydatidosis, which entails the development of a major cyst with multiple compartments in which smaller cysts are evident. This hydatid cyst of the brain behaves like a tumor and may become massive enough to cause focal deficits.

A 27-year-old man develops recurrent episodes of involuntary movements. He abused intravenous drugs for several years and has had several admissions for recurrent infections, including subacute bacterial endocarditis. His involuntary movements are largely restricted to the right side of his body and are associated with hoarseness and difficulty swallowing. The patient has lost 40 lb over the past 4 months. Examination reveals diffuse lymphadenopathy and right-sided hypertonia. His CSF is normal except for a slight increase in protein content. CT reveals a large area of decreased density on the left side of the cerebrum. EEG reveals diffuse slowing over the left side of the head. Biopsy of this lesion reveals oligodendrocytes with abnormally large nuclei that contain darkly staining inclusions. There is extensive demyelination, and there are giant astrocytes in the lesion. Over the course of 1 month, the man exhibits increasing ataxia. Within 2 months, he shows evidence of mild dementia and seizures. Within 3 months of presentation, his dementia is profound, and he has bladder and bowel incontinence. Over the course of a few days, he becomes obtunded and dies.

The answer is c. (Ropper, p 711.) This patient probably had AIDS with PML as a complication of that disease. The inclusion bodies in the oligodendrocyte nuclei are JC virus. Primary infection with JC virus is universal and asymptomatic. Immunosuppression leads to reactivation of the virus. Diagnosis is typically made by MRI, which shows multiple focal well-defined white matter lesions that do not enhance or have mass effect. CSF PCR for JC virus is also available, obviating the need for brain biopsy in most cases. Treatment with cytarabine arabinoside has not been shown to be effective in clinical trials. Fewer than 10% of patients may experience spontaneous remission. PML may also develop with lymphomas, leukemias, or sarcoid, but the incidence of this disease in the US population has expanded greatly since the dissemination of HIV in the population.

A 35-year-old woman has progressive numbness of the right arm and difficulty seeing objects in the right visual field. She is known to be HIV positive, but has not consistently taken medications in the past. On examination, she appears healthy, but has a right homonymous hemianopsia and decreased sensory perception in her right upper extremity and face. Her CD4 count is 75 cells per μL, and her MRI is consistent with a demyelinating lesion of the left parietooccipital area. CSF PCR for JC virus is positive. Which of the following is the most appropriate treatment in this case? a. Amphotericin B b. Cranial radiation c. Highly active antiretroviral therapy (HAART) d. Intravenous acyclovir e. Intravenous ceftriaxone

The answer is c. (Ropper, p 712.) The patient has PML. It is caused by the JC virus, which is a double-stranded DNA virus. The prognosis is poor, but HAART has been known to be effective in improving survival. JC virus is ubiquitous and may be transmitted through respiratory secretions. Cranial radiation is used to treat malignancies. Amphotericin B is used to treat fungal infections. Intravenous acyclovir is not effective against JC virus, but is used to treat herpes simplex virus encephalitis. Intravenous ceftriaxone is used to treat bacterial meningitis.

A 45-year-old right-handed man who has been human immunodeficiency virus (HIV) positive for the past 3 years has noticed some sort of visual change over the past 1 to 2 months. It is difficult for him to describe, but it is some sort of distortion of part of his right visual field. There is a 4-cm rimenhancing lesion in the left occipital lobe that is revealed by MRI. Which of the following tumor types is common in the brain of patients with acquired immune deficiency syndrome (AIDS), but otherwise extremely rare? a. Lymphocytic leukemia b. Metastatic lymphoma c. Primary lymphoma d. Kaposi sarcoma e. Lymphosarcoma

The answer is c. (Ropper, p 729.) Kaposi sarcoma is unusually common in patients with AIDS, but it is rarely metastatic to the brain. Metastatic lymphomas producing meningeal lymphomatosis are not especially rare in the general population, but primary lymphomas (ie, lymphomas apparently arising in the CNS) were rare before the AIDS epidemic. The primary brain lymphoma usually presents as a solitary mass and can occur anywhere in the brain, but it does have a predilection for the periventricular structures.

An elderly patient suffers from a relatively mild head trauma but then subsequently develops a progressive dementia over the course of several weeks. He is most likely to have sustained which of the following? a. An acute subdural hematoma b. An acute epidural hematoma c. A chronic subdural hematoma d. An intracerebral hematoma e. An intracerebellar hematoma

The answer is c. (Ropper, p 835.) Chronic subdural hematoma is relatively common in the elderly and in patients receiving renal dialysis. The subdural fluid becomes isodense with the brain after several days or weeks and may be overlooked on CT scanning. MRI will identify the lesion, even if it is present bilaterally and produces no shift of brain structures from the midline.

A 16-year-old boy is struck on the side of the head by a bottle thrown by a friend involved in a prank. He appears dazed for about 30 seconds, but is lucid for several minutes before he abruptly becomes stuporous. His limbs on the side opposite the site of the blow are more flaccid than those on the same side as the injury. On arrival in the emergency room 25 minutes after the accident, he is unresponsive to painful stimuli. His pulse is 40 beats per minute, with an electrocardiography (ECG) revealing no arrhythmias. His blood pressure in both arms is 170/110 mm Hg. Although papilledema is not evident in his fundi, he has venous distention and absent pulsations of the retinal vasculature. Which of the following is the best explanation for this young man's evolving clinical signs? a. A seizure disorder b. A cardiac conduction defect c. Increased intracranial pressure d. Sick sinus syndrome e. Communicating hydrocephalus

The answer is c. (Ropper, p 858.) Something has abruptly caused increasing intracranial pressure in this young man after his head trauma. Consequently, he is at risk for herniation of the brain transfalcially (across the falx cerebri) or transtentorially (across the tentorium cerebelli). The head trauma produced an intracranial lesion, which is expanding very rapidly. The slowing of his pulse and increase in his blood pressure are due to the Cushing effect of a rapidly expanding intracranial mass. The history is typical for that of an epidural hematoma.

A 26-year-old man develops hemoptysis and dyspnea over the course of 3 months. His physician suspects tuberculosis and starts him on triple therapy with isoniazid (isonicotinic acid hydrazide), rifampin, and ethambutol. After 1 month of treatment, the patient's liver enzymes show slight elevations, but the treatment is continued. The hemoptysis stops by 2 months, but the patient complains of pins-and-needles sensations in his feet. Neurological examination reveals hypoactive deep tendon reflexes in the legs and slightly impaired position sense. Strength is good in all limbs.

The answer is c. (Ropper, p 909.) Any patient treated with isoniazid must receive supplemental pyridoxine. Isoniazid does not interfere with pyridoxine absorption, but it does interfere with its participation in metabolic pathways. Persistently low pyridoxine activity leads to the development of a peripheral neuropathy. This is most likely to be seen as an isolated deficiency in patients on antituberculous therapy.

A 15-year-old boy has multiple angiomatoses of the retina and cysts of the kidney and pancreas. Which of the following brain tumors is most likely to develop in this child? a. Glioblastoma multiforme b. Meningioma c. Hemangioblastoma d. Ependymoma e. Pinealoma

The answer is c. (Ropper, p 986.) With von Hippel-Lindau syndrome, the patient may exhibit tumors in multiple organs. In the brain, hemangioblastomas are the tumors most likely to arise, and these tumors are usually limited to the cerebellum or brainstem. Hemangioblastomas are often multiple and become symptomatic by bleeding into themselves. The initial episode of bleeding may prove lethal.

A 55-year-old right-handed man is admitted to the medical service for pneumonia. The patient normally drinks 4-to-8 beers per day. In anticipation of the seizures, cognitive deterioration, and autonomic instability that might occur during withdrawal, which of the following is the most appropriate measure to take? a. Consult a "detox center" to start planning the patient's discharge. b. Provide intravenous alcohol supplements to blunt the alcohol withdrawal. c. Provide intramuscular or oral chlordiazepoxide several times daily at a dose dictated by the patient's level of agitation. d. Start phenytoin as a single dose nightly. e. Delay pneumonia treatment until the risk of neurological problems abates.

The answer is c. (Ropper, pp 1135-1139 .) Chlordiazepoxide at relatively high doses of 25-to100 mg, four to six times daily, will usually block the more malignant features of both alcohol and benzodiazepine withdrawal. This drug is itself a benzodiazepine, but once the patient has passed through the withdrawal period for the drugs he or she has been taking, the chlordiazepoxide can be systematically and uneventfully reduced. There are no apparent advantages to using an antiepileptic such as phenytoin.

A 50-year-old homeless woman is admitted to the hospital with ulcerations on her feet. She complains of burning in her feet and lower legs, but does not localize the pain to where her skin is ulcerated. She is unable to stand with her eyes closed and her feet together. Her deep tendon reflexes are diffusely hypoactive. On systematic testing of her strength and sensation, decreased pain, position, and vibration sense are evident in her hands and feet. She also has weakness on dorsiflexion of the ankles and wrists.

The answer is c. (Ropper, pp 1258-1261, 1308-1309.) A nerve conduction study should confirm that this woman has a peripheral neuropathy—that is, a disturbance of sensory and motor nerve function in the limbs. Given her social condition, she is at high risk for a nutritional neuropathy. A glove-and-stocking pattern of sensory disturbance is usually seen with lesions that involve peripheral nerves, specifically the nerves extending out into the limbs. The meaning of glove-and-stocking is self-evident: Sensation is disturbed over the hands and the feet, with extension up the arms and the legs being quite variable. With severe neuropathy of the hands or the feet, ulcerations and pressure sores will develop over the skin that is innervated by the damaged nerves. The most severe sensory deficit affects the most terminal elements of the limbs. Metabolic or nutritional problems are the usual causes of a glove-and-stocking pattern of sensory disturbance. Diabetes mellitus, thiamine deficiency, and neurotoxin damage (eg, that caused by some insecticides) are the commonest causes of these sensory disturbances. Affected persons usually report the sensation of pins and needles in the hands and feet, but with some neuropathies, severe pain may develop along with the loss of sensory acuity.

A 15-year-old boy developed a left Bell palsy over the course of 1 week. He was treated with acyclovir and prednisone. Over the next 3 months he seemed to recover almost fully. However, he has noticed involuntary twitching at the left corner of the mouth each time he tries to blink the left eye. This is most likely caused by which of the following? a. A habit spasm b. Cerebellar damage producing impaired coordination c. Aberrant regeneration of the facial nerve d. Trigeminal neuralgia e. Focal seizures

The answer is c. (Ropper, pp 1330-1331.) After injury to the facial nerve, regenerating fibers may be misdirected. This is especially common with Bell palsy (idiopathic facial weakness). Aberrant regeneration is possible only if the nerve cell bodies survive the injury and produce axons that find their way to neuromuscular junctions. Fibers intended for the periorbital muscles end up at the perioral muscles, and signals for eye closure induce mouth retraction. With a habit spasm or idiopathic tic, similar movements may occur, but the movement disorder would not be linked to facial weakness. Trigeminal neuralgia is painful and rare in this age group. Focal seizures may present as involuntary mouth movements, but it would be unlikely that they would be triggered by attempting to blink the eye.

A 27-year-old healthy normal woman is having a routine EEG examination. The study begins with a 5-minute recording of her sleeping. Then she is awakened and given photic stimulation. Next she is alert and awake, lying with her eyes closed in a quiet room. At this point she will exhibit what frequency of EEG activity over the occipital and parietal areas bilaterally? a. 0-to-3 Hz b. 4-to-7 Hz c. 8-to-13 Hz d. 14-to-25 Hz e. 26-to-45 Hz

The answer is c. (Ropper, pp 27-33.) The relaxed adult exhibits α-wave activity at a frequency of 8-to-13 Hz over the posterior aspects of the head; α-activity disappears with eye opening and with concentration on mathematical activities. This brain wave activity should be equally well developed over both sides of the head. As the subject becomes drowsy, the α-activity becomes less obvious. The α-wave is the most prominent EEG finding in most normal individuals and is so named because it was the first EEG-recorded brain rhythm.

A 16-year-old woman has been having attacks of weakness, blurry vision, and loss of consciousness. The symptoms progress over 20-to-30 minutes, then begin to recede and are followed by a throbbing occipital headache. This patient's symptoms are most likely due to which of the following? a. Complex partial seizure b. Common migraine c. Basilar migraine d. Transient ischemic attack e. Orthostatic hypotension

The answer is c. (Ropper, pp 298-299.) As with classic migraine, with basilar migraine, women are more susceptible than men, disturbances of vision are common, the aura usually resolves within 10-to-30 minutes, and the headache invariably follows, rather than precedes, the neurological deficits; however, the character and severity of neurological deficits associated with basilar migraine are distinct. The visual change may evolve to complete blindness. Irritability may develop into psychosis. Rather than a mild hemiparesis, the patient may have a transient quadriplegia. Stupor, syncope, and even coma may appear and persist for hours.

A 56-year-old man with epilepsy is brought into the emergency room. He has been having continuous generalized tonic-clonic seizures for the past 30 minutes. He is treated with 2 mg of intravenous lorazepam. Most physicians recommend using a high dose of intravenous benzodiazepine as part of the management of status epilepticus because it has which of the following qualities? a. Ability to suppress seizure activity for more than 24 hours after one injection b. Lack of respiratory depressant action c. Rapid onset of action after intravenous administration d. Lack of hypotensive effects e. Lack of dependence on hepatic function for its metabolism and clearance

The answer is c. (Ropper, pp 304-335.) At one time the most popular benzodiazepine for use in status epilepticus was diazepam, which has a rapid onset of action in the brain, but is redistributed in the body relatively quickly. Because of this property, patients needed additional medication to protect themselves from recurrent seizure activity as early as 20 minutes after diazepam injection. A less fatsoluble benzodiazepine, lorazepam, has the advantage of acting rapidly but being cleared more slowly from the brain.

A 25-year-old woman was fired from her job after she misplaced papers vital for the company. She had had recurrent episodes for several years during which she performed nonsensical activities such as burying her plates in the backyard, hiding her underwear, and discarding her checkbook. She did not recall what she had done after performing these peculiar activities. She had been referred for psychotherapy, but the episodes became even more frequent after she was started on thioridazine. Her husband observed one episode and noted that she was unresponsive for about 5 minutes and confused for at least 1 hour. She did not fall down or remain immobile during the episodes. As the episodes became more frequent, she noticed that she would develop an unpleasant taste in her mouth, reminiscent of motor oil, just before an episode.

The answer is c. (Ropper, pp 310-313.) Complex partial seizures may be mistaken for a psychiatric problem, especially if the partial seizures do not generalize and produce tonic-clonic seizures. This patient has a typical aura involving an unpleasant smell or taste. These were once called uncinate fits, because they were ascribed to abnormal activity in the uncus of the temporal lobe. Complex partial seizures may arise from a focus of abnormal electrical activity in the temporal lobe, but they do not invariably arise from a temporal lobe focus.

A 28-year-old woman is hit in the left neck while playing lacrosse. Approximately 2 hours later she begins having language difficulties. Her speech is fluent and nonsensical. She cannot understand commands, but repeats well.

The answer is c. (Ropper, pp 466-470.) Transcortical sensory aphasia is similar to Wernicke aphasia with the exception of preserved repetition. Anatomically, the lesion generally occupies the white matter underlying the cortex of Wernicke area. In most cases, the prognosis for improvement is better than that for Wernicke aphasia.

A 35-year-old woman presents with slowly evolving left arm ataxia, left-sided head tilt, dysarthria, and left facial weakness. The patient denies vertigo, tinnitus, or hearing loss. MRI reveals a posterior fossa mass that lies close to the bone and enhances with contrast. Which of the following is the most likely explanation for this lesion? a. Cerebellar infarction b. Cerebellar hemorrhage c. Meningioma d. Schwannoma e. Astrocytoma

The answer is c. (Ropper, pp 628-630.) Any type of stroke in the cerebellum would be expected to evolve over the course of hours, rather than days or weeks. With signs and symptoms that evolve slowly, a neoplasm is more likely. Because there was no involvement of the eighth CN, the most probable neoplasm is a meningioma. This tumor also appears to arise from bone, another indication that it is most likely a meningioma. An astrocytoma is unlikely to occur in the posterior fossa, especially in adults, and would also more clearly arise from brain parenchyma rather than lying close to bone.

A 45-year-old woman presents with worsening right-sided headaches. Examination is significant for a left-sided drift. Brain MRI and CT scans reveal a homogeneously enhancing, round, duralbased, calcified lesion compressing the right frontal lobe. Which of the following is the most appropriate course of action for the management of this case? a. Anticoagulation b. Triple therapy with isoniazid, rifampin, and ethambutol c. Surgical resection d. Chemotherapy e. Craniospinal axis irradiation

The answer is c. (Ropper, pp 628-630.) The diagnosis is almost certainly a meningioma. Intraoperative confirmation of diagnosis and complete resection would be the treatment of choice. Chemotherapy is not helpful because these tumors are notoriously insensitive. Stereotactic radiosurgery may be considered in some cases, but whole craniospinal axis irradiation would not be indicated. Isoniazid, rifampin, and ethambutol may be used to treat tuberculosis. Anticoagulation is indicated for stroke prevention.

A 16-year-old boy with café au lait spots and cutaneous nodules has a gradual decrease of vision in his left eye.

The answer is c. (Ropper, pp 652-653.) The neurofibromatoses are a pair of hereditary neurocutaneous syndromes that result in a variety of congenital and later-occurring abnormalities and neoplasms affecting the skin, nervous system, and other organs. Neurofibromatosis (NF) type 1, also called peripheral NF, is characterized by café au lait spots, which are light to dark brown spots found on the skin; multiple cutaneous and subcutaneous tumors; bone cysts; sphenoid bone dysgenesis; precocious puberty; pheochromocytoma; syringomyelia; glial nodules; cortical dysgenesis; and macrocephaly. Optic nerve tumors are a particularly worrisome complication and may produce blindness in children.

An 82-year-old previously healthy woman with a recent upper respiratory infection presents with generalized weakness, headache, and blurry vision. For the past 2 weeks she has had upper respiratory symptoms that started with a sore throat, nasal congestion, and excessive coughing. She went to her primary care doctor 4 days ago and was diagnosed with sinusitis. She was given a prescription for an antibiotic and took it for 2 days, then stopped. She thereafter had chills, lightheadedness, vomiting, blurry vision, general achiness, and a headache that started abruptly and has not gotten better since. Except for blurry vision, she has not had any other visual symptoms. The blurry vision remains when she closes either eye. She also has eye tenderness with movement and mild photosensitivity. She has no drug allergies. Examination findings include temperature of 102.5°F (39.16°C), nuchal rigidity, and sleepiness. Which of the following is the next most appropriate action in this case? a. Get a brain MRI, then perform a lumbar puncture. b. Give the patient a prescription for oral azithromycin and let her go home. c. Immediately give intravenous ceftriaxone plus ampicillin. d. Immediately start intravenous acyclovir. e. Obtain CSF and blood cultures and observe the patient until the results come back.

The answer is c. (Ropper, pp 668-678.) The immediate concern is that the patient has bacterial meningitis, and she should be treated. A lumbar puncture and blood drawn to obtain cultures should be done; however, it can take a few days for the results to come back, and it may be too late for the patient by then. Oral azithromycin is not the proper treatment for bacterial meningitis. Intravenous acyclovir would be used to treat herpes encephalitis.

A 39-year-old man was treated 1 year ago for a brain abscess. He has largely recovered, but still has occasional word-finding difficulties. Which of the following is the most common symptom in patients with brain abscess? a. Nausea and vomiting b. Ataxia c. Headache d. Neck stiffness e. Seizures

The answer is c. (Ropper, pp 683-686.) Three-fourths of patients with brain abscess have headache. This usually develops within a few weeks of the appearance of the abscess. Only one-third of patients present with seizures or focal neurological deficits. Only one-fourth exhibit papilledema. Brain abscesses may produce remarkably few changes in the CSF until the abscess penetrates into the subarachnoid or intraventricular space. Abscesses that have not yet communicated with the CSF will usually produce only a moderate elevation in the CSF protein content. If the abscess is unsuspected and untreated, it will usually extend to the ventricles. With perforation into the ventricle, the abscess usually proves fatal. The treatment of choice for brain abscess is surgical resection.

A 27-year-old man presents to his primary care doctor with a low-grade fever, headache, and neck stiffness, which have become more bothersome over the past 1-to-2 weeks. CSF and serological testing for Lyme disease is positive, and antibiotic treatment is initiated. The cranial neuropathy most commonly found with Lyme disease is that associated with damage to which CN? a. III b. V c. VII d. IX e. XII

The answer is c. (Ropper, pp 697-699.) Facial weakness may be the only neurological sign of Lyme disease. The neurological deficits usually appear weeks after the initial rash. Untreated neurological disease may persist for months. The facial palsy or optic neuritis that develops with CNS disease is characteristically associated with meningitis.

A 72-year-old right-handed woman has 2 days of headache and fever, followed by worsening confusion. She is taken to the hospital after having a generalized seizure. A head CT is consistent with left temporal hemorrhage and swelling. Localization of encephalitis to the medial temporal or orbital frontal regions of the brain is most consistent with which of the following? a. Treponema pallidum b. Varicella zoster virus c. Herpes simplex virus d. Cryptococcus neoformans e. Toxoplasma gondii

The answer is c. (Ropper, pp 719-721.) Herpes simplex type 1 is the strain usually responsible for a herpetic encephalitis. Type 2 may occur in newborns who have been exposed during passage through the birth canal of a woman with genital herpes. Persons with AIDS are also at risk for either type 1 or type 2. Temporal lobe involvement in the immunocompetent patient may produce unilateral swelling and hemorrhage into the temporal lobe.

A 52-year-old woman with acquired immune deficiency syndrome (AIDS) presents to the emergency room with mild left hemiparesis and altered mental status. A CT scan reveals several rim enhancing lesions with minimal mass effect. Which of the following is the best next step in management? a. Get a cerebral angiogram. b. Order a ventricular CSF aspiration. c. Perform a lumbar puncture and include CSF for Epstein-Barr virus (EBV) PCR in tests ordered. d. Stop all antiretroviral therapy. e. Treat with intravenous acyclovir.

The answer is c. (Ropper, pp 726-730.) The most common etiologies of rim-enhancing brain lesions in AIDS patients are primary CNS lymphoma and Toxoplasma gondii infection. Other etiologies such as bacterial or fungal abscess are also possible. CSF EBV PCR test is highly sensitive and specific for primary CNS lymphoma. Because there is no mass effect, it is safe to do a lumbar puncture, so a ventricular CSF aspiration is not necessary. A cerebral angiogram should be done if you suspect an aneurysm or vascular malformation. These are unlikely in this case. There is no reason to stop all antiretroviral therapy. Intravenous acyclovir is used to treat herpes encephalitis, which is unlikely in this case.

A 61-year-old man with a history of hypertension has been in excellent health until he presents with vertigo and unsteadiness lasting for 2 days. He then develops nausea, vomiting, dysphagia, hoarseness, ataxia, left facial pain, and right-sided sensory loss. There is no weakness. On examination, he is alert, with a normal mental status. He vomits with head movement. There is skew deviation of the eyes, left ptosis, clumsiness of the left arm, and titubation. He has loss of pin and temperature sensation on the right arm and leg and decreased joint position sensation in the left foot. He is unable to walk. Magnetic resonance imaging (MRI) in this patient might be expected to show which of the following? a. Basilar artery tip aneurysm b. Right lateral medullary infarction c. Left lateral medullary infarction d. Left medial medullary infarction e. Right medial medullary infarction

The answer is c. (Ropper, pp 748-772.) Wallenberg, or lateral medullary, syndrome is caused by infarction involving some or all of the structures located in the lateral medulla, including the nucleus and descending tract of the fifth nerve, the nucleus ambiguus, lateral spinothalamic tracts, inferior cerebellar peduncle, descending sympathetic fibers, vagus, and glossopharyngeal nerves. The patient with Wallenberg syndrome has ipsilateral ataxia and ipsilateral Horner syndrome. The trigeminal tract damage may produce ipsilateral loss of facial pain and temperature perception and ipsilateral impairment of the corneal reflex. The lateral spinothalamic damage produces pain and temperature disturbances contralateral to the injury in the limbs and trunk. Dysphagia and dysphonia often develop, with damage to the ninth and tenth nerves.

A 67-year-old woman with a history of type 2 diabetes mellitus and atrial fibrillation presents to the emergency room with left body weakness and slurred speech. The onset was sudden while she was brushing her teeth 1 hour ago, and she was brought immediately to the emergency room. She denies word-finding difficulties, dysesthesia, and headache. She is taking warfarin. Physical examination findings include blood pressure of 205/90 mm Hg and irregularly irregular heartbeat. There is left-side neglect with slurred speech. There is a corticospinal pattern of weakness of the left body, with the face and upper extremity being worse than the lower extremity. Routine chemistries and cell counts are normal. Her international normalized ratio (INR) is 1.8. Which of the following is the most appropriate first step in management? a. Administer tissue plasminogen activator. b. Call a vascular surgery consult for possible endarterectomy. c. Order a brain computed tomography (CT). d. Order a cerebral angiogram. e. Start heparin.

The answer is c. (Ropper, pp 774-775.) This is a good history for cardioembolic stroke—sudden onset, cortical symptoms, atrial fibrillation, and subtherapeutic INR. The immediate goal should be to rule out an intracranial hemorrhage and confirm the diagnosis. Tissue plasminogen activator is the treatment for acute stroke in specific circumstances. However, it is not yet certain that this is a stroke. It may be an intracranial hemorrhage, which would be a contraindication for tissue plasminogen activator. Additionally, an elevated INR in a patient on warfarin is a contraindication for tissue plasminogen activator. Carotid endarterectomy is indicated for some cases when a transient ischemic attack or stroke is believed to be caused by carotid artery narrowing. It is not yet known what caused this patient's event, and this procedure would rarely be done emergently. A cerebral angiogram would be indicated if there was a strong suspicion of an aneurysm or vascular malformation. There is no reason to believe one of these is causing the patient's symptoms. Heparin may be indicated if there is not an intracranial hemorrhage. This must first be established by CT or MRI.

A thorough evaluation reveals that a 69-year-old patient has a symptomatic 90% stenosis of the right internal carotid artery at the bifurcation. Which of the following management options is most likely to prevent a future stroke? a. Warfarin b. Carotid artery angioplasty c. Carotid endarterectomy d. Extracranial-intracranial bypass e. Aspirin

The answer is c. (Ropper, pp 783, 788, 789.) Based on the results of the North American Symptomatic Carotid Endarterectomy Trial (NASCET), it is known that carotid endarterectomy can reduce the risk of stroke in patients with symptomatic stenosis by 70% or more. The risk of ipsilateral stroke was reduced from 26% in the medically treated group to 9% in the surgically treated group. Carotid endarterectomy should be offered to all eligible patients with symptomatic disease of the internal carotid artery. Carotid angioplasty with stenting is an alternative for management of these patients. However, it is less established than endarterectomy. Angioplasty without stenting is not indicated. Extracranial-intracranial bypass has been tried unsuccessfully, although it may still play a role for certain patients with inaccessible lesions or hypoperfusion in the setting of complete occlusions. Aspirin would be appropriate after endarterectomy.

Three days after a subarachnoid hemorrhage, a patient begins to develop neck stiffness and photophobia. This is followed by left-sided weakness and hyperreflexia. Her left plantar response is upgoing. Her physician presumes that these deficits are a delayed effect of the subarachnoid blood. Which of the following is the most appropriate treatment? a. Heparin b. Warfarin c. Nimodipine d. Phenytoin e. Carbamazepine

The answer is c. (Ropper, pp 808-816.) Vasospasm is a relatively common complication of subarachnoid blood and may result in stroke. Nimodipine is used because it decreases the probability of stroke, but it does not prevent it completely. Anticoagulation with heparin or warfarin worsens the patient's prospects because it increases the risk of additional bleeding. Antiepileptic drugs, such as phenytoin and carbamazepine, may reduce the risk of seizure associated with subarachnoid blood and are sometimes given prophylactically. This patient does not have evidence of seizures, however.

A 42-year-old man presents to the emergency room with seizures, mental status change, and vision difficulties. A magnetic resonance imaging (MRI) reveals an abnormally high T2 signal in the posterior cerebral white matter. There is proteinuria, and blood pressure is 210/120 mm Hg. The cerebrospinal fluid (CSF) protein content of this patient is likely to be which of the following? a. Abnormally low b. Normal c. Elevated, but less than 100 mg/dL d. Elevated to between 500 and 1000 mg/dL e. Greater than 2000 mg/dL

The answer is c. (Ropper, pp 822-824.) The elevation with hypertensive encephalopathy is variable because intracranial hemorrhage may occur with the hypertensive crisis, but most patients will have moderate increases in CSF protein. Lowering of the blood pressure may reduce the protein elevation.

A 65-year-old diabetic man has a history of a cerebellar stroke. The stroke occurred 5 years ago, and he says that he has now fully recovered. He cannot recall the symptoms, but his medical records state that he presented with left-sided dysdiadochokinesia. Which of the following was most likely impaired? a. Successive finger movements b. Heel-to-toe walking c. Rapid alternating movements d. Tremor suppression e. Conjugate eye movements

The answer is c. (Ropper, pp 83-84.) Dysdiadochokinesia is usually apparent with cerebellar damage. It is most evident when strength and sensation are intact. Alternately tapping one side of the hand and then the other, or tapping the heel and then alternating with the toe of the foot, is the test usually employed to check this aspect of coordination. Multiple sclerosis in adults and cerebellar tumors in children are two of many causes of problems with this part of the neurologic examination. Focal lesions in the nervous system may produce highly asymmetric dysdiadochokinesia. A variety of movement disorders, such as Parkinsonism and choreoathetosis, may interfere with rapid alternating movements and give the false impression that the patient has a lesion in systems solely responsible for coordination.

A 26-year-old man develops bed wetting and transient sexual dysfunction that resolves over the course of 6 weeks. One month later, he notices a pins-and-needles sensation in his right leg that never clears completely. On examination, he has hyperreflexia in both of his legs and past-pointing in his right arm. His gait is slightly ataxic, and he is unable to perform tandem gait.

The answer is c. (Ropper, pp 883, 888.) This young man has signs and symptoms of multiple sclerosis that are largely referable to the spinal cord. Gait ataxia is an especially common presenting complaint. Impotence is troublesome and common. The CSF fluid picture is distinctive in its elevation of the gamma globulin content. Oligoclonal banding studies of the fluid would most likely be positive.

A 9-year-old boy is brought to your clinic by his parents because he has begun to have episodes of eye fluttering lasting several seconds. Sometimes he loses track of his thoughts in the middle of a sentence. There was one fall off a bicycle that may have been related to one of these events. There are no other associated symptoms, and the episodes may occur up to 20 or more times per day. The boy's development and health have been normal up until this point. He had two head injuries as a young child: the first when he fell off a tricycle onto the ground, and the second when he fell off a playset onto his head. Both episodes resulted in a brief loss of consciousness and he did not think clearly for part of the day afterward, but he had no medical intervention. Which of the following test results is most likely? a. Electroencephalogram (EEG) showing 1-2 Hz spike wave b. Brain magnetic resonance imaging (MRI) showing widespread abnormalities c. EEG showing 2-3 Hz spike wave d. Lumbar puncture with high white blood cell (WBC) count e. Conners Rating Scale abnormalities reported by parents, but not teachers

The answer is c. (Swaiman, pp 1019-1033.) This is a common presentation for primary generalized epilepsy of childhood. An EEG showing the classic 3 Hz spike-and-wave pattern would confirm this diagnosis. 1-2 Hz would be consistent with severe neurological dysfunction and symptomatic generalized epilepsy. Brain MRI would be normal or show an incidental finding. Anatomic problems can cause seizures, but these tests will not provide any information about brain electrical activity. Lumbar puncture is useful for measuring cerebrospinal fluid (CSF) pressure and looking for central nervous system (CNS) inflammation or infection. CNS inflammation or infection may cause seizures. Conners Rating Scale is useful for the evaluation of inattention and hyperactivity.

A 75-year-old man with a history of recent memory impairment is admitted with headache, confusion, and a left homonymous hemianopsia. He has recently had two episodes of brief unresponsiveness. There is no history of hypertension. CT scan shows a right occipital lobe hemorrhage with some subarachnoid extension of the blood. An MRI scan with gradient echo (susceptibility) sequences reveals foci of hemosiderin in the right temporal and left frontal cortex. Which of the following is the most likely cause of this patient's symptoms and signs? a. Gliomatosis cerebri b. Multi-infarct dementia c. Mycotic aneurysm d. Amyloid angiopathy e. Undiagnosed hypertension

The answer is d. (Osborn, pp I[4]:72-75.) Cerebral amyloid angiopathy, or congophilic angiopathy, is the most common cause of lobar hemorrhage in elderly patients without hypertension. The deposition of β-amyloid protein (the same as that found in Alzheimer disease) in blood vessels of the brain leads to disruption of the vessel walls, which predisposes them to hemorrhage. Patients are usually more than 70 years old and may present with multiple cortical hemorrhages, with or without a history of dementia. At times, additional hemorrhages may be seen only on special imaging techniques, such as gradient echo MRI, which magnifies the effects of hemosiderin in regions of prior hemorrhage.

A 70-year-old man with a history of lung cancer develops nausea and vomiting and then becomes lethargic. On examination, he is lethargic but arousable, disoriented, and inattentive. He is weak proximally and has diminished reflexes.

The answer is d. (Ropper, p 1097.) Hypercalcemia may occur as a complication of cancer in up to 5% of patients. It may be a result of parathyroid-related peptide secreted by the tumor itself (usually a lung cancer) or of bone destruction by metastatic disease. The elevated serum calcium decreases membrane excitability, leading to the clinical syndrome of fatigability, lethargy, generalized weakness, and areflexia. In more severe cases, coma and even convulsions can occur. Symptoms usually do not occur until levels reach 14 mg/dL or higher.

A 4-year-old boy develops progressive gait ataxia and limb weakness over the course of 3 months. Neurological assessment reveals diffusely absent deep tendon reflexes, proximal muscle weakness, ophthalmoparesis, and poor pain perception in the feet. Blood tests reveal elevated creatine phosphokinase levels and abnormally high serum bilirubin levels. Further investigations of hepatic function reveal that the child has a cholestatic hepatobiliary disorder, but there is no evidence of hepatic dysfunction sufficient to cause an encephalopathy.

The answer is d. (Ropper, p 1109 .) Vitamin E deficiency that causes neurological disease is rare, but when it does develop it is usually during early childhood. The most common syndrome involves spinocerebellar degeneration, polyneuropathy, and pigmentary retinopathy. Clarke's columns, the spinocerebellar tracts, the posterior columns, the nuclei of Goll and Burdach, and sensory roots are especially likely to exhibit degeneration in persons with vitamin E deficiency. The most obvious symptom of the deficiency is likely to be ataxia.

A 28-year-old man who has recently immigrated from Brazil presents with 3 months of fluctuating but slowly progressive bilateral lower extremity weakness, a little worse on the left side than on the right. After a complete evaluation, a parasite is diagnosed as the etiology. This organism's ova usually damage the nervous system at the level of which of the following? a. Cerebrum b. Cerebellum c. Basal ganglia d. Spinal cord e. Peripheral nerves

The answer is d. (Ropper, p 1196.) Schistosoma mansoni is endemic in the tropics and may produce a subacutely evolving paraparesis. The fluke itself does not invade the spinal cord, but it deposits eggs in the valveless veins of Batson, which drain the intestines and communicate with the drainage from the lumbosacral spinal cord. The patient develops granulomas around the ova that lodge in the spinal cord, and these granulomatous lesions crush the cord.

A 37-year-old alcoholic man awakes with clumsiness of his right hand. Neurologic examination reveals poor extension of the hand at the wrist. He most likely has injured which one of the following nerves? a. Median nerve b. Brachioradialis nerve c. Musculocutaneous nerve d. Radial nerve e. Ulnar nerve

The answer is d. (Ropper, p 1314.) Radial nerve injuries are fairly common in alcoholic persons who may have lost consciousness in awkward positions. These are sometimes referred to as Saturday night palsies. The injury is usually a pressure palsy and produces a wristdrop. The nerve is injured as it courses near the spiral groove of the humerus.

A 27-year-old woman presents to the emergency room following a generalized tonic-clonic seizure that began focally in her left lower extremity. Although the seizure stopped within 1 minute, there was persistent weakness of the left lower extremity lasting several hours. Further testing revealed a small arteriovenous malformation near the motor cortex. Focal weakness lasting for 24 hours following a motor seizure is most likely attributable to which of the following? a. Intracerebral hemorrhage b. Subarachnoid hemorrhage c. Encephalitis d. Todd paralysis e. Hyponatremia

The answer is d. (Ropper, p 309.) Weakness after seizure activity is evidence of a postictal paralysis, or Todd paralysis. Postictal weakness does not suggest bleeding or new areas of cerebrocortical damage, but imaging with CT scan is appropriate to exclude these possibilities. Postictal paralysis may last for many hours, or even days. The precise cause is unknown, but it appears to be caused by some kind of neuronal exhaustion occurring after frequent repetitive discharges. It may reflect depletion of glucose in the neurons in the epileptic focus.

A 38-year-old man, who is immunocompromised because of HIV, presents with 1 month of worsening right headache, ear pain, and fever. He is determined to have malignant external otitis and osteomyelitis of the base of the skull. Culture of the lesion reveals a fungal etiology. What is the most likely causative organism? a. Nocardia b. Cryptococcus neoformans c. Actinomyces d. Aspergillus e. Candida

The answer is d. (Ropper, p 701.) Patients with brain abscesses may exhibit focal signs, seizures, delirium, or less specific neurological findings. These patients often have fever, but the CSF may not reflect the infectious basis of the fever until the abscess ruptures into the ventricles or subarachnoid space. Although Aspergillus is the most common cause of fungal abscesses, it is a relatively uncommon cause of fungal meningitis or meningoencephalitis. Nocardia is not classified as a fungus despite its resemblance to a fungus.

A 59-year-old right-handed woman has been clinically diagnosed with encephalitis. While CSF and MRI studies are pending, a medical student suggests ordering an EEG. Which of the following EEG findings is most associated with herpes encephalitis? a. α Activity over the frontal regions b. β Activity over the temporal regions c. Three-per-second spike-and-wave discharges d. Bilateral, periodic epileptiform discharges e. Unilateral δ activity over the frontal region

The answer is d. (Ropper, p 720.) The periodic discharges seen with herpes encephalitis typically occur over the temporal regions. Slow waves may be evident over the temporal lobes in many persons with severe disease. Seizures commonly occur early in the course of herpes encephalitis, so the EEG may be severely disturbed generally.

A 16-year-old girl with complex partial seizures and mild mental retardation has a birthmark consisting of deep red discoloration extending over her forehead and left upper eyelid. A CT scan of her brain would be likely to reveal which of the following? a. A hemangioblastoma b. A Charcot-Bouchard aneurysm c. An arteriovenous malformation d. A leptomeningeal angioma e. A fusiform aneurysm

The answer is d. (Ropper, p 985.) This patient has encephalofacial angiomatosis (Sturge-Weber syndrome), a congenital disturbance that produces facial cutaneous angiomas with a distinctive and easily recognized appearance, along with intracranial abnormalities such as leptomeningeal angiomas. Persons with the syndrome may be mentally retarded and often exhibit hemiparesis or hemiatrophy on the side of the body opposite the port-wine nevus. Both men and women may be affected, and seizures may develop in affected persons. The nevus associated with Sturge-Weber syndrome usually extends over the sensory distribution of the first division of the trigeminal nerve. The lesion usually stays to one side of the face. Affected persons will usually also have an angioma of the choroid of the eye. Intracranial angioma is unlikely if the nevus does not involve the upper face. Deficits develop as the person matures and may be a consequence of focal ischemia in the cerebral cortex that underlies the leptomeningeal angioma. Hemangioblastomas are vascular tumors seen in association with polycystic disease of the kidney and telangiectasias of the retina (von Hippel-Lindau syndrome). Charcot-Bouchard aneurysms are very small and may be microscopic. They develop in patients with chronic hypertension and most commonly appear in perforating arteries of the brain. The lenticulostriate arteries are most commonly affected. Hemorrhage from these aneurysms is likely, and the putamen is the most common site for hematoma formation. Hemorrhage may extend into the ventricles and lead to subarachnoid blood. Other locations commonly affected include the caudate nucleus, thalamus, pons, and cerebellum. The dentate nucleus of the cerebellum is especially susceptible to the formation of Charcot-Bouchard aneurysms. Fusiform aneurysms are diffusely widened arteries with evaginations along the walls, but without stalks such as occur with the typical berry-shaped structures of the saccular aneurysm. This type of aneurysm may be a late consequence of arteriosclerotic damage to the artery wall.

A 62-year-old right-handed man has "involuntary twitches" of his left hand. He first noticed between 6 months and 1 year ago that when he is at rest, his left hand shakes. He can stop the shaking by looking at his hand and concentrating. The shaking does not impair his activities in any way. He has no trouble holding a glass of water. There is no tremor in his right hand, and the lower extremities are not affected. He has had no trouble walking. There have been no behavioral or language changes. On examination, a left hand tremor is evident when he is distracted. Handwriting is mildly tremulous. He is very mildly bradykinetic on the left. The most likely examination finding would be which of the following? a. Upper motor neuron pattern of weakness on the left b. Lower motor neuron pattern of weakness on the left c. Bilateral upper motor neuron pattern of weakness d. Mild cogwheel rigidity on the left only with distraction e. Bilateral severe cogwheel rigidity

The answer is d. (Ropper, pp 1033-1045.) This patient gives a typical history for early Parkinson disease. The classic triad is asymmetric resting tremor, rigidity, and bradykinesia. The rigidity is generally severe later, not early in the disease. Answers a-c are incorrect as weakness is not a feature of Parkinson disease, and there is no suggestion of any type of weakness in the history.

A patient has had progressive, chronic liver failure for the past 5 years. At the time of death, he would be expected to exhibit changes in which type of brain cells? a. Oligodendrocytes b. Striatal neurons c. Pigmented cells of the substantia nigra d. Astrocytes e. Inferior olivary neurons

The answer is d. (Ropper, pp 1090-1093.) Long-standing hepatic disease may produce a profound encephalopathy, but changes in the brain are notably sparse with portal-systemic encephalopathy. The most obvious change is an increase in Alzheimer type II astrocytes. These astrocytes are relatively large cells. Rarely patients show more dramatic changes, which include neuronal loss and focal necrosis. With chronic alcoholism and hepatic insufficiency, patients exhibit a loss of Purkinje cells in the cerebellum, but this is a consequence of alcohol toxicity or thiamine deficiency rather than of toxic injury from the hepatic dysfunction.

A 75-year-old retired journalist is generally healthy but has noticed worsening problems maneuvering over the past 4 months. He has particular trouble getting out of low seats and off toilets. He most likely has which of the following? a. Poor fine finger movements b. Electromyography results indicating widespread denervation c. Distal muscle weakness d. Elevated creatine kinase e. Gait apraxia

The answer is d. (Ropper, pp 1355-1361.) With primary muscle diseases, such as polymyositis, weakness usually develops in proximal muscle groups much more than in distal groups. This means that weakness will be most obvious in the hip girdle and shoulder girdle muscles. The hip girdle is usually affected before the shoulder girdle. To get out of a low seat, the affected person may need to pull him- or herself up using both arms. Persons with more generalized weakness or problems with coordination are less likely to report problems with standing from a seated position. Poor rapid alternating movements and poor fine finger movements usually develop with impaired coordination, such as that caused by cerebellar damage. With severe weakness in the limbs, patients will do poorly on these tests of function as well. With proximal muscle weakness, the affected person will usually perform relatively well on these tests of distal limb coordination.

A 32-year-old intravenous drug abuser presents with more than 2 weeks of left body weakness. Brain CT scan reveals several ring-enhancing lesions, and an HIV test is positive. Serological, CSF, and MRI testing support the diagnosis of an obligate intracellular parasite. Which of the following is the best treatment for HIV associated with this opportunistic infection? a. Intravenous acyclovir b. Neurosurgical removal of the lesions c. Oral fluconazole d. Sulfadiazine and pyrimethamine e. Thiabendazole

The answer is d. (Ropper, pp 726-730.) The combination of sulfadiazine and pyrimethamine is proper treatment for T gondii infection. Neurosurgical removal of the lesions is not indicated. Oral fluconazole is a treatment for fungal infections. Intravenous acyclovir is used to treat herpes encephalitis. Thiabendazole is used to treat helminth infections. Other possible causes of ringenhancing lesions on CT scan include bacterial abscess or malignancy.

A 16-year-old girl has been diagnosed with migraine headaches. She has identified some triggers and made lifestyle changes, but still has 14 headaches per month. Appropriate long-term management might include a prescription for daily use of which of the following medications? a. Metoclopramide hydrochloride b. Sumatriptan c. Oral contraceptives (OCPs) d. Amitriptyline hydrochloride e. Ergotamine tartrate

The answer is d. (Ropper, pp 166-174.) Several medications are effective as prophylactic agents in the treatment of migraine. These include amitriptyline hydrochloride, propranolol, verapamil, and valproate. Most experts recommend initiating prophylactic therapy only when headaches occur at least one to two times per month. Metoclopramide hydrochloride, sumatriptan, and ergotamine tartrate are appropriately used to treat an acute attack of migraine and should not be prescribed on a daily basis. Daily use of these medications can establish a rebound syndrome that results in a chronic daily headache. OCPs may be associated with either an increase or decrease in the frequency of migraines, but are not generally used as a treatment for migraine. Some experts recommend not prescribing OCPs for patients with migraine for fear of increasing the risk of a stroke, although OCPs are probably safe to use in most patients with common migraine.

A 42-year-old man presents with a sudden and severe headache associated with nausea. The headache reaches maximal intensity within 5 seconds. He has no prior history of headache. Examination is unremarkable. CT and spinal fluid examination show no evidence of blood. He later admits that he had been engaged in sexual activity when the headache occurred.

The answer is d. (Ropper, pp 180-181.) Thunderclap headache refers to the syndrome of the sudden onset of a very severe headache with no apparent structural cause. When a patient presents with "the worst headache of my life," the initial concern should always be for a subarachnoid hemorrhage, particularly in the presence of meningismus, focal deficits, or a change in the level of consciousness. CT scanning is indicated to exclude hemorrhage, but because CT may be negative in up to 5% to 10% of cases of subarachnoid hemorrhage, lumbar puncture is necessary if CT is negative to exclude small amounts of blood. Some reports have suggested that even in the absence of blood on a lumbar puncture, an underlying aneurysm may still be the cause of acute, severe headache, because sudden changes in the wall of the aneurysm may provoke severe pain. These reports would suggest that angiography should be performed in all such patients to exclude aneurysm. It remains unknown, however, whether these cases represent coincidental occurrence of thunderclap headache and an incidental, asymptomatic aneurysm. Several series have shown that many patients with thunderclap headache tend to go on to develop more typical migraine, raising the possibility that the thunderclap headache is simply the initial presentation of their migraine.

A 29-year-old woman comes to the emergency room with facial pain of new onset. She has stabbing pains on the left side of her face just below her eye. These last less than 1 second at a time, but are so severe that she winces involuntarily with each pain. The pain seems to be triggered by drinking cold fluids. The only other problems she has noticed are clumsiness in her right hand and blurred vision in her right eye. Both of these have been present for more than 2 years and have not interfered with her normal activities.

The answer is d. (Ropper, pp 182-183.) Trigeminal neuralgia may develop in the context of multiple sclerosis—an association suggested by this woman's other neurological problems. The development of trigeminal neuralgia (tic douloureux) indicates that demyelination has probably extended to the brainstem and may be involving trigeminal nerve connections. A more detailed history would probably reveal that the patient has had pain in the eye that now has disturbed vision. This is expected with the optic neuritis, which is typically associated with multiple sclerosis. Other symptoms commonly reported at this age by patients with previously undiagnosed multiple sclerosis include bed wetting, changes in speech, and gait instability.

A 46-year-old longshoreman has lower back pain radiating down the posterior aspect of his left leg and paresthesias in the lateral aspect of his left foot. This has been present for 6 months. Strength and bowel and bladder function have been normal. Examination would be most likely to show which of the following? a. Left Babinski sign b. Loss of pinprick sensation over the web space between the first and second digits of the left foot c. Hyperreflexia at the left knee jerk d. Hyporeflexia in the left Achilles tendon reflex e. Decreased rectal tone

The answer is d. (Ropper, pp 196-200.) This patient has a history most consistent with a herniated lumbar disk. The most common locations for lumbar disk herniation are between the fifth lumbar and first sacral vertebrae (producing S1 nerve root compression) and between the fourth and fifth lumbar vertebrae (producing L5 root compression). S1 nerve root compression, or radiculopathy, is associated with pain in the lower back or buttock region, often radiating down the posterior thigh and calf to the lateral and plantar surfaces of the foot and affecting the fourth and fifth digits of the foot. Motor function of the foot and toe flexors, toe abductors, and hamstring muscles may be impaired, but more often it is not. Bowel and bladder function are usually preserved. The loss of the ankle jerk, or Achilles tendon reflex, is often the only objective sign of S1 radiculopathy. Deep tendon reflexes such as the ankle jerk are diminished or lost when there is damage to the sensory fibers from the tendon stretch organs. A Babinski sign is an indication of upper motor neuron damage, which is not expected in this case with preserved autonomic and motor function. Loss of pinprick sensation over the web space between first and second toes is found in association with injury to the fifth lumbar nerve root or to the peroneal nerve. Hyperreflexia of the knee jerk is another sign of an upper motor neuron lesion and would not be expected in this case. Straight-leg raising (Lasègue maneuver) is used to determine whether symptoms are caused by nerve root compression by stretching the nerve root. With compression, lifting the leg passively may be limited to between 20° and 30°. This test may be positive on the contralateral side (crossed straight-leg-raising sign), but it is usually more prominent on the affected side.

A 28-year-old graduate student presents with confusion and mild right hemiparesis developing over the course of an evening. His girlfriend relates that he has been having severe headaches each morning for the past 2 weeks. While being evaluated in the emergency room, he has a generalized tonic-clonic seizure. When examined 2 hours later, he is lethargic and unable to recall recent events, has difficulty naming, and has a right pronator drift. There is mild weakness of abduction of the eyes bilaterally. Funduscopic examination might be expected to show which of the following? a. Pigmentary degeneration of the retina b. Hollenhorst plaques c. Retinal venous pulsations d. Blurring of the margins of the optic disc e. Pallor of the optic disc

The answer is d. (Ropper, pp 236-238.) The presentation with sub-acute onset of morning headaches culminating in confusion, right hemiparesis, and seizure in a young person suggests an expanding mass lesion, most likely a tumor. The weakness of eye abduction bilaterally is what may be referred to as a false localizing sign. Although this suggests injury to the sixth CN bilaterally, the injury is not restricted to the sixth CN. The increase in intracranial pressure (ICP) from the mass causes stretching of the sixth nerve fibers, which consequently leads to their dysfunction. Diplopia may be appreciated only on lateral gaze, which requires full function of the sixth nerve. Funduscopic examination in such a case would most likely reflect changes of increased ICP. The first sign of this is usually blurring of the margins of the optic disc and elevation of the disc because of swelling. Changes in the optic disc—the area in which all the nerve fibers from the retina come together and exit as the optic nerve—occur with problems other than increased ICP (such as optic neuritis), but blurring of the margins should be routinely considered a sign of increased ICP. This is especially true if the appearance of the disc has changed in association with the development of headache, obtundation, and vomiting. Pigmentary degeneration of the retina may occur with some infections, such as congenital toxoplasmosis or cytomegalovirus, or as part of a hereditary metabolic disorder, as in retinitis pigmentosa. Hollenhorst plaques are cholesterol and calcific deposits seen in the retinal arterioles in the setting of atheroembolism to the eye, along with visual loss. Retinal venous pulsations are typically not present when there is increased ICP, although they may also be absent in up to 15% of normal individuals.

A 38-year-old woman says that she is "dizzy." A more careful history reveals that she has an abnormal sensation of movement intermittently. Examination reveals several beats of horizontal rhythmic eye movements on leftward gaze. A T1 MRI image from this patient is shown. Dix-Hallpike test is positive. Which is the most likely diagnosis? a. Ocular bobbing b. Pontine hemorrhage c. Cervicomedullary junction glioma d. Benign positional vertigo e. Brainstem stroke

The answer is d. (Ropper, pp 293-296.) Abnormal patterns of eye movement may help localize disease in the central or peripheral nervous system in patients with vertigo. The retina is negatively charged in comparison with the cornea, which creates a dipole that may be monitored during electronystagmography studies with electrodes placed on the skin about the eyes. Movement of the most posterior elements of the retina toward an electrode is registered as a negative voltage change at that electrode. Damage to the pons may produce characteristic conjugate deviations of the eyes. The conjugate eye movements are rhythmic and directed downward, but they lack the rapid component characteristic of nystagmus. This type of abnormal eye movement is called ocular bobbing. A lesion at the cervicomedullary junction, such as a meningioma at the foramen magnum, will produce a downbeating nystagmus with both eyes rhythmically deviating downward, with the rapid component of this nystagmus directed downward as well. Cervicomedullary refers to the cervical spinal cord and the medulla oblongata. Damage to the midbrain, thalamus, or hypothalamus may disturb eye movements, but down-beating nystagmus would not ordinarily develop with damage to these structures. The pictured MRI is entirely normal. Benign positional vertigo is thought to be caused by otolithic debris within the semicircular canals. In addition to facilitating acclimation to the vertigo, the Dix-Hallpike maneuver may be able to clear the debris from the canal.

A 27-year-old man begins to experience infrequent episodes of nausea, warmth rising through his body, and an unusual odor like rotting fish. His girlfriend notices that afterward he may develop twitching of the right side of his face and an inability to speak for several minutes. Afterward the man appears dazed and cannot remember what has occurred. He has otherwise been well. MRI of his brain is most likely to show a lesion in which of the following areas? a. Left occipital lobe b. Right frontal lobe c. Cribriform plate d. Hippocampus e. Left parietal lobe

The answer is d. (Ropper, pp 310-313.) Many patients with complex partial seizures have a preseizure phenomenon (the aura) that alerts them to an impending seizure. The aura is in fact a simple partial seizure. This patient's aura includes an olfactory hallucination, which is usually associated with lesions of the mesial temporal lobe, particularly the hippocampus or parahippocampal gyrus. Diseases that can affect that region include tumors, trauma, and MTS. Hippocampus means "seahorse" in Latin.

A 16-year-old boy with a history of acute viral myocarditis requires placement of a left ventricular assist device. He has a complicated postoperative course, with fever, bacteremia, and renal failure. On postoperative day 10, he develops continuous rhythmic jerking of the left corner of the mouth, associated with jerking of the left thumb. This persists for 24 hours. He is alert, able to follow commands, and has no gaze deviation. CT shows a small hemorrhagic infarction of the right posterior frontal region.

The answer is d. (Ropper, pp 314-315.) Epilepsia partialis continua refers to a condition of persistent focal motor seizure activity—in essence, a focal motor status epilepticus. The distal hand and foot muscles are most frequently affected. Active or passive movement of the limb may exacerbate the seizure activity. The seizures may persist for hours or for months. The response to therapy is often poor.

A 35-year-old man presented to the emergency room with the acute onset of right body weakness. A diffusion-weighted MRI was positive and is in part shown below. Further imaging sequences indicated a small left frontal intraparenchymal hemorrhage. Within 1 day of admission, the patient's right-sided weakness began to abate, and within 1 week it almost completely resolved. On the sixth day of hospitalization, the patient abruptly lost consciousness and exhibited clonic movements starting in his right side and generalizing to his left side. The movements stopped within 3 minutes, but he had residual right-sided weakness for 24 hours. A head CT scan was unchanged from admission. The most appropriate treatment to institute involves which of the following? a. Heparin b. Recombinant tissue plasminogen activator (r-TPA) c. Lamotrigine d. Levetiracetam e. Warfarin

The answer is d. (Ropper, pp 332, 334.) Anticoagulation with warfarin or heparin and thrombolysis with r-TPA or urokinase are contraindicated in anyone with an intracranial hemorrhage. Focal seizures that secondarily generalize after an intracerebral or subarachnoid hemorrhage occur frequently and are appropriately treated with an antiepileptic drug, such as levetiracetam. Lamotrigine is an anticonvulsant, but would be a very poor choice in this case because this patient needs a drug that will be immediately therapeutic. Lamotrigine must be slowly titrated over many weeks when first started because of the risk of severe rash.

A 53-year-old woman sustains a small left frontal embolic stroke during cardiac catheterization. She has poor naming ability and is nonfluent. Comprehension and repetition are relatively preserved.

The answer is d. (Ropper, pp 466-470.) Transcortical motor aphasia is similar to Broca aphasia with the exception of preserved repetition. Anatomically, the lesion generally occupies left frontal white matter and spares the overlying cortex.

A 37-year-old man presents with visual impairment. Examination reveals a bitemporal hemianopsia. Which of the following tumors is most likely responsible for this finding? a. Optic glioma b. Occipital astrocytoma c. Brainstem glioma d. Pituitary adenoma e. Sphenoid wing meningioma

The answer is d. (Ropper, pp 648-651.) With bitemporal hemianopsia, the visual fields in both eyes are impaired, but only the temporal quadrants of the field in each eye are affected. Pressure on the optic chiasm inferiorly by a tumor arising in or near the sella turcica will crush the fibers crossing in the chiasm from the medial aspects of the optic nerves. The most medial fibers in both optic nerves are contributed by the nasal aspects of the retina. The nasal or medial aspects of the retina receive light from the temporal or lateral aspects of the visual field.

The most striking neurological complication of von Economo encephalitis (encephalitis lethargica), a type of encephalitis that occurred in epidemic proportions along with viral influenza between 1917 and 1928, was which of the following? a. Blindness b. Hearing loss c. Paraplegia d. Parkinsonism e. Incontinence

The answer is d. (Ropper, pp 736-737.) At the onset of encephalitis lethargica, patients often develop transient fevers, lethargy, and headache. Disturbed eye movements are the most common sign of neurological disease during the acute illness. A variety of movement disorders, including chorea, athetosis, dystonia, and myoclonus, develop with the disease. About one in four affected persons die. The most common sequela of the disease is severe, unremitting parkinsonism with signs and symptoms similar to those exhibited with idiopathic parkinsonism. One rather unique feature is the occurrence of oculogyric crises, or episodes in which the eyes deviate to one side or upward, associated with other forms of dystonia and autonomic symptoms, sometimes occurring with great regularity.

A 72-year-old man is brought to the emergency room in a coma. He has a fever and was observed to have a generalized tonic-clonic seizure just prior to arriving in the emergency room. His family reports that he had lethargy and cough about 1 week prior to the acute deterioration. On the day of his seizure, he developed a headache and blurred vision. He had some vomiting early in the day and became more stuporous as the day progressed. There is no evidence of alcohol or drug use.

The answer is d. (Ropper, pp 667-678.) This man with fever, generalized seizure, lethargy, cough, headache, blurred vision, and progressive stupor probably has acute bacterial meningitis. Given his age of 72 and history of probable upper respiratory infection, a pneumococcal meningitis is highly probable. In bacterial meningitis, the CSF typically exhibits elevated protein content, no or few RBCs, an elevated opening pressure, milky or xanthochromic fluid, and a normal or slightly elevated gamma globulin content. If there are relatively few white cells and the CSF protein is not greatly elevated, the fluid may appear clear and colorless. The WBC count will be elevated, and the WBCs in the CSF will consist of both polymorpho-nuclear cells and lymphocytes. A very low CSF glucose content supports the diagnosis of bacterial meningitis. Tuberculous meningitis, however, produces an atypical pattern of CSF changes distinct from that caused by other bacterial pathogens and reminiscent of that caused by fungi.

A 50-year-old woman presents to the emergency room with lethargy, fever, and moderately low blood pressure. She has a fever workup, is started on IV fluids and antibiotics, and is then admitted to the hospital. A diagnosis of bacterial endocarditis is made. On day 2 of her admission, she has developed a right upper extremity drift and her speech has decreased fluency. A head CT reveals a rim-enhancing lesion in the left frontal lobe. Which of the following is the most common site for formation of this type of lesion? a. Putamen b. Thalamus c. Head of the caudate d. Gray-white junction e. Subthalamus

The answer is d. (Ropper, pp 683-686.) Brain abscesses usually start from a microscopic focus of infection at the junction of gray matter and white matter. As the infection develops, a cerebritis appears, and subsequently this focus of infection becomes necrotic and liquefies. Around the enlarging abscess, there is usually a large area of edema.

A 37-year-old woman is noted to have lymphadenopathy on routine physical examination. Following an extensive evaluation, she is diagnosed with sarcoid. She has been entirely normal neurologically. Which cranial nerve (CN) is most likely to be injured in this patient? a. II b. III c. V d. VII e. VIII

The answer is d. (Ropper, pp 690-692.) Facial paresis is the neurological injury most likely to develop with sarcoidosis. Almost half of patients with sarcoidosis and neurological disease have a neurological sign or symptom as the first obvious complication of the sarcoidosis. These patients report progressive weakness of one side of the face with no substantial loss of sensation over the paretic side. They may feel that there is decreased sensitivity to touch on the weak side, but this is more commonly from a loss of tone in the facial muscles than from an injury to the trigeminal nerve. Other CNs especially susceptible to injury in persons with sarcoidosis include II, III, IV, VI, and VIII.

A 55-year-old woman has progressive dementia over the past year. Within the past 3 months she has also developed dysarthria, myoclonus, intention tremor, and hyperreflexia. CSF VDRL is positive. This patient's symptoms are being caused by which of the following? a. A response to penicillin treatment b. An autoimmune reaction c. An acute meningoencephalitis d. A chronic meningoencephalitis e. A chronic rhombencephalitis

The answer is d. (Ropper, pp 692-697.) General paresis is a slowly evolving process that may require years to produce substantial disability. The early symptoms are a subtle dementia, characterized by memory loss and impaired reasoning, with later development of dysarthria, myoclonus, tremor, seizures, and upper motor neuron signs, leading to a bedridden state. Both the meninges and the parenchyma of the brain are involved in this chronic infection. The meninges are thickened and opaque, and a granular ependymitis characteristically develops. Degenerative changes occur throughout the cerebral parenchyma. Penicillin is the treatment of choice for this disease. The damage done to the brain is not mediated by autoimmune or adverse drug reaction mechanisms. This infection produces widespread injury to the brain rather than the restricted brainstem damage that typically occurs with infections that attack structures arising from the embryonic rhombencephalon. With a rhombencephalitis, the pons and medulla oblongata are the principal targets of this disease.

Following several days of low-grade fever and mild neck and head pain, a 10-year-old boy develops bilateral face drooping and difficulty fully closing his eyes. Serum is positive for Borrelia burgdorferi IgM. CSF polymerase chain reaction (PCR) is also positive for this organism's DNA. After B burgdorferi is introduced by the tick that carries it, the skin around the bite develops which of the following? a. An exfoliative dermatitis b. Purpura c. Localized edema d. Erythema chronicum migrans e. Vesicular lesions

The answer is d. (Ropper, pp 697-699.) B burgdorferi is the agent responsible for Lyme disease. It is a spirochete usually transmitted to humans through tick bites. Multiple organ systems are attacked by the spirochete; the nervous system is especially susceptible. Erythema chronicum migrans is an expanding reddish discoloration of the skin that spreads away from the site of the bite as an expanding ring of erythema. It usually evolves over 3-to-4 weeks. This ring of erythema clears spontaneously within about 1 month and is usually associated with some headache and neck stiffness. Some patients with Lyme disease fail to exhibit the rash.

A 35-year-old woman who has received a liver transplant develops meningeal signs and fever. Cerebrospinal fluid (CSF) testing with India ink stain reveals a fungal infection. Which of the following is the cause of this patient's fungal meningitis? a. Aspergillus b. Candida c. Mucor d. Cryptococcus e. Rhizopus

The answer is d. (Ropper, pp 700-701.) Cryptococcosis is usually acquired through the lungs and spreads to the CNS through the bloodstream. In the CNS, it may produce either a meningitis or a meningoencephalitis. The organism has a characteristic capsule, which simplifies its identification. Fungal infections most often occur in the CNS in persons with defects in their immune systems. These defects may be secondary to a viral infection, as with AIDS, or they may be a consequence of immunosuppressive drug exposure. Patients on immunosuppressive treatment after organ transplants and those with lymphoproliferative disorders, such as lymphocytic leukemia, were the most common victims of CNS fungal infections before the start of the AIDS epidemic. Aspergillus, Candida, Mucor, and Rhizopus can also cause CNS fungal infections, but rarely meningitis. Aspergillus tends to cause abscesses in immunocompromised individuals, and Mucor affects mostly diabetics.

A 73-year-old man with a history of hypertension has a 10-minute episode of left-sided weakness and slurred speech. On further questioning, he relates three brief episodes in the past month of sudden impairment of vision affecting the right eye. His examination now is normal. Which of the following is the most appropriate next diagnostic test? a. Creatine phosphokinase (CPK) b. Holter monitor c. Visual evoked responses d. Carotid artery Doppler ultrasound e. Conventional cerebral angiography

The answer is d. (Ropper, pp 778-787.) This patient is experiencing the classical symptoms of extracranial internal carotid artery disease, which include episodes of ipsilateral transient monocular blindness (amaurosis fugax) and contralateral transient ischemic attacks consisting of motor weakness. Patients with symptomatic extracranial carotid artery disease have a high likelihood of going on to develop strokes (approximately 26% over 2 years on medical therapy). The appropriate test to confirm the suspicion of carotid stenosis is a Doppler ultrasound of the carotid arteries. This test utilizes the fact that sound waves will bounce back from particles moving in the bloodstream— primarily RBCs—at a different frequency depending on the velocity and direction of the blood flow. A great deal of important information about the structure of the blood vessel can be obtained in this way. Although angiography can also provide this information, it is invasive, carries a risk of causing a stroke, and is more expensive. MRI and MRA would also be an appropriate next step in evaluation, but is not one of the answer choices.

A previously healthy 7-month-old infant is brought to the emergency room after having had three generalized convulsions. The infant has a stiff neck, is poorly responsive to the examiner, and has a rectal temperature of 38.9°C (102°F). The parents report that 1 day of diarrhea preceded this episode.

The answer is d. (Swaiman, pp 1573-1576.) Seizures associated with fever at this age are worrisome and must be aggressively investigated. The neck stiffness, fever, and recurrent seizures necessitate a spinal tap to allow examination of the CSF. The fluid should be checked for the opening pressure, Gram staining, protein and glucose content, cell count, bacterial and fungal cultures, acidfast bacillus (AFB) staining, and flagellated treponemal antigen (FTA-ABS). Antibiotic treatment should be started immediately if there is any indication of an infection. Haemophilus influenzae was commonly responsible for meningitis at this early age prior to the availability of vaccination against it, but infections with a variety of bacteria, including gram-negative bacteria, are also possible causes of the clinical scenario described.

A 65-year-old man was forced to retire from iron working because of a neurological condition which has progressed over the past several years. It is characterized by tremor, rigidity, and bradykinesia worse on the left side. The symptoms are somewhat alleviated by treatment with Ldopa/carbidopa. This patient's resting tremor is most likely to do which of the following upon falling asleep? a. It becomes more rapid. b. Its amplitude increases. c. It generalizes to limbs that were uninvolved when the patient was awake. d. It disappears. e. It transforms into choreiform movements.

The answer is d. (Watts, p 235.) Patients with Parkinson disease often have a characteristic pillrolling tremor of the hand while they are awake. The tremors associated with Parkinson disease are worse when the patient is at rest and not moving the affected limb. Paradoxically, this resting tremor ceases when relaxation progresses to sleep. In fact, most tremors and other types of movement disorders caused by disease of the caudate, putamen, and globus pallidus (ie, the basal ganglia) and of the substantia nigra remit during sleep. Choreiform movements are jumping or dance-like movements and occur with Wilson disease (hepatolenticular degeneration) and Huntington disease, a hereditary degenerative disease of the basal ganglia.

An 85-year-old man is being evaluated for gait difficulties. He says that he frequently trips walking up stairs or on uneven surfaces. On examination, it is found that joint proprioception is absent in his toes. People with impaired position sense will usually fall if they simultaneously stand with their feet together and do which of the following? a. Flex the neck b. Extend their arms in front of them c. Flex the knees d. Turn the head e. Close their eyes

The answer is e. (Ropper, p 113 .) Standing with the feet together and the eyes closed is the Romberg test. The person with poor position sense needs visual cues to remain standing. This test was introduced as a helpful way to check for deficits associated with tabes dorsalis, a form of neurosyphilis. In tabes dorsalis, the dorsal columns of the spinal cord are damaged. These dorsal or posterior columns carry the nerve fibers that transmit vibration and position sense to the brain. With the widespread use of penicillin, tabes dorsalis has become rare in industrialized nations, and impaired position sense is much more likely to be a consequence of diabetes mellitus, spondylosis, or alcoholism. In performing the test, it is important to remember that even normal people will tend to sway slightly with their eyes closed, and that those with loss of cerebellar function or vestibulopathy will also sway more with loss of visual cues.

The patient with impaired vitamin B12 absorption is likely to develop a positive Romberg test because of damage to which of the following? a. Cerebellar vermis b. Cerebellar hemispheres c. Spinal cord lateral columns d. Basal ganglia e. Spinal cord posterior columns

The answer is e. (Fauci, p 1499.) Both the lateral and posterior columns of the spinal cord are damaged with cobalamin deficiency, but a positive Romberg sign develops with impaired position sense, a sensory modality carried in the posterior columns of the cord. Because both sensory and motor functions are disturbed with cobalamin deficiency, the resulting condition is called combined systems disease. The microscopic changes in the posterior and lateral columns of the spinal cord in the patient with combined systems disease include demyelination, gliosis, and vacuolar degeneration. The regions of the spinal cord most severely damaged are the lower cervical and upper thoracic. The vacuolar changes observed arise in the myelin sheaths of very large nerve fibers. Although this starts as a predominantly demyelinating lesion, it evolves into axonal loss. Patients develop spasticity and weakness, as well as disturbed vibration and position sense. The clinical picture becomes more confused because a peripheral neuropathy may also develop with cobalamin deficiency. The peripheral neuropathy of the sort occurring with vitamin B12 deficiency would ordinarily produce hyporeflexia. The lateral column damage, which involves the corticospinal tracts, would ordinarily cause hyperreflexia. Because both peripheral nerves and corticospinal tracts are damaged with vitamin B12 deficiency, the effect on reflexes is difficult to predict and often changes over time. The patient will usually start with hyperreflexia and then develop either clonus or hyporeflexia.

A 56-year-old woman has been on dialysis for the past 10 years owing to chronic renal failure from cystic kidney disease. Which of the following is the most reliable treatment for the peripheral neuropathy associated with her condition? a. Thiamine supplements b. Clonazepam c. Phenytoin d. Minoxidil e. Renal transplant

The answer is e. (Fauci, pp 1779-1781.) B vitamins are generally replaced when patients receive dialysis. Thiamine is water-soluble and so is easily lost during dialysis, but even replacing thiamine is not nearly as effective in retarding or reversing the neuropathy of chronic renal failure as is renal transplantation. There are presumed to be neurotoxins in the blood of patients with uremia that are not removed by routine dialysis.

An 18-year-old woman tells her physician that she has throbbing right-sided headaches. They are most common just prior to her menstruation and are associated with nausea and photophobia. Examination at the time of the office visit is normal. A T1 sagittal image from her MRI is shown below. The location of the cerebellar tonsil in the MRI scan suggests which of the following? a. Arnold-Chiari type 1 malformation b. Arnold-Chiari type 2 malformation c. Giant cisterna magna d. Dandy-Walker syndrome e. Normal posterior fossa

The answer is e. (Osborn, pp I[1]:11.) Bone has low signal on this T1-weighted image of the head, and the cerebellar tonsil is sitting above the opening of the foramen magnum. The bone marrow in the occipital bone may be seen as high signal (white). With Arnold-Chiari malformations, the tonsil would be expected to sit below the foramen magnum. With Dandy-Walker syndrome or giant cisterna magna, the tonsil would be inapparent or at least sitting much more cephalad. The history provided by the patient is typical of common migraine.

A 64-year-old woman slips and falls on an icy sidewalk. She hits the side of her head on the curb. After a momentary loss of consciousness she recovers, but is in some pain. Fifteen minutes later her level of consciousness begins to fluctuate and she is brought to the emergency room comatose. Magnetic resonance imaging (MRI) of the patient's head within the first few hours of injury will most likely reveal which of the following? a. A normal brain b. Intracerebral hematoma c. Temporal lobe contusion d. Subarachnoid hemorrhage e. Epidural hematoma

The answer is e. (Ropper, p 858.) The history is typical for an epidural hematoma. Damage to the middle meningeal artery allows blood at arterial pressures to dissect in the potential space that exists between the dura mater and the periosteum of the skull. With MRI, the epidural hematoma should be evident soon after the injury and will certainly be evident by the time the patient is symptomatic.

A 28-year-old man presents to the emergency room with a severe headache. It is different than any that he has ever had before. It is in the right posterior region and is not throbbing. The headache started suddenly, about 5 hours ago, while he was watching television and eating pizza. He is now noticing some mild neck stiffness and blurry vision. Examination is significant for weakness of abduction of the right eye. Which of the following is the most definitive test for identifying intracranial aneurysms? a. MRI scanning b. CT scanning c. Single photon emission computed tomography (SPECT) d. Positron emission tomography (PET) e. Cerebral angiography

The answer is e. (Ropper, p 19.) Computed tomographic scanning is especially sensitive to intracerebral hemorrhage, but not to unruptured aneurysms unless they are more than 5 mm across. Even such relatively large aneurysms may not be revealed by CT scanning unless there is bleeding from the aneurysm or distortion of adjacent structures by the aneurysm. Microscopic aneurysms may be localizable on CT only because of the high signal left near the aneurysm by telltale blood. In most cases of aneurysmal bleeding, angiography is needed to characterize and localize the lesion. The resolution of PET, MRI, SPECT, and CT of intracranial aneurysms is insufficient to enable surgical correction of the lesion to proceed without demonstration of the aneurysm on angiography.

A 57-year-old woman is involved in a motor vehicle accident in which she strikes the windshield and is briefly unconscious. She makes a full recovery, except that 3 months later she notices that she cannot taste the food she is eating. This is most likely caused by which of the following? a. Medullary infarction b. Temporal lobe contusion c. Sphenoid sinus hemorrhage d. Phenytoin use to prevent seizures e. Avulsion of olfactory rootlets

The answer is e. (Ropper, p 219.) Anosmia is one of the more common long-term cranial nerve deficits after head injury, though it is present in only 6% in one series. It is often associated with ageusia (loss of taste). It can be very disabling and discouraging to patients. Approximately one-third of patients recover. It is caused by avulsion of olfactory nerve rootlets due to acceleration- deceleration injury at the cribriform plate. Damage may be unilateral or bilateral.

A 33-year-old woman has an acute onset of right orbital pain after a tennis match. The following morning, her 10-year-old son comments that her right eye looks funny. On examination, she has a mild right ptosis and anisocoria. The right pupil is 2 mm smaller than the left, but both react normally to direct light stimulation. Visual acuity, visual fields, and eye movements are normal. The site of injury is caused by interruption of fibers from which of the following structures? a. Optic tract b. Optic chiasm c. CN III d. T1 nerve root e. Superior cervical ganglion

The answer is e. (Ropper, p 271.) The presence of ptosis and miosis indicate oculosympathetic palsy, or Horner syndrome. This indicates injury to the sympathetic supply to the eye. This pathway begins in the hypothalamus, travels down through the lateral aspect of the brainstem, synapses in the intermediolateral cell column of the spinal cord, exits the spinal cord at the level of T1, and synapses again in the superior cervical ganglion. From there, postganglionic fibers travel along the surface of the common carotid and internal carotid arteries until branches leave along the ophthalmic artery to the eye. Fibers of the sympathetic nervous system, which are destined to serve the sudomotor function of the forehead, travel with the external carotid artery. Thus diseases affecting the internal carotid artery and the overlying sympathetic plexus do not produce anhidrosis, the third element of Horner syndrome. In this case, the occurrence of painful Horner syndrome acutely after vigorous activity is virtually diagnostic of carotid artery dissection. Dissections may occur more frequently in migraineurs. The preservation of visual fields and acuity excludes significant disease of the optic tract and chiasm, which also would not be expected to cause ptosis. Lesions of cranial nerve (CN) III do cause ptosis, but they would also be expected to cause ipsilateral mydriasis, or pupillary enlargement, not miosis. The degree of ptosis is usually much more severe in third nerve palsy than in Horner syndrome; this is because CN III supplies the levator palpebrae, the primary levator of the lid, whereas the sympathetics supply Müller muscle, which plays an accessory role. The sympathetic pathway does exit the spinal cord at T1, but injury at this location would not cause orbital pain, which is typical of carotid arterial dissection.

A 50-year-old right-handed man has presented to a neurologist because of gradually progressive hearing loss of the right ear. He denies worsening balance or ringing in his ears. There is no associated pain. A vibrating tuning fork is applied to the center of his forehead. The sound is louder in his left ear. This finding suggests which of the following? a. Bilateral sensorineural hearing loss. b. Bilateral conductive hearing loss. c. Right ear conductive hearing loss. d. Left ear sensorineural hearing loss. e. Right ear sensorineural hearing loss

The answer is e. (Ropper, p 281.) The vibrations from a tuning fork placed on top of the head are transmitted through the skull to both ears. Bone conduction of sound through the skull should be equal in both ears. With sensorineural hearing loss, the patient will hear the midline fork more loudly in the unaffected ear. Sensorineural hearing loss is the deafness that develops with injury to the receptor cells in the cochlea or to the cochlear division of the auditory nerve. In conductive hearing loss, the vibrations of the tuning fork are perceived as louder in the affected ear. With this type of hearing loss, the injury is in the system of membranes and ossicles designed to focus the sound on the cochlea. Impairment of the conductive system causes the vibrations of the tuning fork to be transmitted to the cochlea directly through the skull. Much like a person with cotton stuffed into the external auditory meati, the patient with the conductive hearing loss has impaired perception of sound coming from around him or her but an enhanced perception of his or her own voice. This type of tuning fork test is called the Weber test.

A 17-year-old right-handed boy has had infectious meningitis eight times over the past 3 years. He has otherwise been generally healthy and developed normally. Recurrent meningitis often develops in persons with which of the following? a. Otitis media b. Epilepsy c. Multiple sclerosis d. Whipple disease e. CSF leaks

The answer is e. (Ropper, p 674.) A CSF leak indicates a communication between the subarachnoid space and the surface of the body. This leak most often occurs through the nose as rhinorrhea or through the ear as otorrhea. The CSF may be distinguished from other fluid discharged from the nose or ear by its relatively obvious glucose content. The most common basis for a CSF leak is head trauma.

A 40-year-old man died from complications related to AIDS. Prior to his death, there had been a steady cognitive decline. Both HIV and cytomegalovirus infections in the brain characteristically produce which of the following? a. Senile plaques b. Intraneuronal amyloid c. Intranuclear inclusions d. Intracytoplasmic inclusions e. Microglial nodules

The answer is e. (Ropper, p 729.) The microglial nodules occurring with HIV are associated with syncytial cells in the brain and spinal cord, a cell type not typically seen with cytomegalovirus (CMV) encephalitis. Cytomegalovirus is a common CNS opportunistic agent in patients with AIDS. With HIV infection, the microglial nodules are distributed around blood vessels throughout the brain. With CMV, the nodules are more characteristically subpial and subependymal.

A 37-year-old woman develops cholecystitis and requires cholecystectomy. Her family advises the physicians involved that she has a long history of alcoholism and benzodiazepine use, including diazepam, lorazepam, and clonazepam. Approximately 7 days after the surgery, the patient becomes increasingly agitated, delusional, and suspicious. Routine investigations reveal no evidence of focal or systemic infection. Hepatic, renal, and hematologic parameters are largely normal. Within 24 hours of these cognitive and affective changes, the patient has a generalized tonic-clonic seizure. MRI and computed tomography (CT) studies of the brain are normal, and her CSF is unremarkable. In consideration of the abuse history provided by the family, medication orders prior to the surgery should have included which of the following? a. Haloperidol b. Chlorpromazine c. Trihexyphenidyl d. Prochlorperazine e. Thiamine

The answer is e. (Ropper, pp 1109-1114 .) This woman was at risk for Wernicke encephalopathy. She should have received supplemental thiamine for at least 3 days, even though this would not have prevented the cognitive deterioration that she exhibited. There was no indication for using a neuroleptic (eg, haloperidol, chlorpromazine, or prochlorperazine), even though her alcohol and benzodiazepine use placed her at risk for developing a withdrawal psychosis. The anticholinergic trihexyphenidyl would not be appropriate as either a neuroleptic or an antiepileptic.

A 42-year-old woman is being treated with methotrexate for Wegener granulomatosis. Methotrexate may be associated with the syndrome of reversible posterior leukencephalopathy. She is also at risk for megaloblastic anemia because methotrexate disturbs the metabolism of which of the following? a. Cobalamin b. Iron c. Copper d. Pyridoxine e. Folate

The answer is e. (Ropper, pp 1172-1173 .) Methotrexate inhibits dihydrofolate reductase, thereby interfering with the metabolism of folate. As is the case with cobalamin deficiency, this results in a megaloblastic anemia. Defective DNA synthesis underlies the marrow disturbance that is seen with both folate and cobalamin deficiencies.

An 18-year-old man notices tingling about his ankles 2 weeks after an upper respiratory tract infection. Within 2 days, he has weakness in dorsiflexion of both feet, and within 1 week he develops problems with walking. He has no loss of bladder or bowel control. His weakness progresses rapidly over the ensuing week and necessitates his being placed on a ventilator to support his breathing. He is quadriplegic, but retains control of his eye movements. CSF studies reveal a protein content of greater than 1 g/dL with a normal white cell count. There are no red blood cells in the CSF.

The answer is e. (Ropper, pp 1261-1270.) The loss of strength associated with Guillain-Barré syndrome usually reaches a nadir within 2 weeks of the onset of symptoms. Sensation is usually preserved except for paresthesias in the feet or lower legs. Weakness is usually symmetric and often follows an ascending pattern of involvement. Tendon reflexes in the weak limbs are usually hypoactive or absent. Bladder and bowel control remain intact, but the patient usually exhibits some autonomic dysfunction, such as tachycardia and excessive sweating, which is rarely life-threatening. Eye muscles may be affected with Guillain-Barré syndrome, particularly in the condition called the Miller-Fisher variant. Before artificial ventilators were available, these patients often died from respiratory complications. The CSF with this disease typically reveals an elevated protein content with a relatively normal or only slightly abnormal white cell count and an invariably normal glucose content. This helps to distinguish it from poliomyelitis, a cause of paralysis that produces an alteration in both the protein and WBC content of the CSF consistent with a viral meningitis.

A 55-year-old woman is being examined because of difficulty walking. The clinician notices the presence of fine twitching movements beneath the surface of the tongue and wasting of one side of the tongue. This finding suggests which of the following? a. Pseudobulbar affect b. Aberrant reinnervation of muscles from CN X c. Aberrant reinnervation of muscles from CN XII d. Denervation of muscles from CN X e. Denervation of muscles from CN XII

The answer is e. (Ropper, pp 1336-1337.) The hypoglossal nerve provides motor innervation to the tongue. The fine movements noted under the surface of the tongue with injury to the hypoglossal nerve are called fasciculations and are an indication of denervation. They are presumed to occur through hypersensitivity to acetylcholine acting at the denervated neuromuscular junction. Atrophy and fasciculations are likely to occur together and are highly suggestive of denervation of the tongue. This is most often seen with brainstem disease, such as stroke or bulbar amyotrophic lateral sclerosis (ALS), or with transection of the hypoglossal nerve.

A 43-year-old woman describes lancinating pains radiating into the right side of her jaw. This discomfort has been present for more than 3 years and has started occurring more than once a week. The pain is paroxysmal and routinely triggered by cold stimuli, such as ice cream and cold drinks. She has sought relief with multiple dental procedures and has already had two teeth extracted. Multiple neuroimaging studies reveal no structural lesions in her head. Assuming there are no contraindications to the treatment, a reasonable next step would be to prescribe which of the following? a. Clonazepam, 1 mg orally three times daily b. Diazepam, 5 mg orally two times daily c. Divalproex sodium, 250 mg orally three times daily d. Indomethacin, 10 mg orally three times daily e. Carbamazepine, 100 mg orally three times daily

The answer is e. (Ropper, pp 182-183.) This woman probably has trigeminal neuralgia (tic douloureux). The treatment options for this facial pain disorder include carbamazepine. Although carbamazepine is a potent antiepileptic medication, other antiepileptic medications, such as phenobarbital and divalproex sodium, are usually ineffective in blunting the pain. Phenytoin and gabapentin are other antiepileptics useful in the management of trigeminal neuralgia. Analgesics and antiinflammatory drugs, such as indomethacin, are notably ineffective in managing this disorder.

A 39-year-old left-handed woman is being treated with carbamazepine for lancinating pain in her left face. The pain is paroxysmal, usually occurring without apparent reason, but seems sometimes to be brought on by a cold breeze. Both trigeminal neuralgia and atypical facial pain involve pain that may be which of the following? a. Lancinating b. Paroxysmal c. Associated with anesthetic patches d. Abolished with resection of the gasserian ganglion e. Unilateral

The answer is e. (Ropper, pp 185-186.) Unlike patients with trigeminal neuralgia, who describe paroxysmal, lancinating pains, patients with atypical facial pain usually feel a constant, deep pain. Although atypical facial pain is often bilateral, it may be unilateral and fairly limited in its distribution. The cheek, nose, or zygomatic regions are often affected by this idiopathic pain syndrome. The pain is often sensitive to antidepressant medication, a characteristic that has led some to suggest that the syndrome is invariably caused by depression. Progressive loss of sensation in the distribution of the fifth cranial nerve (CN) should prompt a careful search for an underlying malignancy invading the nerve either intracranially or in the face.

A 34-year-old woman is having her medications tapered in the epilepsy-monitoring unit. She has a convulsive seizure that does not stop after 5 minutes, even after she receives a lorazepam injection. A second intravenous drug is given. Infusing which of the following antiepileptic drugs at more than 50 mg/minute in an adult may evoke a cardiac arrhythmia? a. Carbamazepine b. Diazepam c. Phenobarbital d. Clonazepam e. Phenytoin

The answer is e. (Ropper, pp 304-335.) Rapid infusion of phenytoin may produce a cardiac arrhythmia or hypotension. Phenytoin should not be administered at rates greater than 50 mg/minute in adults or 1 mg/(kg-min) in children to reduce the chances of this reaction occurring. Thus it usually requires approximately 20 minutes to administer a 1000-to 1500-mg standard loading dose of phenytoin in an emergent setting such as status epilepticus. Fosphenytoin, a water-soluble prodrug of phenytoin, has the advantage of causing fewer infusion site reactions. It can be given at doses of up to 150 mg/minute in an adult, with risks of cardiac dysrhythmia similar to those of phenytoin. Another advantage of fosphenytoin is that it can be administered intramuscularly when intravenous access is problematic. Carbamazepine is not administered intravenously at all. Rapid infusion of phenobarbital may produce hypotension or respiratory arrest, but is much less likely to depress cardiac activity. Diazepam and clonazepam are safer than phenobarbital, but rapid infusion of excessively high doses may depress blood pressure and other autonomic functions.

A 7-year-old boy has recurrent staring episodes while at school. His school performance is poor. The episodes never last more than 30 seconds, and afterward he immediately resumes normal attention. There are no lip-smacking movements or other automatisms. He never falls down during the episodes. If he is walking or eating during the episode, he merely stops. He is unaware of this behavior.

The answer is e. (Ropper, pp 307-308.) This boy is probably having generalized absence seizures. The EEG will probably show the typical 3-Hz spike-and-wave pattern characteristic of this type of seizure disorder. Even if this patient does not have the typical pattern, the character of his episodes suggests epilepsy (ie, a tendency to have recurrent seizures), and the EEG should help characterize the type of epilepsy. Because a structural lesion could cause seizures, although not typically generalized absence seizures, an MRI or CT of the head should be performed to establish that no correctable lesion is present.

An 82-year-old man has had a slow, stepwise cognitive deterioration. A brain MRI is consistent with the diagnosis of dementia caused by multiple cerebral infarcts. Naming is impaired. Comprehension, repetition, and fluency are relatively maintained.

The answer is e. (Ropper, pp 466-470.) Anomic aphasia consists of an isolated word-finding deficit. It is the least localizable of the major aphasias. It is common in patients with diffuse brain dysfunction.

A 56-year-old right-handed woman who had breast cancer 1 year ago began having neurological problems about 1 week ago. She began experiencing nausea, vomiting, and numbness in the right hand and foot. Today she is experiencing crescendo pain in the left retroorbital area. Her headache is throbbing and positional, particularly when she tries to bend forward. The headache was intense in the morning, and at times it woke her up last night. On examination, the only deficits are loss of double simultaneous tactile stimulation and right lower facial droop when smiling. Which of the following is the most appropriate next action? a. Administer intravenous prochlorperazine b. Give the patient a prescription for zolmitriptan c. Make a follow-up appointment for next month d. Order an electroencephalogram (EEG) to rule out seizures e. Get a brain MRI

The answer is e. (Ropper, pp 612-655.) The headache is typical of that caused by intracranial hypertension. Additionally, the patient has focal neurological symptoms and signs. This creates particular concern about a brain tumor or hemorrhage, and the patient should be evaluated as soon as possible. An appointment next month is too late. Intravenous prochlorperazine is a good treatment for status migrainosus; however, this history is atypical for such a diagnosis, and more serious problems should be ruled out first in the emergency room. Zolmitriptan is a treatment for migraines. This history is not typical for migraine, and zolmitriptan is also relatively contraindicated in patients with complex migraine. This history is very atypical for seizures, and an EEG is not likely to provide useful information in this case.

A 55-year-old woman presents with mild unsteadiness, tinnitus, and hearing loss.

The answer is e. (Ropper, pp 644-646.) Schwannomas usually develop on the vestibular division of CN VIII and are pathologically derived from Schwann cells rather than nerve tissue and are therefore more appropriately called vestibular schwannomas than acoustic neuromas (the traditional name). Although this is not the division of the nerve that carries information from the cochlea, the cochlear division is crushed as the tumor expands. This type of tumor is especially likely with NF type 2, a hereditary disorder characterized by a variety of tumors in the skin and nervous system.

A 19-year-old man develops obvious personality changes over the course of 2 weeks. He becomes agitated with little provocation and abuses his wife both verbally and physically. His behavior is sufficiently atypical for it to prompt his relatives to seek psychiatric assistance for him. While being interviewed by a psychiatrist, he becomes unresponsive and develops generalized convulsions with opisthotonic posturing, tonic-clonic limb movements, and urinary incontinence. He is hospitalized for investigation of his seizure disorder. On initial examination, he is noted to have a low-grade fever and a mild left hemiparesis. His CSF opening pressure is 210 mm H2O. His CSF cultures yield no growth, and his electroencephalogram (EEG) reveals polyspike-and-wave discharges originating in the right temporal lobe. A CT of his brain reveals focal swelling of the right temporal lobe.

The answer is e. (Ropper, pp 719-721.) With herpes encephalitis in the person who is not immunodeficient, the first clinical signs of disease are likely to be psychiatric. Depression, irritability, and labile affect are especially common. The organic basis for the encephalopathy usually becomes self-evident when the affected person has a seizure. Because the temporal lobe is especially involved by herpes encephalitis, the initial seizure is likely to be complex partial, but seizures often become more generalized. EEG will usually reveal the focal character of the cerebral damage. Intracranial pressure is usually increased with a fulminant infection. Temporal lobe swelling may be severe enough to produce lethal herniation.

A 35-year-old woman is bitten by a small doglike wild animal while camping. The animal immediately runs away. Her skin is barely broken, and, besides feeling a little frightened, she says that she is fine. Despite this, her friend convinces her to be evaluated in the nearest emergency room. Which of the following viruses that typically invade the CNS by extending centripetally (ie, inward away from the periphery) along peripheral nerves is the woman most at risk for? a. Mumps b. Measles c. Varicella zoster d. Polio e. Rabies

The answer is e. (Ropper, pp 721-723.) Mumps, measles, and varicella zoster infection appear to be acquired primarily by way of the respiratory tract. The poliovirus is an enterovirus, which means it enters primarily through the gastrointestinal tract. Rabies is transmitted by animal bites and reaches the CNS by migration in neuronal processes, presumably as it is swept along by retrograde axoplasmic flow. This is believed to be an unusual method of viral spread to the CNS. Most viruses that do produce CNS disease are carried to the CNS in the bloodstream rather than along neuronal processes with infected axoplasm.

A 9-year-old girl is playing in a wooded area of her backyard. She notices a furry animal in the brush. As it does not seem to fear her, she approaches to pet it. As soon as she touches the creature, it bites her and runs away. Her parents bring her to the emergency room for evaluation. The emergency room physician is extremely concerned that the patient may have been exposed to a deadly virus and orders immediate injections of immunoglobulin. From the brain, this pathogen establishes itself for transmission to another host by spreading to which of the following? a. Intestines b. Nasopharynx c. Lungs d. Bladder e. Salivary glands

The answer is e. (Ropper, pp 721-723.) Rabies is usually spread through the saliva of an infected animal. Introduction of saliva into a bite wound allows the virus to inoculate muscles or subcutaneous tissues. After introduction of the virus, the incubation period until fulminant infection appears extends from a few days to over 1 year, but usually ranges from 1-to-2 months. Bites of the head and face carry the greatest risk of causing fatal disease. Early after exposure, the patient will often complain of pain or paresthesias at the site of the animal bite. Animals transmitting the virus include dogs, bats, skunks, foxes, and raccoons. Dehydration as a complication of rabies is no longer likely because intravenous fluids can be given to completely replace what the hydrophobic patient cannot consume by mouth. Other complications of rabies include a paralytic form of the disease that progresses to quadriplegia (dumb rabies) in 20% of patients. With the classic form of the disease, the patient will also exhibit intermittent hyperactivity.

A 22-year-old woman is brought to the hospital in a coma. She has had changes in her behavior characterized by excessive suspiciousness and facetiousness over the month prior to her hospitalization. One week prior to her hospitalization, she had visual and auditory hallucinations. Drug testing reveals no apparent illicit drug use. On the day of admission, she had a generalized seizure and lapsed into a coma. MRI shows unilateral changes in the temporal lobe.

The answer is e. (Ropper, pp 724-725.) This young woman with progressive behavioral disturbances, hallucinations, seizures, and obtundation probably has a herpes simplex type 1 encephalitis. The CSF with herpes simplex encephalitis often has some RBCs in addition to the primarily mononuclear increase in WBCs. The CSF protein is elevated, but the glucose content is relatively normal with this viral infection. As the CSF protein increases, the percentage that is gamma globulin generally increases. This is not an indication that the problem is an infection, but this increase in total protein and gamma globulin component does occur with infections. The opening pressure may be markedly elevated, but the fluid may remain clear or be only slightly cloudy if the WBC count does not increase substantially.

A 51-year-old woman with an 8-month history of neurological decline dies after a severe bout of aspiration pneumonia. Autopsy of her brain reveals extensive loss of granule cells in the cerebellum and other changes most obvious in the cerebellar cortex. Fine vacuoles give the brain a spongiform appearance. No senile plaques are evident. The patient could have acquired this progressive disease through which of the following means? a. Sexual intercourse b. A blood transfusion c. Consumption of raw fish d. An upper respiratory infection e. Growth hormone treatment

The answer is e. (Ropper, pp 737-741.) The most likely cause of spongiform encephalopathy in this middle-aged woman is Creutzfeldt-Jakob disease. This is classified as a prion disease and can be transmitted via infected nervous system tissue, including dura mater grafts, and occasionally via growth hormone preparations acquired from cadaver pituitary glands. A similar disease (kuru) occurs in Fore Islanders of New Guinea and is presumed to spread through the ritual handling or eating of human brain tissue. As of 1999, 113 cases due to growth hormone preparations had been reported, with incubation periods ranging from 5-to-30 years.

A 52-year-old woman develops progressive dementia, tremors, gait ataxia, and myoclonic jerks over the course of 6 months. Her speech is slow and slurred, and hand movements are clumsy. No members of her immediate family have a history of degenerative neurological disease. MRI of the head reveals a subtle increase in T2 signal in the basal ganglia bilaterally. EEG reveals disorganized background activity with periodic sharp-wave discharges that occur repetitively at 1-second intervals and extend over both sides of the head. Arteriogram reveals no vascular abnormalities. The clinical picture is most consistent with which of the following? a. Multi-infarct dementia b. Tabes dorsalis c. Friedreich disease (Friedreich ataxia) d. Subarachnoid hemorrhage e. Spongiform encephalopathy

The answer is e. (Ropper, pp 737-741.) This patient has a subacute to chronic progressive disease characterized by a combination of dementia, tremor, ataxia, and myoclonus. The EEG and MRI findings are typical of a spongiform encephalopathy. Multi-infarct dementia and subarachnoid hemorrhage would be expected to produce at least one very discrete event, and the imaging studies would be expected to show evidence of infarcts or other vascular abnormalities. Friedreich disease may produce some dementia, but it is not a prominent part of the clinical deterioration. This patient is also much older than would be consistent with Friedreich disease.

A 39-year-old woman has diplopia several times a day for 6 weeks. She consults a physician when the double vision becomes unremitting and also mentions a dull pain behind her right eye. When a red glass is placed over her right eye and she is asked to look at a flashlight off to her left, she reports seeing a white light and a red light. The red light appears to her to be more to the left than the white light. Her right pupil is more dilated than her left pupil and responds less briskly to a bright light directed at it than does the left pupil. Before any further investigations can be performed, the woman develops the worst headache of her life and becomes stuporous. Her physician discovers that she has marked neck stiffness and photophobia. The physician performs a transfemoral angiogram. This radiologic study is expected to reveal that the woman has which of the following? a. An arteriovenous malformation b. An occipital astrocytoma c. A sphenoidal meningioma d. A pituitary adenoma e. A saccular aneurysm

The answer is e. (Ropper, pp 808-816.) The clinical picture suggests that a saccular aneurysm has become symptomatic by compressing structures about the base of the brain and subsequently leaking. The patient has papillary dilation of the right eye and weakness of extraocular muscles, suggesting compression of CN III by the aneurysm. Aneurysms enlarge with age and usually do not bleed until they are several millimeters across. Persons with intracerebral or subarachnoid hemorrhages before the age of 40 are more likely to have their hemorrhages because of arteriovenous malformations than because of aneurysms. Aneurysms occur with equal frequency in men and women below the age of 40; however, in their 40s and 50s, women are more susceptible to symptomatic aneurysms. This is especially true of aneurysms that develop on the internal carotid on that segment of the artery that lies within the cavernous sinus. An angiogram is useful in establishing the site and character of the aneurysm. A CT scan would be more likely to reveal subarachnoid, intraventricular, or intraparenchymal blood, but it would reveal the structure of an aneurysm only if it were several (>5) millimeters across. An MRI or magnetic resonance angiography (MRA) will also reveal relatively large aneurysms. This patient had a transfemoral angiogram, a technique that involves the introduction of a catheter into the femoral artery; the catheter is threaded retrograde in the aorta and up into the carotid or other arteries of interest.

A 42-year-old woman is involved in a head-on collision with a lamp-post at 50 mph. Her head hits the windshield. She is highly likely to have an intracranial hemorrhage in which one of the following structures? a. Occipital lobe b. Thalamus c. Putamen d. Parietal lobe e. Temporal lobe

The answer is e. (Ropper, pp 858-862.) The temporal lobes and inferior frontal lobes are frequently involved in traumatic brain injuries. The continued forward movement of the brain within the bony cranial vault, which has suddenly decelerated at impact, leads to these anterior brain structures striking the inside of the skull with great force, creating contusions in these areas. The rough surfaces of the cribriform plate and the middle cranial fossa also lead to injury in these locations. These injuries are referred to as the coup injuries because they reflect the direct blow to the brain. So-called contrecoup injury may also occur at the diametrically opposed region of the brain (generally, the occipital lobes) when there is rebound movement into the overlying skull there. Damage to the temporal lobe may produce symptoms and signs by virtue of compression of adjacent brain structures. As a hematoma expands, uncal herniation may crush the brainstem. Less progressive injuries may disturb memory or even language comprehension. Wernicke area, which is important in language comprehension, is sufficiently posterior on the temporal lobe to escape injury in most cases of frontal head trauma.

An 18-year-old girl riding on the back of her boyfriend's motorcycle without a helmet is brought in with a left frontal skull fracture and cortical contusion. Her Glasgow Coma Scale (GCS) is 10. She is admitted to the intensive care unit. She has had no seizures. Which of the following is true regarding anticonvulsant therapy in this case? a. It is contraindicated owing to risk of rash. b. It is best achieved using phenobarbital. c. It is likely to cause increased cerebral edema. d. It is indicated to reduce the incidence of late posttraumatic epilepsy. e. It is indicated to reduce the incidence of early posttraumatic seizures.

The answer is e. (Ropper, pp 864-865.) There is evidence that prophylactic phenytoin reduces the incidence of seizures after head injury. Because early posttraumatic seizures may lead to increased morbidity and prolonged hospital stays, it is reasonable in some situations to treat patients prophylactically. There is no evidence that prophylactic treatment reduces the long-term (months-toyears later) risk of developing posttraumatic epilepsy.

A patient with bilateral posterior fossa masses has café au lait spots and reports a family history of bilateral hearing loss at a relatively young age. A gene abnormality should be suspected on which chromosome? a. 5 b. 13 c. 17 d. 21 e. 22

The answer is e. (Ropper, pp 983-984.) Meningiomas and bilateral acoustic neuromas occur with increased frequency in type 2 neurofibromatosis, a dominantly inherited disorder arising with a deletion on the long arm of chromosome 22. Women with breast cancer and other gynecologic cancers are also at increased risk of developing meningiomas, perhaps because of sex steroid receptors on these tumors that enhance their growth when gynecologic disturbances occur. Estrogen or progesterone antagonists may be useful in the management of these tumors, but tamoxifen, an estrogen inhibitor, paradoxically stimulates the growth of meningioma cells. Chromosome 17 is associated with NF1. People with NF1 ("peripheral" NF) are commonly affected by cutaneous manifestations, but would not be expected to have a family history of bilateral hearing loss at a young age.

A 7-year-old girl has been diagnosed with a brain tumor. The parents are both well educated with a scientific background and have many questions. During the course of the discussion, you might tell them that most brain tumors in children are which of the following? a. Metastatic lesions from outside the central nervous system (CNS) b. Oligodendrogliomas c. Glioblastomas multiforme d. Meningiomas e. Infratentorial

The answer is e. (Swaiman, p 1687.) The posterior fossa is the usual location for brain tumors in children. Medulloblastomas, ependymomas, and cerebellar (or brainstem) gliomas account for most of the tumors that occur before puberty. Other common tumors developing intracranially in children include optic gliomas and metastatic leukemias.

A 9-year-old developmentally delayed girl has precocious puberty and poorly controlled seizures. Her seizures are typically preceded by episodes of uncontrollable laughter. Which of the following mass lesions might explain her symptoms? a. Craniopharyngioma b. Choroid plexus papilloma c. Giant aneurysm d. Metastatic carcinoma e. Hypothalamic hamartoma

The answer is e. (Swaiman, pp 1701, 2115-2116.) Hypothalamic hamartomas are nonneoplastic malformations involving neurons and glia in the region of the hypothalamus. They may be discovered incidentally, either on imaging performed for other reasons or at autopsy, or they may cause symptoms referable to the hypothalamus. Most often, the latter involves neuroendocrine functions, causing precocious puberty or acromegaly caused by overproduction of growth hormone-releasing hormone. Patients may also experience paroxysms of laughter, known as gelastic seizures. They may be cured surgically. Craniopharyngiomas are epithelial neoplasms arising in the sellar and third ventricular regions. They may cause hypopituitarism and visual field disturbances. Choroid plexus papillomas usually develop intraventricularly and do not extend down into the sella turcica. These tumors affect both children and adults, but they are rare. They are benign if they are surgically accessible and are extirpated early in their evolution. Giant aneurysms occur in many locations, but typically do not cause gelastic seizures or precocious puberty. Metastatic carcinoma generally occurs in older patients and would not be expected to cause these symptoms.

A 19-year-old woman describes recurrent memory problems. Her fiancé reports that she seems to be inattentive for minutes at a time several times a week. She never injures herself during these episodes, but she cannot recall what happened, and, on one occasion, she became lost while walking home. An ambulatory EEG demonstrates evolving spike activity originating in the left temporal lobe during one of the episodes. The EEG pattern does not generalize. CT and MRI scanning of the brain reveal no structural abnormalities. Conversations with the woman's parents reveal that she had febrile seizures when she was 3 years old, which abated with antipyretic treatment alone.

The answer is f. (Ropper, p 332.) This young woman is having complex partial seizures without secondary generalization. She has episodic altered consciousness associated with a temporal lobe seizure focus and antedated by febrile seizures. Levetiracetam is the best choice because of its relatively good efficacy and adverse effect profile. Felbamate, phenobarbital, primidone, and divalproex sodium may also be effective at controlling the seizures, but all have side effect profiles making them poor first choices in this case.

A 50-year-old woman is found wandering in the street and is brought to the emergency room by the police. She is disoriented to time, place, and person, but has no evidence of head trauma. She staggers when she tries to walk, but she has no detectable alcohol in her blood. Eye movements are abnormal with paresis of conjugate gaze, and horizontal nystagmus is apparent. Relatives are contacted, and they report that this woman has a long history of alcohol abuse.

The answer is f. (Ropper, pp 1109-1114 .) An apparently acute deterioration in cognitive function in an alcoholic may stem from any one of several causes. Bleeding from esophageal varices may have produced a profound anemia. Inapparent head trauma may have produced a subarachnoid hemorrhage or subdural hematomas. If the patient's problem is caused by a nutritional deficiency, it is most likely thiamine deficiency associated with alcoholism. That this patient has no alcohol in her blood at the time of the deterioration is irrelevant. The triad of dementia, gait difficulty, and oculomotor paresis is characteristic of Wernicke encephalopathy, the rapidly progressive and potentially lethal form of thiamine deficiency. Peripheral neuropathy commonly develops with thiamine deficiency, but it is not a component of the encephalopathy caused by thiamine deficiency.

A 26-year-old woman with a 7-year history of epilepsy develops a generalized convulsion while shopping. She is taken to an emergency room, but no one accompanying her is aware of a previous history of epilepsy. Because she has a protracted postictal period, numerous investigations are performed over the course of the next hour. CT scan is completely normal, but her arterial blood gases reveal a mild acidosis.

The answer is f. (Ropper, pp 13-19.) This CSF profile is essentially normal. With idiopathic seizures, the CSF should be normal. Seizure activity does not ordinarily drive up the CSF protein content or significantly change the cellular content of the fluid. Occasionally, there is a mild pleocytosis of up to 80 cells/μL, which peaks 1 day postictally. The acidosis that is observed in this patient is inconsequential and is routinely found during the early postictal period after a generalized tonic-clonic seizure.

A 70-year-old right-handed woman with a history of polio describes 1 month of increasing difficulty rising from a chair and walking. She also has trouble combing her hair and cooking, and there is mild swallowing trouble, but only with solids. Her legs and upper arms are painful and mildly swollen. Periungual telangiectasias are seen. Erythrocyte sedimentation rate (ESR) is 61

The answer is f. (Ropper, pp 1355-1361.) This patient probably has a myopathy. The elevated ESR and periungual telangiectasias suggest an inflammatory myopathy, and specifically dermatomyositis. Creatine phosphokinase would help to confirm that this is a primary muscle disease. Additional tests that will be of use are electromyography (EMG) and muscle biopsy. EMG may show brief, low-voltage action potentials, fibrillation potentials, positive sharp waves, polyphasic units, and some evidence of denervation. Muscle biopsy may show changes characteristic of the particular inflammatory myopathies. In polymyositis, extensive necrosis of muscle fiber segments is seen with macrophage and lymphocyte infiltration. In dermatomyositis, the picture is quite different: There is perifascicular muscle fiber atrophy, and the inflammatory infiltrate occurs in the perimysial connective tissue rather than throughout the muscle fibers themselves. In addition, electron microscopy shows characteristic tubular aggregates. In the rarer inclusion body myositis, the appearance is similar to that of polymyositis except that rimmed vacuoles are also seen.

A 76-year-old man develops a dull left-sided head pain with some radiation of the discomfort to the right side of the head. He has no nausea or vomiting with the pain, but has lost 10 lb over the previous 2 months. His erythrocyte sedimentation rate is 102 mm/h, and he is mildly anemic. An extensive investigation for malignancy reveals no signs of lymphoma, carcinoma, or leukemia.

The answer is f. (Ropper, pp 178-179.) The greatest risk from temporal arteritis is loss of vision in association with the headache. The erythrocyte sedimentation rate is usually dramatically elevated, and the abolition of symptoms with corticosteroid therapy is equally dramatic. Temporal arteritis is largely nonexistent in persons less than 50 years of age and rare in those less than 60. Many patients exhibit persistent fevers and progressive weight loss. The temporal arteries are likely to be pulseless or at least thickened. Biopsy of the artery often reveals a giant cell arteritis.

A 37-year-old man develops involuntary twitching movements in his left thumb. Within 30 seconds, he notices that the twitching has spread to his entire left hand and that involuntary movements have developed in his left forearm and the left side of his face. He cannot recall what happened subsequently, but his wife reports that he fell down and the entire left side of his body appeared to be twitching. He appeared to be unresponsive for about 3 minutes and confused for another 15 minutes. During the episode, he bit his tongue and wet his pants.

The answer is f. (Ropper, pp 309-310.) With a Jacksonian march, or sequential seizure, the patient develops focal seizure activity that is primarily motor and spreads. This type of seizure often secondarily generalizes, at which point the patient loses consciousness and may have a generalized tonic-clonic seizure. The hand is a common site for the start of a Jacksonian march. The face may be involved early because the thumb and the mouth are situated near each other on the motor strip of the cerebral cortex.

A 62-year-old man with a history of myocardial infarction awakens with a dense right-sided hemiplegia. His eyes are tonically deviated to the left, and he does not respond to threat on the right side of his visual field. He appears to be alert and responds to pain on the left side of his body. His speech is unintelligible and nonfluent, and he follows no instructions. Efforts to get him to repeat simple phrases consistently fail.

The answer is f. (Ropper, pp 466-470.) Given the patient's history of cardiovascular disease, one must suspect that this man has sustained a stroke of the left cerebral hemisphere. Either the left internal carotid artery or the left middle cerebral artery is probably occluded. The area of infarction would be expected to include the frontal, temporal, and parietal lobe cortices. The tonic gaze deviation indicates damage to the frontal eye field on the left, which normally directs the eyes contralaterally. The right visual field loss occurs with damage to the optic radiation in the left hemisphere. Such large left hemisphere lesions are associated with a global aphasia, which is characterized by impaired comprehension, repetition, and fluency.

Over the course of 6 months, a 50-year-old immigrant from Eastern Europe develops problems with bladder control, an unsteady gait, and pain in his legs. On examination, it is determined that he has absent deep tendon reflexes in his legs, markedly impaired vibration sense in his feet, and a positive Romberg sign. Despite his complaint of unsteady gait, he has no problems with rapid alternating movement of the feet, and no tremors are evident. He has normal leg strength. The pain in his legs is sharp, stabbing, and paroxysmal. His serum glucose and glycohemoglobin levels are normal.

The answer is f. (Ropper, pp 695-696.) Tabes dorsalis is caused by Treponema pallidum, the agent responsible for all types of neurosyphilis, but it is a disease entity distinct from general paresis, the form of neurosyphilis in which personality changes and dementia do occur. With tabes dorsalis, the patient develops a leptomeningitis. The posterior columns of the spinal cord and the dorsal root ganglia are hit especially hard by degenerative changes associated with this form of neurosyphilis. Tabes dorsalis is a form of neurosyphilis that usually becomes symptomatic decades after the initial treponemal infection. The gait ataxia and positive Romberg sign in this patient are manifestations of absent position sense. Bladder and bowel control may be profoundly disturbed, presumably on the basis of dorsal spinal root disease. The bladder is usually hypotonic (flaccid), and megacolon may develop. Patients with tabes dorsalis routinely exhibit abnormal (Argyll Robertson) pupils and optic atrophy. The glucose and glycohemoglobin should be checked to eliminate the more common cause of impaired position sense in the United States, diabetes mellitus. That this patient is from a part of the world that has relatively poor health care is relevant because this form of syphilis is rarely seen in persons who have spent most of their lives in countries with easy access to antibiotics.

A 32-year-old South African woman develops irritability, sleeplessness, and fatigue. Her family believes that she is depressed, but neurological assessment establishes prominent short- and longterm memory problems. She has anemia and an obvious dermatitis on her face. Her diet is strictly vegetarian and limited almost entirely to grains, such as corn

The answer is g. (Ropper, p 1109 .) Persons with limited diet devoid of animal fats and rich in corn are at risk for pellagra, a nutritional deficiency of nicotinic acid or its precursor, tryptophan. This disease typically affects the skin, digestive tract, CNS, and hematopoietic system. People with diets limited to corn (maize) are especially vulnerable because of the low levels of tryptophan and niacin in this grain.

A 29-year-old immigrant from El Salvador is brought to the emergency room after a generalized seizure. After awakening, he relates that he has had two or three episodes of unexplained loss of consciousness in the past 2 years. He has otherwise been healthy. He served in the Salvadoran military for 3 years. His examination is normal. CT scan with contrast reveals two small hyperintense foci in the right frontal lobe, as well as a 1-cm cystic lesion with a nodular focus within it in the left frontal region. The cyst wall of the latter lesion enhances with contrast. The two right frontal lesions do not enhance.

The answer is g. (Ropper, p 706.) Cysticercosis is produced by the larval form (cysticercus) of the pork tapeworm, Taenia solium. This is the most common neurological infection throughout the world, occurring most commonly in South America, Southeast Asia, and Africa. It is transmitted by fecal-oral contact; tapeworm eggs hatch in the human GI tract, invade the bowel mucosa, and migrate throughout the body, particularly into CNS, muscle, eye, and subcutaneous tissues. Cysticercal infection of muscles produces a nonspecific myositis. Brain involvement may lead to seizures. The lesions in the brain may calcify and often appear as multiple small cysts spread throughout the cerebrum. Treatment of neurocysticercosis is controversial. The best effort should be made to find the carrier of the tapeworm (possibly the patient himself) and treat with antiparasitics.

A 5-year-old girl has frequent staring spells and does not respond when her mother calls her name during these episodes. She never falls down or bites her tongue, but she does have occasional lip smacking during episodes. EEG reveals a 3 Hz spike-and-wave pattern that occurs for less than 10 seconds at a time but several times an hour. The child has normal motor and cognitive development.

The answer is g. (Ropper, pp 307-308.) This girl has generalized absence attacks. This may be a manifestation of a more complex epilepsy syndrome or may occur as an isolated finding. Generalized absence attacks have no aura and no postictal period. The affected child has no warning that an attack is about to occur and is usually unaware that one has occurred unless it is more than a few seconds long. In fact, generalized absence seizures are most often only a few seconds long. Ethosuximide is the drug of choice, but it may cause gastrointestinal distress. Divalproex sodium is effective in many of the children who cannot tolerate ethosuximide or who are not well controlled on that antiepileptic. If the absence seizures are associated with generalized tonic-clonic seizures, divalproex sodium is a better choice. Some antiepileptic drugs, such as the sodium channel blockers phenytoin and carbamazepine, can actually worsen generalized from onset seizures.

A 71-year-old man develops headache and slight difficulty speaking while having sexual intercourse. He has a long-standing history of hypertension, but has been on medication for more than 7 years. He makes frequent errors in finding words and follows complex commands somewhat inconsistently. The most obvious defect in his language function is his inability to repeat the simplest of phrases without making repeated errors. An emergency CT scan reveals an intracerebral hemorrhage in the left parietal lobe that appears to communicate with the lateral ventricle.

The answer is g. (Ropper, pp 466-470.) According to one classic model of language organization formulated by the neurobehaviorist Norman Geschwind, the expressive language centers in the frontal lobe and the receptive centers in the temporal lobe communicate in large part along the arcuate fasciculus, which extends through the temporal and parietal lobes. This man appears to have suffered an acute hemorrhage associated with chronic hypertension. The blood extended into the lateral ventricle, which was the probable cause of the headache. Patients with the rare syndrome of conduction aphasia have problems with repetition that are more obvious than their problems with comprehension. Their speech usually does not sound very fluent.

A 29-year-old man relates that he has had recent headaches only when standing up. The headaches resolve quickly when he lies down and are accompanied by mild nausea. His examination is normal.

The answer is h. (Ropper, pp 606-607.) Headaches that occur on standing indicate the presence of intracranial hypotension. Most often, this is the result of recent lumbar puncture, either for diagnostic purposes or after spinal anesthesia. The hole in the dura created by the spinal tap presumably allows fluid to continue leaking out, and this creates a condition of decreased pressure within the spinal canal, which causes traction on the pain-sensitive meninges of the brain. Other causes of intracranial hypotension include continued leak of cerebrospinal fluid (CSF) from the subarachnoid space after head trauma, neurosurgery, or even pneumonectomy (thoracoarachnoid fistula); occult pituitary tumor; a leak from a dural tear in the spinal root sleeves; traumatic nerve root avulsion; or systemic illness such as dehydration, diabetic coma, uremia, or meningoencephalitis. With leakage of CSF into nasal passages, the patient may complain of rhinorrhea. In some cases, no cause is apparent even after a thorough evaluation.

A 40-year-old man was involved in an automobile accident. There is an obvious laceration on his head, and he has neck pain. Police at the scene report that he was unconscious when they arrived, but the patient cannot recall this loss of consciousness. In fact, he cannot remember the accident or events within 10 minutes prior to the accident. On examination, he has obvious neck stiffness and photophobia. Within a few hours of his arrival at the emergency room, he develops vomiting. Lumbar puncture is delayed until after an MRI can be obtained. The tap is performed 2 days after the accident because the patient is still confused and irritable.

The answer is g. (Ropper, pp 811-812 .) This man, involved in an automobile accident, probably has subarachnoid blood associated with his head trauma. This is suggested by his neck stiffness, photophobia, and vomiting. That he had transient loss of consciousness and that there was obvious trauma to his head support the notion that he sustained enough of a blow to his head to produce intracranial bleeding of some sort. Even if the neuroimaging studies do not reveal any contusion, he could still have a substantial accumulation of blood in the subarachnoid space from damage to vessels in the arachnoid itself. A high CSF protein content and xanthochromia suggest that much of the blood in the CSF has already broken down by the time of the tap. Many RBCs will persist for days with a substantial subarachnoid hemorrhage. The WBC count will be elevated because the subarachnoid blood is irritating and produces chemical meningitis. The opening pressure may be slightly elevated if there has been much bleeding into the subarachnoid space.

A 78-year-old man has a cardiac arrest while being treated in an emergency room for chest pain. Resuscitation is initiated immediately, but profound hypotension is observed for at least 20 minutes. A cardiac rhythm is restored, but the patient remains unconscious for the next 3 days. When he is awake, alert, and extubated, his speech is limited to repetition of words and sounds produced by those around him. He has no apparent comprehension of language and produces few sounds spontaneously. Whenever the patient is spoken to, he fairly accurately repeats what was said to him.

The answer is h (mixed transcortical aphasia). (Ropper, pp 466-470.) With protracted hypotension, this patient suffered a watershed infarction. The cortex at the limits of the supply of the principal cerebral arteries was inadequately perfused, and the resulting infarction isolated the speech areas in the frontal and temporal lobes from the cortex in other parts of the cerebrum. Language usually does not recover substantially after this type of infarction.

A 37-year-old female navy officer presents with 3 days of confusion and seizures. Her colleagues report that she has been acting strangely for 3 days. This is followed by generalized status epilepticus. The woman has previously been well. She has traveled to the Caribbean several times annually, and she has a new pet cat. General examination discloses epitrochlear lymphadenopathy. Neurological examination shows the woman to be in status epilepticus. CSF is negative; MRI shows increased signal in the pulvinar bilaterally.

The answer is h. (Fauci, p 990.) Cat-scratch disease produces a regional adenitis, frequently involving epitrochlear nodes caused by scratches on the patient's arm from an infected animal. The causative agent is B henselae (rarely Afipia felis). In immunocompetent hosts, it may produce a selflimited aseptic meningitis. In HIV-infected individuals, it may produce a more virulent encephalitis associated with status epilepticus. Typically, these patients have disseminated disease, including distinctive skin lesions composed of neovascular proliferation (bacillary angiomatosis). On rare occasions, immunocompetent patients may have encephalitis as well. An MRI may show a characteristic increased signal intensity in the pulvinar, suggesting a tropism of the organism or immune response to this particular structure in the posterior thalamus.

A 9-month-old girl from famine-stricken Ethiopia exhibits profound apathy and indifference to her environment. She is afebrile and appears to have no significant infections at the time of her initial evaluation. Her hair is sparse, and slight edema is evident about her ankles. She is well below the fifth percentile for height in her age group. With handling she becomes irritable, but throughout her examination she exhibits little spontaneous movement. Her mother reports having seen transient tremors in the girl's hands a few weeks earlier, but these abated after a few days.

The answer is h. (Ropper, p 1108 .) Protein deficiency states, such as those occurring with kwashiorkor, produce a wide range of neurological signs and symptoms. Although the CNS is somewhat sheltered from the ravages of malnutrition, severe protein-calorie deficiencies during childhood development may leave the child neurologically impaired for life. Even when dietary supplements have been introduced to correct the chronic deficiency, the children are likely to exhibit little improvement in mobility or alertness for weeks or months.

A 13-year-old girl has headaches and diplopia. On examination, she has impaired upward gaze, lid retraction, and convergence-retraction nystagmus. Her pupils react on convergence but not to light.

The answer is h. (Ropper, p 641.) Pineocytomas are histologically benign lesions affecting the region of the pineal gland. They arise from the parenchymal cells of the pineal gland. This patient's symptoms and signs constitute Parinaud syndrome, which may include loss of vertical gaze, loss of pupillary light reflex, lid retraction, and convergence-retraction nystagmus, in which the eyes appear to jerk back into the orbit on attempted upgaze. This syndrome occurs in lesions owing to involvement of the dorsal midbrain in the region of the superior colliculus. Other tumors appearing in the pineal region that can produce a similar clinical picture include germ cell tumors (germinomas), teratomas, and gliomas. Malignant pineal tumors, or pineoblastomas, may also occur, and are similar histologically to medulloblastomas.

A 21-year-old cocaine-abusing man develops seizures that persist for more than 30 minutes before emergency medical attention is available. When examined nearly 1 hour later, he is still exhibiting tonic-clonic movements and has never recovered consciousness.

The answer is h. (Ropper, pp 304, 307, 332-334.) Status epilepticus is defined as a seizure that lasts continuously for 30 minutes or a series of seizures over a 30-minute period without the patient regaining full consciousness between them. Status epilepticus constitutes a medical emergency, because the longer the seizures last, the worse the morbidity and mortality. Complications of status epilepticus include respiratory failure, aspiration, acidosis, hypotension, rhabdomyolysis, renal failure, and cognitive impairment.

A 65-year-old man with a history of atrial fibrillation is brought into the emergency room at 1:00 PM because of the acute onset of right-sided weakness and inability to speak beginning at noon. On examination, he is alert but unable to speak. He follows simple one-step commands. There is left gaze deviation and impaired rightward gaze. Flaccid paresis of the right face and arm is present, but he is able to lift his right leg off the bed. Reflexes are decreased on the right side.

The answer is i. (Fauci, pp 2489-2491.) This man has suffered an acute cerebrovascular event. The most important test in the immediate period is a CT scan of the head to establish whether the lesion is hemorrhagic or ischemic. If there is no evidence of hemorrhage on head CT, then the patient is within the 3-hour time window permitting therapy with intravenous recombinant tissue plasminogen activator (r-TPA), which has been shown to improve functional outcome after stroke.

A 30-year-old man with AIDS develops headaches and left hemiparesis and is found to have a right frontal white matter homogeneously enhancing lesion.

The answer is i. (Primary CNS Lymphoma)(Ropper, pp 630-631.) Patients with AIDS are at risk for numerous CNS infections, but have an increased frequency of only two tumor types: lymphoma and Kaposi sarcoma (KS). KS may metastasize to the CNS, but lymphoma is routinely primary to the CNS. This tumor may produce blindness through direct invasion of the optic nerve.

A 7-month-old boy develops generalized limb extension and neck flexion spasms that occur more than 20 times daily and are associated with altered consciousness. EEG reveals diffuse, high-voltage, polyspike-and-slow-wave discharges between spasms and suppression of these bursts during the spasms. A sibling died with a brainstem glioma, and the father has several large areas of hypopigmented skin in the shape of ash leaves. The infant had obvious psychomotor retardation even before the appearance of the spasms.

The answer is i. (Ropper, pp 313-314.) This child has West syndrome, a generalized seizure disorder of infants characterized by recurrent spasms, the EEG pattern of hypsarrhythmia, and retardation. Several different diseases cause West syndrome. The family history in this case suggests tuberous sclerosis as the underlying problem. ACTH is the best of the given choices.

A previously healthy 25-year-old woman develops acute loss of vision in her left eye. She awakens with pain in the eye and reduction of her acuity to perception of light and dark. She delays seeing a physician for 1 week, during which time her acuity gradually improves sufficiently to allow her to read. On examination, the physician discovers she has slurred speech and poor rapid alternating movements with the left hand. Ocular dysmetria is evident in both eyes. Her tandem gait is grossly impaired. The physician obtains an EEG, which is normal.

The answer is i. (Ropper, pp 874-896.) Optic neuritis is often the first symptom that motivates the patient with multiple sclerosis to consult a physician. Clumsiness, stumbling, and other symptoms of ataxia are usually dismissed as inconsequential by the patient. Even those with profoundly slow and slurred speech are often unaware of their dysarthria. When the patient finally does consult a physician, multiple neurological abnormalities are usually evident. This patient would be expected to have a positive swinging flashlight test (Marcus Gunn pupil) and evidence of widespread demyelination on MRI of the head.

A 65-year-old woman develops pain and paresthesias in her feet. On examination, she has loss of reflexes, stocking distribution sensory loss, and mild distal weakness. Serum protein electrophoresis reveals a monoclonal gammopathy, and bone marrow biopsy reveals plasma cell dyscrasia.

The answer is i. (paraproteinemic neuropathy) (Ropper, pp 1289-1290.) Polyneuropathy may occur in up to 15% of patients with multiple myeloma. This generally takes the form of a chronic distal symmetrical sensory or sensorimotor neuropathy. In some cases, the neuropathy progresses more aggressively, and the patient becomes confined to a wheelchair. Spinal fluid protein may be elevated, and the illness has the appearance of a chronic inflammatory demyelinating polyneuropathy. Up to 20% of patients referred for evaluation of polyneuropathy may have an underlying monoclonal paraproteinemia. In the absence of an obvious malignancy, this is called a monoclonal gammopathy of undetermined significance, but a hematologic malignancy may later turn up in as many as one-third of such patients.

An 81-year-old man with chronic lymphocytic leukemia develops pain and burning over the right side of his face. Within a few days, a vesiculopapular rash in the distribution of the first division of the trigeminal nerve appears. The vesicles become encrusted, and the burning associated with the rash abates. Within 1 month the rash has largely resolved, but the man is left with a dull ache over the area of the rash that is periodically punctuated by shooting pains. Imipramine 100 mg nightly helps reduce the intensity of the chronic pain.

The answer is j (postherpetic neuralgia). (Ropper, pp 183-184.) The rash preceding the facial pain was probably caused by herpes zoster, a virus that erupts in the severely ill elderly and in immunosuppressed persons. The virus is manifested earlier in life as chickenpox and remains dormant for decades in most people. Tricyclic drugs, such as imipramine hydrochloride, are often more useful than analgesics in suppressing the pain associated with this postviral syndrome.

A 35-year-old woman is found unconscious on the floor of her apartment. A bottle of cleaning fluid is found on a table near her. One of the contents indicated in the fluid is carbon tetrachloride. The ambulance crew notes that the patient is breathing independently, but her breath has a distinctly fetid odor unlike that associated with the cleaning fluid. Her limbs are flaccid, and she groans when she is moved. She does not respond to inquiries and is poorly responsive to pain. A serum ammonia level obtained at the emergency room is 250 mg/dL, triple the normal level. EEG reveals triphasic waves, most prominently over the front of the head.

The answer is j. (Ropper, pp 1090-1093.) Carbon tetrachloride is a potent hepatic toxin. This woman may have attempted to commit suicide by drinking cleaning fluid. As hepatic damage progressed, she developed fetor hepaticus, a distinctive smell to her breath that reflects a profound metabolic disturbance. The serum ammonia level rose as liver function declined. The triphasic waves typically seen in hepatic encephalopathy may occur with uremia and other causes of metabolic encephalopathy.

A 17-year-old boy reports involuntary jerking movements in his arms when he awakens. This has occurred during the day after a nap as well as in the morning after a full night's sleep. Over the next few months, he developed similar jerks during the day, even when he had been awake for several hours. He did not lose consciousness with these muscle jerks, but did occasionally fall. On one occasion, jerks in his legs caused a fall that resulted in a fractured wrist.

The answer is j. (Ropper, pp 97, 98, 308.) Myoclonic seizures may be generalized or partial. They are most commonly seen in the epilepsy syndrome called benign juvenile myoclonic epilepsy (BJME). Unlike sleep myoclonus, the episodes occur when the affected person wakes up rather than when he or she is falling asleep. Myoclonic jerks may be triggered by light flashes or loud sounds. Benign juvenile myoclonic epilepsy accounts for 4% of all cases of epilepsy. More than half of those with BJME have generalized tonic-clonic seizures as well as myoclonic seizures.

A 17-year-old man has headache and photophobia on awakening. His physician discovers a low-grade fever and resistance to neck flexion. The physician advises the patient to take acetaminophen and remain in bed for the next 24 hours. Within 12 hours, the patient develops nausea and more intense headache. He seems disoriented and inappropriately lethargic. His family brings him to an emergency room. The emergency room physician notes a petechial rash on the legs and marked neck stiffness. CSF examination reveals a glucose content of 5 mg/dL, protein content of 87 mg/dL, and cell count of 112 leukocytes, with 70% polymorphonuclear cells.

The answer is l. (Ropper, p 673.) With acute bacterial meningitis, the time between the first symptoms and death may be only days. A petechial rash developing over the lower parts of the body in the setting of fever, headache, nuchal rigidity, photophobia, and stupor must be considered presumptive evidence of a meningococcal meningitis. Rapid diagnosis and treatment are essential if the patient is to survive. The spinal fluid typically reveals a low glucose content, high protein content, and leukocytosis with a large number of polymorphonuclear cells. Treatment with intravenous penicillin G, 12 million-to-15 million U daily (divided into four to six doses), early in the course of illness may decide whether the patient survives more than a few hours or days.

A 57-year-old woman with a history of smoking has a 3-month history of hip and shoulder weakness. She also complains of xerostomia. There are no sensory symptoms, and she is cognitively intact. On examination, she is orthostatic. There is proximal muscle weakness, but she has increasing muscle strength with repetitive activity of her muscles. Eye movements are normal.

The answer is l. (Ropper, pp 1417-1418.) The Lambert-Eaton myasthenic syndrome (LEMS) shares some features with myasthenia gravis, notably proximal muscle weakness. It usually develops subacutely, however, and spares the bulbar musculature and eyes. There is also little response to anticholinesterase drugs. A characteristic feature is the increase in strength briefly after repeated muscle activation. Most cases are associated with an underlying oat cell carcinoma of the lung or other malignancy. In other cases, LEMS may be associated with other autoimmune illness. The underlying defect is the loss of function of the voltage-sensitive calcium channels in the presynaptic nerve terminal at the neuromuscular junction, attributed to cross-linking and aggregation by pathologic IgG autoantibodies. Various immune-modulating therapies, as well as 3,4-diaminopyridine, have been used with varying success. Removal of the underlying malignancy may also be curative.

A 23-year-old woman with a history of hemophilia notices progressive memory difficulty. She has required little hematologic support, but she did receive transfusion of factor VIII at least five times over the past 7 years. Neurological examination reveals word-finding difficulty, poor recent and remote memory, gait ataxia, mild dysarthria, and alabile affect. Her right plantar response is extensor, and her left brachioradialis reflex is hyperactive with transient clonus. An MRI of the brain is unrevealing.

The answer is n. (Ropper, p 717.) This woman is at relatively high risk for AIDS encephalopathy because she has required transfusion of clotting factors that have until recently been available only from pooled samples of blood products. The neurological deficits that she exhibits are not specific for HIV-1-associated subacute encephalomyelitis (AIDS encephalitis) and are quite compatible with multiple sclerosis (MS). That her MRI does not reveal plaques of demyelination scattered throughout the brain makes the diagnosis of MS improbable. To establish the diagnosis of AIDS encephalopathy, HIV-1 antibodies should be sought and the helper/suppressor (CD4/CD8) Tlymphocyte ratio should be checked. Patients with symptomatic AIDS usually have a CD4/CD8 Tlymphocyte ratio of less than 0.5.

A 35-year-old businessman has sleep attacks. He runs a chain of dry-cleaning stores, but does not usually work with the cleaning fluids. He reports falling asleep several times during the workday, even at business meetings and during interviews. He has developed the sleep attacks only after gaining more than 100 lb. His weight at the time of the examination is 324 lb.

The answer is o. (Ropper, p 390.) Obesity associated with hypersomnia qualifies as pickwickian syndrome if the patient exhibits other characteristic features, such as sleep apnea. The patient with this syndrome is likely to have hypoxemia and pulmonary hypertension. Smoking increases the risk of developing the syndrome. Sleep attacks usually abate with cessation of smoking and weight loss.


संबंधित स्टडी सेट्स

Similarity and Proportions Retake Guide

View Set

Unit 7_STATISTICAL QUALITY CONTROL

View Set

BIO 220 - Chapter 43 - HW and Quiz

View Set

Chapter 7: Portable fire extinguishers

View Set

Lewis Ch. 23 - Integumentary Problems, Lewis - Med-Surg Nursing - Study Guide - Ch. 11, alterations in skin integrity Questions

View Set

Module 13 Chapter 23 Quiz Natural Selection and Evolution

View Set